Case-based Atlas of Cardiac Imaging (Jan 3, 2024)_(9819956196)_(Springer) 9789819956197, 9789819956203

125 50

english Pages [591] Year 2024

Report DMCA / Copyright

DOWNLOAD FILE

Polecaj historie

Case-based Atlas of Cardiac Imaging (Jan 3, 2024)_(9819956196)_(Springer)
 9789819956197, 9789819956203

Table of contents :
Preface
Contents
About the Editor
1: Chest Radiograph in Heart Disease
1.1 Contour of the Heart
1.1.1 Postero-anterior (PA) View (Fig. 1.2)
1.1.2 Lateral View (Fig. 1.3)
1.1.3 Situs Evaluation
1.1.4 Heart Size
1.2 Radiological Signs of Enlarged Cardiac Chambers
1.2.1 Right Atrium (RA)
1.2.2 Left Atrium (LA)
1.2.3 Left Ventricle (LV)
1.2.4 Right Ventricle (RV)
1.2.5 Aorta
1.2.6 Lung Vasculature
1.2.7 Normal Radiographic Anatomy
1.2.8 Increase in Lung Vascularity (Pulmonary Plethora)
1.2.9 Decreased Pulmonary Blood Flow (Pulmonary Oligemia)
1.2.10 Pulmonary Arterial Hypertension (PAH)
1.2.11 Pulmonary Venous Hypertension (PVH)
1.2.12 Asymmetrical Pulmonary Blood Flow or Pleonemia (Fig. 1.38)
1.3 Algorithmic Approach to Localization of Congenital Heart Disease by Chest Radiograph
1.3.1 Left-to-Right (L-R) Shunt
1.3.2 Large Aorta
1.3.3 RSOV [2]
1.3.4 Aorto-pulmonary Window
1.3.5 Coronary Arteriovenous Fistula
1.3.6 Normal Aorta
1.3.7 Pre-tricuspid Shunt
1.3.8 ASD
1.3.9 PAPVC
1.3.10 L-R Shunt Beyond the Tricuspid Valve
1.3.11 Right-to-Left (R-L) Shunt
1.3.12 Large Aorta with Decreased Lung Vascularity
1.3.13 TOF
1.3.14 Pulmonary Atresia with VSD
1.3.15 Tricuspid Atresia
1.3.16 Single Ventricle with PS
1.4 Normal to Small Aorta with Increased Vascularity
1.4.1 D-TGA [5]
1.4.2 Large Pedicle with Increased Vascularity
1.4.3 Truncus Arteriosus (TA)
1.4.4 TAPVC
1.4.5 Normal Aorta with Decreased Lung Vascularity
1.4.6 Ebstein’s Anomaly
1.4.7 Valvular PS
1.5 Other Conditions
1.5.1 Hypoplastic Left Heart Syndrome
1.5.2 Coarctation of Aorta
1.5.3 Congenital Absence of Pericardium
1.5.4 Rheumatic Heart Disease
1.5.5 EMF
1.5.6 Chronic Constrictive Pericarditis (CCP)
1.5.7 LV Aneurysms
References
Part I: Congenital Heart Diseases
2: Algorithmic Approach to Imaging Diagnosis of Congenital Heart Disease
3: Imaging in Tetralogy of Fallot
3.1 Case 3.1
3.1.1 Clinical Presentation
3.1.2 Echo
3.1.3 What Investigation Will You Do Next?
3.1.3.1 Goals of Imaging in CHD
3.1.4 Morphologic Features
3.1.5 Functional Parameters
3.1.6 What Are the Imaging Findings?
3.1.7 Final Diagnosis
3.2 Case 3.2
3.2.1 Final Diagnosis
3.3 Case 3.3
3.3.1 Final Diagnosis
3.3.2 Differential Diagnosis of Tetralogy of Fallot
3.4 Case 3.4
3.4.1 Final Diagnosis
3.4.2 Differentiating Features from Classical TOF
3.5 Case 3.5
3.5.1 Final Diagnosis
3.5.2 Differentiating Features from Classical TOF
3.6 Case 3.6
3.6.1 Final Diagnosis
3.6.2 Differentiating Features from Classical TOF
3.7 Case 3.7
3.7.1 Final Diagnosis
3.7.2 Differentiating Features from Classical TOF
3.8 Case 3.8
3.8.1 Final Diagnosis
3.8.2 Differentiating Features from Classical TOF
3.9 Tetralogy of Fallot
3.9.1 Introduction
3.9.2 Embryology and Pathoanatomy of TOF
3.9.3 Genetic Abnormalities
3.9.4 Associated Anatomical Abnormalities
3.9.5 Imaging in Tetralogy of Fallot
3.9.6 Spectrum of Imaging Abnormalities in TOF
3.10 Case 3.9
3.10.1 Final Diagnosis
3.11 Case 3.10
3.11.1 Final Diagnosis
3.12 Case 3.11
3.12.1 Final Diagnosis
3.13 Case 3.12
3.13.1 Final Diagnosis
3.14 Case 3.13
3.14.1 Final Diagnosis
3.15 Case 3.14
3.15.1 Final Diagnosis
3.16 Case 3.15
3.16.1 Final Diagnosis
3.17 TOF: Spectrum of Branch PA Involvement
3.17.1 Case 3.16: LPA Atresia
3.17.2 Case 3.17: LPA Ostial Stenosis
3.17.3 Case 3.18: Diffuse LPA Stenosis Proximal to PDA Insertion Site with Post-stenotic Dilatation
3.17.4 Case 3.19
3.17.4.1 Final Diagnosis
3.17.5 Case 3.20
3.17.5.1 Final Diagnosis
3.17.6 Case 3.21
3.17.6.1 Final Diagnosis
3.17.7 Case 3.22
3.17.7.1 Final Diagnosis
3.17.8 Case 3.23: Pentalogy of Fallot (TOF with ASD)
3.17.8.1 Final Diagnosis
3.18 TOF with Coronary Anomalies
3.18.1 Case 3.24: Hypertrophied Conal Artery Crossing the Infundibulum
3.18.2 Case 3.25: LAD—Common Origin with RCA from Right Coronary Sinus
3.18.3 Case 3.26: LM—Common Origin with RCA from Right Coronary Sinus
3.19 TOF with Aortopulmonary Collaterals
3.19.1 Criteria for Significant APC
3.19.2 Prerequisites for Embolization of APC
3.19.3 Case 3.27: Significant Systemic to PA Collaterals
3.20 How Imaging Impacts Management?
3.21 Special Situations in TOF Surgery
Appendix 1: Imaging Protocol
CT Angiography Imaging Protocol
Cardiac MRI Protocol
Catheter Angiography for Evaluation of TOF
Flow Rate for Ventriculogram
Flow Rate for Aortic Root Angiogram
Appendix 2: Reporting Template
CTA Angiography Reporting Template
Cardiac MR Reporting Template
References
4: Imaging in Double-Outlet Right Ventricle
4.1 Case 4.1
4.1.1 Chest X-Ray
4.1.2 What Is the Next Investigation?
4.1.3 What Are the Imaging Findings
4.1.3.1 Final Diagnosis
4.2 Case 4.2
4.2.1 Echo
4.2.2 What Are the Imaging Findings
4.2.2.1 Final Diagnosis
4.2.2.2 Differential Diagnosis
4.3 Case 4.3
4.3.1 Final Diagnosis
4.4 Case 4.4
4.4.1 Impression
4.4.2 Single Ventricle Physiology of RV Morphology, PS
4.5 Double-Outlet Right Ventricle
4.5.1 Introduction
4.5.2 Classification
4.5.3 Associations
4.5.4 Goals of Imaging and Role of Various Imaging Modalities
4.5.5 How Imaging Impacts Management?
4.5.6 Controversies
4.5.6.1 Future Perspective: 3D Printing
Appendix 1: Imaging Protocol
Appendix 2: Reporting Template
5: Imaging in Tricuspid Atresia
5.1 Case 5.1
5.1.1 Chest Radiograph
5.1.2 Differential Diagnosis
5.1.3 Echo
5.1.4 What Investigation Will You Do Next?
5.1.5 What Are the Imaging Findings
5.1.6 Case Summary
5.1.6.1 Differential Diagnosis of Tricuspid Atresia: Table 5.1
5.2 Case 5.2
5.2.1 Final Diagnosis
5.3 Case 5.3
5.3.1 Final Diagnosis
5.4 Case 5.4
5.4.1 Final Diagnosis
5.5 Spectrum of Imaging Abnormalities in Tricuspid Atresia
5.6 Case 5.5
5.6.1 Final Diagnosis
5.7 Case 5.6
5.7.1 Final Diagnosis
5.8 Case 5.7
5.8.1 Final Diagnosis
5.9 Case 5.8
5.9.1 Final Diagnosis
5.10 Case 5.9
5.10.1 Final Diagnosis
5.11 Tricuspid Atresia
5.11.1 Introduction
5.11.2 Embryology and Pathoanatomy
5.11.3 Classification
5.12 Imaging in Tricuspid Atresia
5.13 How Imaging Impacts Management?
5.13.1 Medical Management
5.13.2 Surgical Management
5.13.3 First Stage (Neonates)
5.13.3.1 Type I (Tricuspid Atresia with Normally Related Great Artery)
5.13.3.2 Type II (Tricuspid Atresia with Transposition)
5.13.4 Second (3–6 Months) and Third Stage (2–5 Years)
Appendix 1: Imaging Protocol
Appendix 2: Reporting Template
References
6: Imaging in Transposition of Great Arteries
6.1 Case 6.1
6.1.1 Case Presentation
6.1.2 Differential Diagnosis
6.1.3 What Investigation Will You Do Next?
6.1.4 Final Diagnosis
6.2 Case 6.2
6.2.1 Case Presentation
6.2.2 Final Diagnosis
6.3 Case 6.3
6.3.1 Case Presentation
6.3.2 Final Diagnosis
6.4 Case 6.4
6.4.1 Case Presentation
6.4.2 Final Diagnosis
6.5 Case 6.5
6.5.1 Case Presentation
6.5.2 Final Diagnosis
6.6 Transposition of Great Arteries
6.6.1 Introduction
6.6.2 Embryology and Pathoanatomy
6.6.3 Genetic Abnormalities
6.6.4 Associated Anatomical Abnormalities
6.6.5 Role of Imaging in dTGA
6.6.6 Clinical Presentation
6.6.7 Imaging Findings
6.6.8 How Imaging Impacts the Management
Appendix 1: Imaging Protocol
Appendix 2: Reporting Template
References
7: Imaging in Truncus Arteriosus
7.1 Case 7.1
7.1.1 Chest X-Ray
7.1.2 Differential Diagnosis
7.1.3 Echo
7.1.4 What Investigation Will You Do Next?
7.1.5 What Are the Imaging Findings
7.1.5.1 Final Diagnosis
7.1.5.2 Differential Diagnosis
TAPVC: Cardiac Type
7.2 Truncus Arteriosus
7.2.1 Introduction
7.2.2 Embryology and Pathoanatomy of TAPVC
7.2.2.1 Genetic Abnormalities
7.2.3 Clinical Findings
7.2.4 Imaging
7.3 Spectrum of Imaging Abnormalities in Truncus Arteriosus
7.4 Case 7.2
7.4.1 What Are the Imaging Findings
7.4.1.1 Final Diagnosis
7.5 Case 7.3
7.5.1 What Are the Imaging Findings
7.5.1.1 Final Diagnosis
7.6 Case 7.4
7.6.1 What Are the Imaging Findings
7.6.1.1 Final Diagnosis
7.7 Case 7.5
7.7.1 What Are the Imaging Findings
7.7.1.1 Final Diagnosis
7.7.2 How Imaging Impacts Management [6]:
7.7.3 Controversies and Future Perspectives
Appendix 1: Imaging Protocol
Appendix 2: Reporting Template
References
8: Imaging in Total Anomalous Pulmonary Venous Connections and Partial Anomalous Pulmonary Venous Connections
8.1 Case 8.1
8.1.1 Chest X-Ray
8.1.2 Differential Diagnosis
8.1.3 Echo
8.1.4 What Investigation Will You Do Next?
8.1.5 What Are the Imaging Findings?
8.1.6 Final Diagnosis
8.2 Case 8.2
8.2.1 What Are the Imaging Findings?
8.2.2 Final Diagnosis
8.3 Total (Complete) Anomalous Pulmonary Venous Return
8.3.1 Introduction
8.3.2 Embryology and Pathoanatomy of TAPVC
8.3.3 Clinical Findings
8.3.4 Associated Anatomical Abnormalities
8.3.5 Imaging
8.4 Spectrum of Imaging Abnormalities in Anomalous Pulmonary Venous Connections
8.5 Case 8.3
8.5.1 What Are the Imaging Findings?
8.5.2 Final Diagnosis
8.6 Case 8.4
8.6.1 What Are the Imaging Findings?
8.6.2 Final Diagnosis
8.7 Case 8.5
8.7.1 What Are the Imaging Finding?
8.7.2 Final Diagnosis
8.8 Partial Anomalous Pulmonary Venous Return
8.8.1 Introduction
8.8.2 Pathoanatomy
8.8.3 Clinical Features
8.8.4 Variations: Scimitar Syndrome
8.9 Spectrum of Partial Anomalous Systemic Venous Drainage
8.10 Case 8.6
8.10.1 Chest X-Ray
8.10.2 Echo
8.10.3 What Are the Imaging Findings?
8.10.4 Final Diagnosis
8.11 Case 8.7
8.11.1 What Are the Imaging Findings?
8.11.2 Final Diagnosis
8.11.3 How Imaging Impacts the Management
8.11.4 Indications and Timing of Surgery (All Are Class I Recommendations)
8.11.5 Important Determinants of Long-Term Prognosis
8.11.6 Recommendations for Follow-Up
Appendix 1: Imaging Protocol
Appendix 2: Reporting Template
References
9: Imaging in Ebstein’s Anomaly
9.1 Case 9.1
9.1.1 Chest X-Ray
9.1.1.1 Echo
9.1.1.2 Diagnosis
9.1.2 What Investigation Will You Do Next?
9.1.3 What Are the Imaging Findings?
9.1.4 Final Diagnosis
9.2 Case 9.2
9.2.1 Final Diagnosis
9.2.2 Differential Diagnosis
9.3 Case 9.3
9.3.1 Echo
9.3.2 Final Diagnosis
9.3.3 Differentiating Features from Ebstein’s
9.4 Case 9.4
9.4.1 Echo
9.4.2 Diagnosis
9.5 Case 9.5
9.5.1 Echo
9.5.2 Final Diagnosis
9.6 Case 9.6
9.6.1 Echo
9.6.2 Final Diagnosis
9.6.3 Differentiating Features from Ebstein’s
9.7 Case 9.7
9.7.1 Final Diagnosis
9.7.2 Differentiating Features from Ebstein’s
9.8 Case 9.8
9.8.1 Final Diagnosis
9.8.2 Differentiating Features from Ebstein’s
9.9 Ebstein’s Anomaly
9.9.1 Introduction
9.9.2 Embryology and Pathoanatomy
9.9.3 Classification
9.9.4 Etiology
9.9.5 Associated Anatomical Abnormalities
9.9.6 Left Ventricular Involvement
9.9.7 Imaging
9.9.8 Variations of Ebstein’s Anomaly
9.10 Case 9.9
9.10.1 Final Diagnosis
9.11 Case 9.10
9.11.1 What Are the Imaging Findings?
9.11.2 Final Diagnosis
9.12 Case 9.11
9.12.1 What Are the Imaging Findings?
9.12.2 Final Diagnosis
9.13 Case 9.12
9.13.1 Final Diagnosis
9.14 Case 9.13
9.14.1 Echo
9.14.2 What Are the Imaging Findings?
9.14.3 Final Diagnosis
9.15 Case 9.14
9.15.1 ECG
9.15.2 Final Diagnosis
9.16 Case 9.15
9.16.1 Echo
9.16.2 Final Diagnosis
9.17 How Imaging Impacts the Management
9.17.1 Neonates
9.17.2 Children and Adults
9.17.3 Medical Management
9.17.4 Adjuvant Procedures
9.17.4.1 Anti-arrhythmic Measures
9.17.5 Management of Index Case
9.17.6 Controversies and Future Directions
Appendix 1: Imaging Protocol
Appendix 2: Reporting Template
References
10: Imaging in Hypoplastic Left Heart Syndrome
10.1 Case 10.1
10.1.1 Clinical Presentation
10.1.2 Investigations
10.1.2.1 Chest X-Ray
10.1.2.2 Echo
10.1.3 What Investigation Will You Do Next?
10.1.3.1 Goals of Orthogonal Imaging in This Case
10.1.4 What Are the Imaging Findings?
10.1.4.1 Final Diagnosis
10.2 The Spectrum of HLHS
10.2.1 Imaging Attributes Essential for the Diagnosis of HLHS
10.3 Case 10.2
10.3.1 Clinical Presentation
10.3.2 Echo
10.3.3 What Investigation Will You Do Next?
10.3.4 What Are the Imaging Findings?
10.3.4.1 Final Diagnosis
10.3.4.2 Differential Diagnosis
10.4 Case 10.3
10.4.1 Clinical Presentation
10.4.2 Echocardiography
10.4.3 Final Diagnosis
10.4.4 Difference from HLHS
10.5 Case 10.4
10.5.1 Clinical Presentation
10.5.2 Chest X-Ray
10.5.3 Echocardiography
10.5.4 Final Diagnosis
10.5.5 Difference from HLHS
10.6 Case 10.5
10.6.1 Clinical Presentation
10.6.2 Final Diagnosis
10.6.3 Difference from HLHS
10.7 Hypoplastic Left Heart Syndrome
10.7.1 Introduction
10.7.2 Pathophysiology of HLHS
10.7.3 Genetic and Other Abnormalities
10.7.4 Clinical Features
10.7.5 Associations
10.7.6 Imaging in HLHS
10.8 How Imaging Impacts the Management [3]
10.8.1 Goals of Three-Staged Surgical Approach
10.8.1.1 Stages
Appendix 1: Imaging Protocol
Appendix 2: Reporting Template
References
11: Imaging in Ventricular Septal Defect
11.1 Case 11.1
11.1.1 Chest Radiography
11.1.2 What Is the Next Investigation?
11.1.3 What Should Be the Next Investigation?
11.1.4 What Are the Imaging Findings? Cardiac Catheterization Findings
11.1.5 Cardiac Angiogram
11.1.6 Final Diagnosis
11.2 Companion Cases of VSD
11.3 Case 11.2
11.4 Case 11.3
11.5 Case 11.4
11.6 Case 11.5
11.7 Case 11.6
11.8 Case 11.7
11.9 Case 11.8
11.10 Case 11.9
11.11 Case 11.10
11.12 Case 11.11
11.13 Case 11.12
11.14 Case 11.13
11.15 Case 11.14
11.16 Case 11.15
11.17 Case 11.16
11.18 Case 11.17
11.19 VSD
11.19.1 Definition
11.19.2 Classification
11.19.3 Can Also Be Classified as Follows
11.19.4 Epidemiology
11.19.5 Anatomy of Normal Interventricular Septum [2]
11.19.6 Embryology of Ventricular Septum
11.19.7 Natural History
11.19.8 Clinical Features
11.20 Imaging
11.20.1 Goals of Imaging
11.20.1.1 Anatomic Features
11.20.1.2 Hemodynamic Evaluation
11.20.2 Imaging Modalities
11.20.3 Angiographic Views for Demonstration of VSD (Refer to Appendix 2) [3]
11.20.4 Comparison Between Various Imaging Modalities for the Evaluation of VSD (Table 11.1)
11.20.4.1 VSD Routability
Criteria’s for VSD Routability (Fig. 11.21)
11.20.4.2 Factors Precluding VSD Routability
11.20.4.3 Subpulmonic VSD
11.20.4.4 Non-committed VSD
11.20.4.5 Imaging Algorithm [5, 6]
11.20.4.6 How Imaging Impacts Management [7–9]
11.20.4.7 Role of Cross-Sectional Imaging in Post-operative Period
11.20.4.8 Conclusion
Appendix 1: Imaging Protocol
CT Angiography Imaging Protocol
Cardiac MRI Protocol
Catheter Angiography Views
Appendix 2: Reporting Template
Cardiac MR Reporting Template
References
12: Imaging in Atrioventricular Canal Defects (AVSD)
12.1 Case 12.1
12.1.1 Clinical Presentation
12.1.2 Final Diagnosis
12.2 Case 12.2
12.2.1 Clinical Presentation
12.2.2 Final Diagnosis
12.3 Case 12.3
12.3.1 Clinical Presentation
12.3.2 Final Diagnosis
12.4 Discussion: Endocardial Cushion Defects
12.4.1 Introduction
12.4.2 Epidemiology
12.4.3 Classification
12.4.3.1 Anatomy of Atrioventricular Canal Defects
12.4.4 Clinical Presentation
12.4.5 Cardiac Computed Tomography (CT) in the Evaluation of AV Canal Defects
References
13: Imaging in Aortopulmonary Window
13.1 Case 13.1
13.1.1 Clinical Presentation
13.1.2 Differential Diagnosis
13.1.3 What Is the Next Investigation?
13.1.4 What Are the Imaging Findings in CTA?
13.1.5 Final Diagnosis
13.1.6 Differential Diagnosis: Truncus Arteriosus
13.2 Aortopulmonary Window
13.2.1 Introduction
13.2.2 Etiology
13.2.3 Natural History
13.2.4 Morphological Types [1]
13.2.5 Shunt Physiology
13.2.6 Associations [3]
13.2.7 Goals of Imaging
13.2.7.1 Anatomic Features
13.2.7.2 Hemodynamic Evaluation
13.2.8 Role of Various Imaging Modalities
13.2.9 Glossary: Spectrum of Cases of Aortopulmonary Window (APW)
13.3 Case 13.2
13.3.1 Clinical Presentation
13.3.2 Investigations
13.3.2.1 Chest Radiograph
13.3.2.2 Echo
13.3.3 What Investigation Will You Do Next?
13.3.4 What Are the Imaging Findings?
13.3.5 Final Diagnosis
13.4 Case 13.3
13.4.1 Clinical Presentation
13.4.2 Investigations
13.4.2.1 Chest Radiograph
13.4.2.2 ECG
13.4.2.3 Echo
13.4.3 Provisional Diagnosis
13.4.4 What Investigation Will You Do Next?
13.4.5 What Are the Imaging Findings?
13.4.6 Final Diagnosis
13.5 Case 13.4
13.5.1 Clinical Presentation
13.5.2 Investigations
13.5.2.1 Chest Radiograph
13.5.2.2 ECG
13.5.2.3 Echo
13.5.3 Provisional Diagnosis
13.5.4 What Investigation Will You Do Next?
13.5.5 What Are the Imaging Findings?
13.5.6 Final Diagnosis
13.6 Case 13.5
13.6.1 Clinical Presentation
13.6.2 Clinical Examination
13.6.3 Investigations
13.6.3.1 Chest Radiograph
13.6.3.2 ECG
13.6.3.3 Echo
13.6.4 Provisional Diagnosis
13.6.5 What Investigation Will You Do Next?
13.6.6 What Are the Imaging Findings?
13.6.7 Final Diagnosis
13.6.8 How Imaging Impacts Management [4–8]
13.6.8.1 Timing of Closure
13.6.8.2 Contraindication for Closure
13.6.8.3 Method of Closure [6–8]
13.6.8.4 Role of Imaging in Post-operative Period
13.6.9 Controversies
13.6.9.1 Future Perspective
3D Printing
References
14: Imaging in Sinus of Valsalva Aneurysm
14.1 Case 14.1
14.1.1 Case Presentation
14.1.2 Final Diagnosis
14.1.3 What Investigation Will We Do Next?
14.1.4 What Should Be the Next Investigation?
14.1.5 Diagnosis
14.2 Case 14.2
14.2.1 Clinical Presentation
14.2.2 Diagnosis
14.2.3 Final Diagnosis
14.3 Sinus of Valsalva Aneurysm
14.4 Imaging for SVA [4]
14.4.1 Goals of Imaging
14.4.1.1 Anatomic Features
14.4.1.2 Hemodynamic Evaluation
14.4.2 Imaging Modalities for Suspected Case of SVA
14.5 Spectrum of Cases
14.5.1 Case 14.3
14.5.1.1 Case Presentation
14.5.1.2 Diagnosis
14.5.2 Case 14.4
14.5.2.1 Case Presentation
14.5.2.2 Diagnosis
14.5.3 Case 14.5
14.5.3.1 Case Presentation
14.5.3.2 Diagnosis
14.5.4 Case 14.6
14.5.4.1 Case Presentation
14.5.4.2 Diagnosis
14.5.5 Case 14.7
14.5.5.1 Case Presentation
14.5.5.2 Diagnosis
14.5.6 Case 14.8
14.5.6.1 Case Presentation
14.5.6.2 Diagnosis
14.5.7 Case 14.9
14.5.7.1 Case Presentation
14.5.7.2 Diagnosis
14.6 Complications of SVA
14.7 Role of Imaging in Management of SVA
References
15: Imaging of Patent Ductus Arteriosus
15.1 Case 15.1
15.1.1 Case Presentation
15.1.2 Differential Diagnosis
15.1.3 Echo
15.1.4 What Is the Next Investigation?
15.1.5 What Are the Imaging Findings?
15.1.5.1 Diagnosis
15.2 Case 15.2
15.2.1 Clinical Presentation
15.2.2 Diagnosis
15.3 Case 15.3
15.3.1 Case Presentation
15.3.2 Diagnosis
15.4 Case 15.4
15.4.1 Case Presentation
15.4.2 Diagnosis
15.5 Patent Ductus Arteriosus
15.5.1 Introduction
15.5.2 Epidemiology
15.5.3 Embryology
15.5.4 Clinical Features
15.5.5 Associations
15.5.5.1 Ductus-Dependent Circulations
15.6 Goals of Imaging
15.6.1 Anatomic Features
15.6.2 Hemodynamic Evaluation
15.7 Imaging Modalities
15.7.1 Imaging in Management Planning
15.7.2 Guidelines for Closure of PDA According to Indian Guidelines [8]
15.7.2.1 Age for Closure of PDA
15.7.2.2 Closure of PDA
15.7.2.3 Contraindication for Closure
References
16: Imaging in Cor Triatriatum
16.1 Case 16.1
16.1.1 Clinical Presentation
16.1.2 Echo
16.1.3 What Investigation Will You Do Next?
16.1.4 What Are the Imaging Findings?
16.1.5 Final Diagnosis
16.2 Differential Diagnosis of COR Triatriatum
16.2.1 Mitral Supravalvular Ring
16.3 Cor Triatriatum
16.3.1 Introduction
16.3.2 Embryology and Anatomical Features of Cor Triatriatum Sinister
16.3.3 Embryology and Anatomical Features of Cor Triatriatum Dexter
16.3.4 Associated Anatomical Abnormalities
16.3.5 Symptomatology
16.4 Imaging
16.4.1 Goals of Imaging
16.4.1.1 Anatomic Features
16.4.1.2 Functional Evaluation
16.4.1.3 Hemodynamic Evaluation
16.4.2 Imaging Modalities
16.5 Spectrum of Imaging Abnormalities in Cor Triatriatum
16.6 Case 16.2
16.6.1 Clinical Presentation
16.6.2 Final Diagnosis
16.7 Case 16.3
16.7.1 Clinical Presentation
16.7.2 Final Diagnosis
16.8 Case 16.4
16.8.1 Clinical Presentation
16.8.2 Final Diagnosis
16.9 Management
16.10 Role of Percutaneous Balloon Dilatation [6, 7]
References
17: Imaging in Vascular Rings
17.1 Case 17.1
17.1.1 Differential Diagnosis
17.1.2 What Is the Next Investigation?
17.1.3 What Are the Imaging Findings?
17.2 Case 17.2
17.3 Vascular Rings and Sling
17.3.1 Introduction
17.3.1.1 Definitions
17.3.1.2 Classification
17.3.2 Epidemiology
17.3.3 Embryology
17.3.4 Associations
17.3.5 Clinical Features
17.3.6 Imaging
17.3.6.1 Goals of Imaging
Anatomic Features
Functional Evaluation:
Hemodynamic Evaluation:
17.3.6.2 Imaging Modalities
Imaging Evaluation of Tracheomalacia
Fiberoptic Bronchoscopy and Virtual Bronchoscopy [6]
17.3.6.3 Imaging Algorithm
17.3.6.4 Spectrum of Imaging Abnormalities (Vascular Rings and Slings)
Complete Rings
Incomplete Rings
17.4 Case 17.3
17.4.1 Imaging Findings
17.5 Case 17.4
17.5.1 Imaging Findings
17.6 Case 17.5
17.6.1 Chest X-Ray
17.7 Case 17.6
17.8 Case 17.7
17.9 Case 17.8
17.9.1 Chest X-Ray
17.10 Case 17.9
17.10.1 Chest X-Ray
17.10.2 Specific Features of Vascular Ring Morphologies
17.10.3 Management
17.10.3.1 Role of Cross-Sectional Imaging in Post-Operative Period
17.10.4 Future Prospective
17.10.4.1 3D Printing
Appendix 1: Imaging Protocol
Appendix 2: Reporting Template
References
18: Imaging in Heterotaxy Syndromes
18.1 Case 18.1
18.1.1 Clinical Presentation
18.1.2 Echo
18.1.3 What Will Be the Next Investigations?
18.1.4 Orthogonal Imaging
18.1.5 Final Diagnosis
18.2 Case 18.2
18.2.1 Clinical Presentation
18.2.2 Final Diagnosis
18.3 Case 18.3
18.3.1 Clinical Presentation
18.3.2 Final Diagnosis
18.4 Case 18.4
18.4.1 Clinical Presentation
18.4.2 Final Diagnosis
18.5 Heterotaxy Syndromes
18.5.1 Introduction
18.5.2 Components
18.5.3 Clinical Features
18.5.4 Embryology
18.6 Imaging
18.6.1 Goals of Imaging
18.6.1.1 Anatomic Features
18.6.1.2 Functional and Hemodynamic Evaluation
18.6.2 Imaging Modalities
18.7 How Imaging Impacts Management
18.7.1 Role of Cross-Sectional Imaging in Post-operative Period
References
19: Imaging in Coarctation of Aorta
19.1 Case 19.1
19.1.1 Clinical Presentation
19.1.2 Diagnosis
19.1.3 Next Step in Management
19.1.4 Final Diagnosis
19.1.5 Differential Diagnosis
19.1.5.1 In Aortic Coarctation
19.1.5.2 In Patients with Aortic Arch Interruption
19.1.6 NSAA
19.1.7 Pseudocoarctation
19.2 Coarctation of Aorta
19.2.1 Introduction [1]
19.2.1.1 Types
19.2.2 Etiology [2]
19.2.3 Genetic Predisposition
19.2.4 Pathophysiology [2]
19.2.4.1 In Utero
19.2.4.2 In Neonate
19.2.4.3 Adult
19.2.5 Associations
19.2.6 Natural History
19.2.7 Goals of Imaging [2, 3]
19.2.8 Plain Radiograph
19.2.9 Role of Echocardiography
19.2.10 Role of Orthogonal Imaging (CTA/MRA)
19.2.11 Advantages of MR over CT
19.2.12 Catheter Angiography
19.2.13 Role of 3D Printing [4]
19.2.14 Role of 4D Flow [5]
19.3 Case 19.2
19.3.1 Clinical Presentation
19.3.2 What Are the Imaging Findings?
19.3.3 Final Diagnosis
19.4 Case 19.3
19.4.1 Clinical Presentation
19.4.2 What Are the Imaging Findings?
19.4.3 Final Diagnosis
19.5 Case 19.4
19.5.1 Clinical Presentation
19.5.2 What Are the Imaging Findings?
19.5.3 Final Diagnosis
19.5.4 Companion Case
19.6 Case 19.5
19.6.1 Clinical Presentation
19.6.2 Final Diagnosis
19.7 Case 19.6
19.7.1 Clinical Presentation
19.7.2 Final Diagnosis
19.8 How Imaging Impacts Management?
19.8.1 Aim
19.8.2 Indications for Intervention
19.8.3 Ideal Age for Intervention
19.8.4 Mode of Intervention
19.8.5 Indications of Using a Covered Stent (Provided the Anatomy Is Suitable)
19.8.6 Types of Surgeries and their Indications
19.8.7 Follow-Up Recommendation
19.8.8 Role of Imaging in Assessing Long-Term Complications (Recoarctation/Aneurysm) [2, 3]
19.8.9 Prognostic Markers
19.8.10 Future Perspectives
Appendix 1: Imaging Protocol
Appendix 2: Reporting Template
References
Part II: Cardiomyopathies
20: Cardiac MR Imaging in Myocarditis
20.1 Case 20.1
20.1.1 Clinical Presentation
20.1.2 Chest Radiograph
20.1.3 ECG
20.1.4 Echocardiography
20.1.5 What Investigation Will You Do Next?
20.1.6 Provisional Diagnosis
20.1.7 What Investigation Will You Do Next?
20.1.8 Cardiac MRI
20.1.9 Final Diagnosis
20.2 Case 20.2
20.2.1 Chest Radiograph
20.2.2 ECG
20.2.3 Trop T
20.2.4 Echo
20.2.5 Cardiac MRI
20.2.6 Final Diagnosis
20.2.7 Is There a Role of Catheter Angiography in This Case?
20.3 Case 20.3
20.3.1 Provisional Diagnosis
20.3.2 What Investigation Will You Do Next?
20.3.3 Final Diagnosis
20.4 Case 20.4
20.4.1 Chest Radiograph
20.4.2 ECG
20.4.3 Trop T
20.4.4 Echocardiography
20.4.5 Provisional Diagnosis
20.4.6 What Investigation Will You Do Next?
20.4.7 Final Diagnosis
20.5 Case 20.5
20.5.1 Chest Radiograph
20.5.2 ECG and Cardiac Markers
20.5.3 Echocardiography
20.5.4 Provisional Diagnosis
20.5.5 What Investigation Will You Do Next?
20.5.6 Final Diagnosis
20.6 Discussion
20.6.1 Introduction
20.6.2 Classification/Types [2]
20.6.3 Etiology
20.6.4 Pathophysiology
20.6.5 Natural History [3]
20.6.6 Diagnostic Criteria
20.6.7 Goals of Imaging
20.6.8 Role of Various Imaging Modalities for a Suspected Case of Myocarditis
20.6.9 Predictors of Outcome/Prognostic Markers [2]
20.6.10 Management Strategies
20.6.11 Controversies
20.6.12 Future Perspective
References
21: Imaging in Cardiac Sarcoidosis
21.1 Case 21.1
21.1.1 Clinical Presentation
21.1.2 Chest Radiograph
21.1.3 Electrocardiography (ECG)
21.1.4 Echocardiography (ECHO)
21.1.5 What Investigation Will You Do Next?
21.1.6 CMR
21.1.7 CMR Diagnosis
21.1.8 Transbronchial Biopsy of Mediastinal Lymph Nodes
21.1.9 Final Diagnosis
21.1.10 Differential Diagnosis of Cardiac Sarcoidosis
21.2 Case 21.2
21.2.1 Clinical Presentation
21.2.2 Final Diagnosis
21.3 Case 21.3
21.3.1 Clinical Presentation
21.3.2 Final Diagnosis
21.4 Case 21.4
21.4.1 Clinical Presentation
21.4.2 Final Diagnosis
21.5 Case 21.5
21.5.1 Final Diagnosis
21.6 Case 21.6
21.6.1 Final Diagnosis
21.7 Case 21.7
21.7.1 CMR Diagnosis
21.7.2 Pet
21.7.3 Lymph Node Biopsy
21.7.4 Final Diagnosis
21.8 Case 21.8
21.8.1 Final Diagnosis
21.9 Discussion
21.9.1 Etiology and Pathophysiology
21.9.2 Natural History
21.9.3 Diagnostic Criteria
21.9.4 Imaging Modalities for Suspected Case of Cardiac Sarcoidosis
21.9.5 The Goals of CMR Include
21.9.6 Prognostic Role of CMR
21.9.7 LGE/Fibrosis in Case of Cardiac Sarcoid
21.9.8 Complications
21.9.9 Role of Imaging in Management
21.9.10 ICD Placement
References
22: Cardiac MR Imaging in Non Ischemic Dilated Cardiomyopathy
22.1 Case 22.1
22.1.1 Clinical Presentation
22.1.2 Chest Radiograph
22.1.3 Provisional Diagnosis
22.1.4 What Investigation Will You Do Next?
22.1.5 Final Diagnosis
22.2 Case 22.2
22.2.1 Clinical Presentation
22.2.2 What Investigation Will You Do Next?
22.2.3 Provisional Diagnosis
22.2.4 What Investigation Will You Do Next?
22.2.5 Final Diagnosis
22.3 Case 22.3
22.3.1 Clinical Presentation
22.3.2 Provisional Diagnosis
22.3.3 What Investigation Will You Do Next?
22.3.4 What Are the Imaging Findings?
22.3.4.1 Final Diagnosis
22.4 Case 22.4
22.4.1 Clinical Presentation
22.4.2 Provisional Diagnosis
22.4.3 What Investigation Will You Do Next?
22.4.4 Clinical Details
22.4.4.1 Final Diagnosis
22.5 Case 22.5
22.5.1 Clinical Presentation
22.5.2 Provisional Diagnosis
22.5.3 What Investigation Will You Do Next?
22.5.4 Final Diagnosis
22.6 Case 22.6
22.6.1 Clinical Presentation
22.6.2 Provisional Diagnosis
22.6.3 What Investigation Will You Do Next?
22.6.4 Final Diagnosis
22.6.4.1 Possible Etiology
22.6.4.2 Dilated Cardiomyopathy in Pediatric Age Group
22.7 Case 22.7
22.7.1 Clinical Presentation
22.7.2 Final Diagnosis
22.8 Discussion
22.8.1 Introduction
22.8.2 Etiology
22.8.3 Diagnostic Approach for Nonischemic Dilated Cardiomyopathy
22.8.4 Goals of Imaging
22.8.5 Role of Various Imaging Modalities for a Suspected Case of Nonischemic Cardiomyopathy
22.8.6 Management Strategies [8–11]
22.8.6.1 Evaluation for ICD
22.8.6.2 Role of LGE in Decision Making for ICD Placement
22.8.6.3 Screening of DCM [12]
References
23: Cardiac MR Imaging in Left Ventricular Noncompaction
23.1 Case 23.1
23.1.1 Clinical History
23.1.2 Investigations
23.1.2.1 Chest Radiograph
23.1.3 The Differential Diagnosis Included
23.1.4 Next Step in Evaluation?
23.1.5 Case Summary
23.2 Case 23.2
23.2.1 CMR Diagnosis
23.3 Case 23.3
23.3.1 CMR Diagnosis
23.3.2 Final Diagnosis
23.4 Case 23.4
23.4.1 CMR Diagnosis
23.5 Left Ventricular Noncompaction Cardiomyopathy
23.5.1 Etiopathogenesis
23.5.2 Pathophysiology and Clinical Features
23.5.3 Diagnostic Criteria
23.6 Imaging Modalities for Suspected Case of LVNC
23.6.1 Goals of CMR
23.6.2 Complications
23.7 How Imaging Impacts Management?
23.7.1 Genetic and Family Screening
23.7.2 Anticoagulation
23.7.3 Sudden Cardiac Death
23.7.4 Role of MRI in Management
References
24: Cardiac MR Imaging in Arrhythmogenic Ventricular Cardiomyopathy
24.1 Case 24.1
24.1.1 Clinical Presentation
24.1.2 Electrocardiography
24.1.3 Echocardiography
24.1.4 The Differential Diagnosis Will Include
24.1.5 What Investigation Will You Do Next?
24.1.6 CMR
24.1.7 Final Diagnosis
24.1.8 Differential Diagnosis
24.2 Case 24.2
24.2.1 Clinical Presentation
24.2.2 Final Diagnosis
24.3 Case 24.3
24.3.1 Clinical Presentation
24.3.2 Final Diagnosis
24.4 Discussion
24.4.1 Etiology and Pathophysiology
24.4.2 Clinical Characteristics
24.4.3 Imaging Modalities for Suspected Case of Arrhythmogenic Ventricular Cardiomyopathy
24.4.3.1 Goals of CMR
24.4.4 Morphology
24.4.5 Function
24.4.6 LGE
24.4.7 Changing Spectrum of ARVC
24.4.8 Predictors of Poor Prognosis and Long-Term Mortality on CMR
24.4.9 Complications
24.4.10 Management
24.4.11 Role of CMR
24.4.12 Controversies and Future Perspectives
24.4.12.1 Controversies Related to Revised Task Force Criteria
24.4.13 Strain Imaging
References
25: Cardiac MR Imaging in Hypertrophic Cardiomyopathy
25.1 Case 25.1
25.1.1 Clinical Presentation
25.1.2 Echocardiography
25.1.3 Provisional Diagnosis
25.1.4 Differential Diagnosis
25.1.5 What Investigation Will You Do Next?
25.1.6 Final Diagnosis
25.1.7 Note
25.1.7.1 Isolated Basal Septal Hypertrophy
25.2 Case 25.2
25.2.1 Clinical Presentation
25.2.2 Provisional Diagnosis
25.2.3 Differential Diagnosis
25.2.4 What Investigation Will You Do Next?
25.2.5 Final Diagnosis
25.3 Case 25.3
25.3.1 Clinical Presentation
25.3.2 Provisional Diagnosis
25.3.3 Differential Diagnosis
25.3.4 What Investigation Will You Do Next?
25.3.5 Final Diagnosis
25.4 Case 25.4
25.4.1 Clinical Presentation
25.4.2 Provisional Diagnosis
25.4.3 What Investigation Will You Do Next?
25.4.4 Final Diagnosis
25.5 Case 25.5
25.5.1 Clinical Presentation
25.5.2 Provisional Diagnosis
25.5.3 What Investigation Will You Do Next?
25.5.4 Final Diagnosis
25.6 Case 25.6
25.6.1 Clinical Presentation
25.6.2 Final Diagnosis
25.6.3 Screening of Relatives of Patients Diagnosed with HCM
25.7 Case 25.7
25.7.1 Clinical Presentation
25.7.2 Provisional Diagnosis
25.7.3 What Investigation Will You Do Next?
25.7.4 Final Diagnosis
25.8 Case 25.8
25.8.1 Clinical Presentation
25.9 Discussion
25.9.1 Introduction
25.9.2 Etiopathogenesis
25.9.3 Histopathology
25.9.4 Natural History [1]
25.9.5 Morphological Pattern: HCM Phenotypes
25.9.6 Diagnostic Criteria [1]
25.9.7 Goals of Imaging
25.9.8 Role of Various Imaging Modalities for a Suspected Case of Hypertrophic Cardiomyopathy
25.9.9 Predictors of Sudden Cardiac Death
25.9.10 Management Strategies/How Imaging Impacts Management
25.9.10.1 Postprocedure Imaging
25.9.10.2 Controversies
25.9.10.3 Future Perspective
References
26: Imaging in Cardiac Amyloidosis
26.1 Case 26.1
26.1.1 Clinical Presentation
26.1.2 Electrocardiography
26.1.3 Echocardiography
26.1.4 The Differential Diagnosis Will Include
26.1.5 What Investigation Will You Do Next?
26.1.6 CMR
26.1.7 Final Diagnosis
26.1.8 Differential Diagnosis
26.2 Case 26.2
26.2.1 Clinical Presentation
26.2.2 Final Diagnosis
26.3 Case 26.3
26.3.1 Clinical Presentation
26.3.2 Final Diagnosis
26.4 Discussion
26.4.1 Etiology and Pathophysiology
26.4.1.1 Light Chain (AL) Amyloidosis
26.4.1.2 Transthyretin (ATTR) Amyloidosis
26.4.2 Imaging Modalities for Suspected Case of Cardiac Amyloidosis
26.4.2.1 Goals of CMR
26.4.2.2 CMR Findings in Cardiac Amyloidosis Are Tabulated in Table 26.3
26.4.2.3 Predictors of Poor Prognosis and Long-Term Mortality on CMR
26.4.3 Complications
26.4.4 Management
26.4.5 Role of MRI in Determining Response to Treatment
References
27: Cardiac MR Imaging in Restrictive Cardiomyopathy
27.1 Case 27.1
27.1.1 Clinical Presentation
27.1.2 Chest Radiograph
27.1.3 Echocardiography
27.1.4 Provisional Diagnosis
27.1.4.1 Based on Echo Findings the Differentials to Be Included Are
27.1.5 What Investigation Will You Do Next?
27.1.6 Final Diagnosis
27.1.7 The Differential Diagnosis Will Include
27.2 Case 27.2
27.2.1 Clinical Presentation
27.2.2 Echocardiography
27.2.3 Provisional Diagnosis
27.2.4 Cardiac MR
27.2.5 Final Diagnosis
27.2.6 Note
27.3 Case 27.3
27.3.1 Clinical Presentation
27.3.2 Echocardiography
27.3.3 ECG
27.3.4 Provisional Diagnosis
27.3.5 Differentials to Be Considered Are
27.3.6 Cardiac MR
27.3.7 Final Diagnosis
27.4 Case 27.4
27.4.1 Clinical Presentation
27.4.2 Echocardiography
27.4.3 Provisional Diagnosis
27.4.4 Cardiac MR
27.4.5 Final Diagnosis
27.5 Case 27.5
27.5.1 Clinical Presentation
27.5.2 Echocardiography
27.5.3 Provisional Diagnosis
27.5.4 Differential to Be Considered
27.5.5 Cardiac MR
27.5.6 Final Diagnosis
27.6 Case 27.6
27.6.1 Clinical Presentation
27.6.2 Echocardiography
27.6.3 Provisional Diagnosis
27.6.4 Cardiac MR
27.6.5 Final Diagnosis
27.7 Case 27.7
27.7.1 Clinical Presentation
27.7.2 Echocardiography
27.7.3 Provisional Diagnosis
27.7.4 Cardiac MR
27.7.5 Final Diagnosis
27.8 Case 27.8
27.8.1 Clinical Presentation
27.8.2 Echocardiography
27.8.3 Provisional Diagnosis
27.8.4 Cardiac MR
27.8.5 Final Diagnosis
27.9 Case 27.9
27.9.1 Clinical Presentation
27.9.2 Echocardiography
27.9.3 Provisional Diagnosis
27.9.4 Cardiac MR
27.9.5 Final Diagnosis
27.10 Case 27.10
27.10.1 Clinical Presentation
27.10.2 Echocardiography
27.10.3 Provisional Diagnosis
27.10.4 Cardiac MR
27.10.5 Final Diagnosis
27.11 Case 27.11
27.11.1 Clinical Presentation
27.11.2 Echocardiography
27.11.3 Provisional Diagnosis
27.11.4 Cardiac MR
27.11.5 Final Diagnosis
27.12 Discussion
27.12.1 Introduction
27.12.2 Etiology and Pathogenesis (Table 27.14)
27.12.3 Clinical Presentation
27.12.4 Imaging
27.12.5 Goals of CMR
27.13 Endomyocardial Fibrosis
27.13.1 Introduction
27.13.2 Etiology and Pathogenesis
27.13.3 Clinical Presentation
27.13.4 Imaging
27.13.5 How Imaging Impacts the Management?
27.13.6 Current Controversies and Future Aspects
References
28: Imaging in Cardiac Tuberculosis
28.1 Case 28.1
28.1.1 Clinical Presentation
28.1.2 CMR
28.2 Case 28.2
28.2.1 Clinical Presentation
28.2.2 Final Diagnosis
28.3 Case 28.3
28.3.1 Clinical Presentation
28.3.2 Final Diagnosis
28.4 Case 28.4
28.4.1 Clinical Presentation
28.4.2 Final Diagnosis
28.5 Case 28.5
28.5.1 Clinical Presentation
28.5.2 Final Diagnosis
28.6 Case 28.6
28.6.1 Clinical Presentation
28.6.2 Final Diagnosis
28.7 Case 28.7
28.7.1 Clinical Presentation
28.7.2 Final Diagnosis
28.8 Case 28.8
28.8.1 Clinical Presentation
28.8.2 Final Diagnosis
28.9 Dviscussion
28.9.1 Introduction
28.9.2 Pathophysiology and Clinical Features
28.9.2.1 Acute Pericarditis
28.9.2.2 Effusive Pericarditis
28.9.2.3 Constrictive Pericarditis
28.9.2.4 Myopericarditis
28.9.2.5 Myocarditis
References
29: Imaging in Iron Overload Cardiomyopathy
29.1 Case 29.1
29.1.1 Clinical History
29.1.2 Echo
29.1.3 ECG
29.1.4 Provisional Diagnosis
29.1.5 Final Diagnosis
29.2 Case 29.2
29.2.1 Clinical History
29.3 Echo
29.3.1 Tissue Iron Quantification
29.3.2 Final Diagnosis
29.4 Case 29.3
29.4.1 Clinical History
29.4.2 Echo
29.4.3 Final Diagnosis
29.5 Case 29.4
29.5.1 Clinical History
29.5.2 Echo
29.5.3 Final Diagnosis
29.6 Iron Overload Cardiomyopathy
29.6.1 Introduction
29.6.2 Etiology and Pathophysiology
29.6.3 Clinical Presentation
29.6.4 Imaging
References
30: Cardiac MR Imaging in Ischemic Cardiomyopathy
30.1 Case 30.1
30.1.1 Clinical History
30.1.2 Chest X-Ray
30.1.3 ECG
30.1.4 Differential Diagnosis
30.1.5 What Is the Next Investigation?
30.1.6 What Are the Imaging Findings?
30.1.7 Diagnosis
30.2 Ischemic Cardiomyopathy
30.2.1 Introduction
30.2.2 Epidemiology
30.2.3 Pathophysiology
30.2.4 Imaging
30.2.4.1 Goals of Imaging
30.2.5 Imaging Modalities
30.2.6 Role of CMR in Ischemic Cardiomyopathy
30.2.6.1 Assessment of Morphologic Features with Global and Segmental Function
30.2.6.2 Assessment of Myocardial Perfusion
30.2.6.3 Evaluation of LGE
30.2.6.4 Prognostic Factors Based on CMR
30.2.7 Role of Cardiac CTA in Ischemic Cardiomyopathy
30.2.7.1 Detection of Coronary Artery Stenosis
30.2.8 Recommendations
References
31: Imaging in Complications of Myocardial Ischemia
31.1 Case 31.1
31.1.1 Clinical Presentation
31.1.2 Chest Radiograph
31.1.3 Echocardiography
31.1.4 Differential Diagnosis
31.1.5 What Will Be the Next Investigations?
31.1.6 CMR
31.1.7 CT Angiography
31.1.8 Final Diagnosis
31.2 Case 31.2
31.2.1 Clinical Presentation
31.2.2 Final Diagnosis
31.2.3 How to Differentiate Between These Entities?
31.3 Case 31.3
31.3.1 Clinical Presentation
31.3.2 CMR
31.3.3 Final Diagnosis
31.4 Case 31.4
31.4.1 Clinical Presentation
31.4.2 Final Diagnosis
31.4.3 What Is the Role of Imaging Here?
31.5 Discussion
31.5.1 Introduction
31.5.2 Complications
31.5.3 Classification
31.5.4 Epidemiology
31.5.5 Clinical Features
31.6 Goals of Imaging
31.6.1 Anatomic Features
31.6.2 Functional and Hemodynamic Evaluation
31.6.3 Imaging Modalities
31.6.4 Spectrum of Imaging Abnormalities
31.6.5 Specific Features of Morphologies [3]
31.6.6 Management
References
32: Imaging in Takotsubo Cardiomyopathy
32.1 Case 32.1
32.1.1 Clinical Presentation
32.1.2 Chest Radiograph
32.1.3 ECG
32.1.4 Echo
32.1.5 What Investigation Will You Do Next?
32.1.6 Provisional Clinical Diagnosis
32.1.7 Differential Diagnosis
32.1.8 Next Step in Evaluation?
32.1.9 Cardiac MRI
32.1.10 Final Diagnosis
32.1.11 Follow-Up Echo (After 3 Weeks)
32.1.12 Differential Diagnosis
32.2 Discussion
32.2.1 Introduction
32.2.2 Pathophysiology
32.2.3 Natural History
32.2.4 Diagnostic Criteria
32.2.5 Role of MRI in a Suspected Case of Takotsubo Cardiomyopathy [1]
32.2.6 Types of Takotsubo Cardiomyopathy
32.2.7 Role of Strain Imaging
32.2.8 LGE/Fibrosis in Case of Takotsubo Cardiomyopathy
32.2.9 Estimation of Scar Tissue
32.2.10 Complications
32.2.11 Independent Predictors of Long-Term Mortality
32.2.12 Management
32.2.13 Recovery Criteria During Follow Up Imaging
References
33: Imaging in Post Heart Transplant Patients
33.1 Case 33.1
33.1.1 Clinical History
33.2 Case 33.2
33.2.1 Clinical History
33.3 Case 33.3
33.3.1 Clinical History
33.4 Imaging in Heart Transplant
33.4.1 Natural History
33.4.2 Acute Rejection
33.4.3 Chronic Rejection/Cardiac Allograft Vasculopathy (CAV)
33.4.4 Predictors of Poor Prognosis and Long-Term Mortality on CMR
33.4.5 Problems with Imaging of Heart Transplant Recipients with CT
33.4.6 How Imaging Impacts Management?
References
Part III: Cardiac Masses
34: Imaging Approach to Cardiac Masses
34.1 Case 34.1
34.1.1 Clinical History
34.1.2 Investigations
34.1.2.1 Electrocardiogram (ECG)
34.1.2.2 Chest Radiography (CXR)
34.1.2.3 Inflammatory Markers
34.1.2.4 Echocardiogram
34.1.3 Provisional Diagnosis
34.1.4 What Investigation Will You Do Next? Cardiac MR or CT or Both?
34.1.5 Is There a Role of Cardiac CT?
34.1.6 Differential Diagnosis of Left Atrial Myxoma
34.1.6.1 Left Atrial Thrombus Thrombus, Paraganglioma, Hemangioma, Lipoma, Inflammatory/Infective masses, Secondaries
34.2 Case 34.2: Left Atrial Thrombus
34.3 Case 34.3: Primary Malignant Atrial Sarcoma
34.4 Case 34.4: Paraganglioma
34.5 Case 34.5: Lipoma
34.6 Case 34.6: Papillary Fibroelastoma
34.7 Imaging Features of Cardiac Myxoma
34.7.1 Radiograph
34.7.2 Echocardiography
34.7.3 Computed Tomography: Done Primarily for Coronary Evaluation in Low to Intermediate CAD Risk
34.7.4 Magnetic Resonance Imaging
34.7.5 Management
34.8 Case 34.8
34.8.1 Clinical History
34.8.2 Investigations
34.8.2.1 Electrocardiogram (ECG)
34.8.2.2 Chest X-Ray
34.8.2.3 Echocardiogram
34.8.3 Provisional Diagnosis
34.8.4 Diagnosis
34.8.5 Differential Diagnosis
34.9 Case 34.9: Right Atrial Myxoma
34.10 Case 34.10: Tuberculoma
34.11 Case 34.11: Cardiac Lymphoma
34.12 Case 34.12: Right Atrial Thrombus
34.13 Case 34.13: Cardiac Angiosarcoma
34.13.1 Radiography
34.13.2 Echocardiography
34.13.3 Computed Tomography
34.13.4 Magnetic Resonance Imaging
34.14 Miscellaneous Tumors
34.14.1 Case 34.14: Rhabdomyoma
34.14.2 Case 34.15: Fibroma
34.15 Case 34.16: Calcified Amorphous Tumor
Appendix 1
CT Angiography Imaging Protocol
Cardiac MRI Protocol
Appendix 2
Reporting Points
Part IV: Coronary Artery Anomalies
35: Imaging in Anomalies of Coronary Artery Origin
35.1 Case 35.1
35.1.1 Clinical History
35.1.2 Chest X-Ray
35.1.3 ECG
35.1.4 Echocardiography
35.1.5 Provisional Diagnosis
35.1.6 What Investigation Will You Do Next?
35.1.6.1 Goals of Imaging in Coronary Anomalies
35.1.7 Final Diagnosis
35.1.8 Differential Diagnosis Includes the Following
35.2 Case 35.5
35.2.1 Clinical History
35.2.2 Final Diagnosis: Adult ALCAPA
35.2.3 Differential Diagnosis of Diffuse Coronary Artery Dilatation Includes
35.3 Anomalous Origin of the Left Coronary Artery from Pulmonary Artery
35.3.1 Introduction
35.3.2 Embryology
35.3.3 Pathophysiology of ALCAPA
35.3.3.1 Infants Who Are More Likely to Survive Until Adulthood Have Following Features Including [1–4]
35.3.4 Associated Anatomical Abnormalities [1–4]
35.3.5 Variations in the Anomalous Origin of Coronaries from the Pulmonary Artery [1–4]
35.3.5.1 This Syndrome Has Been Characterized in Four Different Variants
35.3.6 ARCAPA
35.3.6.1 Imaging in ALCAPA [1–6]
35.3.6.2 Spectrum of Other Anomalous Coronary Artery Origin from Pulmonary Artery Includes
35.4 Management
35.4.1 Management of ARCAPA
References
36: Imaging in Anomalies of Coronary Artery Course
36.1 Case 36.1
36.1.1 Clinical History
36.1.2 Final Diagnosis
36.2 Case 36.2
36.2.1 Clinical History
36.2.2 Final Diagnosis
36.2.3 Differential Diagnosis
36.3 Anomalous Origin of Coronary with Malignant Interatrial Course
36.3.1 Pathophysiology
36.3.2 Imaging Findings
36.4 Management
36.4.1 Surgical Management
36.4.2 Coronary Unroofing
36.4.3 Reimplantation
36.4.4 CABG
36.4.5 Postoperative Complications
36.4.6 Normal Anatomy of the Coronary Arteries
36.5 Classification of Coronary Anomalies Origin and Course
36.6 Classification of Coronary Anomalies Origin and Course
36.7 Imaging of Coronary Anomalies [1–5]
36.7.1 Goals of Imaging in Coronary Anomalies
36.7.2 Anomalies of Origin and Course
36.7.2.1 High Take-off
36.7.2.2 Multiple Ostia
36.7.2.3 Single Coronary Artery
36.7.2.4 Origin of the Coronary Artery or Branch from the Opposite or Noncoronary Sinus and an Anomalous Course
36.7.2.5 Dual LAD [4]
36.7.2.6 Myocardial Bridging
References
37: Imaging in Anomalies of Coronary Artery Termination
37.1 Case 37.1
37.1.1 Clinical Presentation
37.1.2 Chest Radiograph
37.1.3 Differential Diagnosis
37.1.4 Echocardiography
37.1.5 What Investigation Will You Do Next?
37.1.6 CT Angiography
37.1.7 Final Diagnosis
37.1.8 Differential Diagnosis of Coronary Cameral Fistula
37.2 Case 37.2
37.2.1 Clinical Presentation
37.2.2 Final Diagnosis
37.2.3 Differentiating Features from Coronary Cameral Fistula
37.3 Case 37.3
37.3.1 Clinical Presentation
37.3.2 Final Diagnosis
37.3.3 Differentiating Features from Coronary Cameral Fistula
37.4 Spectrum of Imaging Abnormalities in Coronary Cameral Fistula
37.4.1 Case 37.4
37.4.1.1 Final Diagnosis
37.4.2 Case 37.5
37.4.2.1 Final Diagnosis
37.4.3 Case 37.6
37.4.3.1 Final Diagnosis
37.5 Discussion
37.5.1 Introduction
37.5.2 Embryology and Pathoanatomy
37.5.3 Classification
37.5.4 Imaging in Coronary Artery Fistulas
37.5.4.1 Goals of Imaging
37.5.4.2 Imaging Modalities
37.5.4.3 Chest Radiographs
37.5.4.4 Computed Tomography
Coronary Arteries
Noncoronary Cardiac Findings in Congenital Coronary Artery Fistulas
37.5.4.5 Catheter Angiography
37.5.4.6 Cardiac MRI
Hemodynamics
Myocardial Ischemia
37.5.5 How Imaging Impacts Management?
References
38: Imaging in Coronary Artery Aneurysms
38.1 Case 38.1
38.1.1 Electrocardiography
38.1.2 Echocardiography
38.1.3 What Investigation Will You Do Next?
38.1.4 Final Diagnosis
38.1.5 Etiology
38.1.6 Differential Diagnosis
38.2 Case 38.2
38.2.1 Final Diagnosis
38.2.2 Etiology
38.3 Case 38.3
38.3.1 Final Diagnosis
38.3.2 Etiology
38.4 Case 38.4
38.4.1 Final Diagnosis
38.4.2 Etiology
38.5 Discussion
38.5.1 Introduction
38.6 Nomenclature
38.6.1 Coronary Artery Aneurysms
38.6.2 Ectasia
38.7 Classification of Coronary Artery Dilatation (1)
38.7.1 Vessel Wall Composition
38.7.2 Shape
38.7.3 Diffuse Dilatation: ECTASIA
38.7.4 Clinical Characteristics and Complications
38.7.5 Imaging Assessment of Coronary Artery Aneurysm
38.7.6 Management of Atherosclerotic Coronary Aneurysms
38.7.6.1 Medical Management
38.7.6.2 Indications of Intervention
38.7.6.3 Intervention
References
Part V: Miscellaneous
39: Advances in Cardiovascular MRI in Heart Failure
39.1 Introduction [1, 2]
39.2 Advances in CMRI
39.2.1 Strain Imaging [3, 4]
39.2.2 Mapping and ECV [5–7]
39.2.3 LGE Quantification and 3D-LGE [8, 9]
39.2.4 Diastolic Dysfunction [1, 2]
39.2.5 4D Flow [10–12]
39.2.6 Diffusion Tensor Imaging [13, 14]
39.2.7 ASL [15, 16]
39.2.8 Simplifying MRI-Conditional Implant Scans [17–21]
39.2.9 Artificial Intelligence (AI) in Cardiovascular Imaging [22–25]
39.2.10 Metabolic Imaging [26–32]
References
40: Artificial Intelligence in Cardiac Imaging
40.1 Machine Learning
40.1.1 Data and Information
40.1.2 Features
40.2 How Does the Machine Learn?
40.2.1 Algorithm and Model
40.2.2 Validation
40.3 AI in Medical Imaging
40.4 AI in Cardiovascular Imaging
40.5 Pitfalls and Problems
40.6 Conclusion
40.6.1 The Vision for AI and Cardiac Imaging
References

Citation preview

Sanjiv Sharma Editor

Case-based Atlas of Cardiac Imaging

123

Case-based Atlas of Cardiac Imaging

Sanjiv Sharma Editor

Case-based Atlas of Cardiac Imaging

Editor Sanjiv Sharma Departments of Radiology and Interventional Radiology National Institute of Medical Sciences, NIMS University Jaipur, India Department of Cardiovascular Radiology and Endovascular Interventions All India Institute of Medical Sciences New Delhi, India

ISBN 978-981-99-5619-7    ISBN 978-981-99-5620-3 (eBook) https://doi.org/10.1007/978-981-99-5620-3 © The Editor(s) (if applicable) and The Author(s), under exclusive license to Springer Nature Singapore Pte Ltd. 2023 This work is subject to copyright. All rights are solely and exclusively licensed by the Publisher, whether the whole or part of the material is concerned, specifically the rights of translation, reprinting, reuse of illustrations, recitation, broadcasting, reproduction on microfilms or in any other physical way, and transmission or information storage and retrieval, electronic adaptation, computer software, or by similar or dissimilar methodology now known or hereafter developed. The use of general descriptive names, registered names, trademarks, service marks, etc. in this publication does not imply, even in the absence of a specific statement, that such names are exempt from the relevant protective laws and regulations and therefore free for general use. The publisher, the authors, and the editors are safe to assume that the advice and information in this book are believed to be true and accurate at the date of publication. Neither the publisher nor the authors or the editors give a warranty, expressed or implied, with respect to the material contained herein or for any errors or omissions that may have been made. The publisher remains neutral with regard to jurisdictional claims in published maps and institutional affiliations. This Springer imprint is published by the registered company Springer Nature Singapore Pte Ltd. The registered company address is: 152 Beach Road, #21-01/04 Gateway East, Singapore 189721, Singapore Paper in this product is recyclable.

Preface

Advances in CT and MRI technologies are fast changing the way we practice cardiac imaging. The advent of these noninvasive orthogonal imaging techniques has helped create new imaging algorithms for an improved anatomic, hemodynamic, and functional diagnosis in most cardiovascular diseases. These techniques help provide an accurate diagnosis, appropriate treatment planning, as well as prognostication of outcomes. They also go a long way in better understanding of the natural history of cardiovascular diseases. Gainful utilization of clinical cardiac imaging requires judicious application of these techniques. There is an urgent need for a case-­ based learning opportunity for understanding the advantages and limitations of conventional and advanced imaging in various clinical scenarios. This case-based atlas has been designed to provide the learning opportunity for optimal use of imaging in cardiovascular diseases. Chest radiograph is the most widely available, easy to obtain, and an economical starting point for investigation of any patient with a suspected heart disease. This atlas provides an in-depth description of reading a chest radiograph in heart disease, using images from over 100 cases for a comprehensive learning experience and building an appropriate algorithm in different clinical settings. Real-world clinical cases seen in our day-to-day practice have been analyzed to demonstrate the value of multimodality imaging in the management and follow-up of patients with congenital and acquired cardiovascular diseases. While this atlas provides an overview of the clinical presentation, etiology, pathophysiology, and treatment of a vast array of cardiac diseases, it is not meant to be an exhaustive treatise on cardiac pathologies and their management. The analytical approach is intended to act as a guide for the treating physician radiologist, cardiologist, or surgeon for inculcating an evidence-based approach for choosing the right imaging algorithm in the given clinical situation. Our goal is to provide a framework to the reader for recognizing different imaging patterns of various cardiac pathologies and inculcate a rational imaging-based investigative approach that allows extraction of maximum diagnostic and prognostic information from the pertinent images. The highly visual design of the atlas will enable it to act as a quick and ready reference. The atlas introduces the reader to a chest radiograph-based approach to cardiac diseases with the subsequent sections on congenital cardiac defects, cardiac masses, cardiomyopathies, and coronary-related pathologies beginning with description of the imaging approach, followed by cases comprehensively covering the gamut of clinical scenarios that may be encountered in clinical practice. The atlas encompasses over 300 cases featuring over 2000 high-quality radiological images and schematic illustrations aimed at providing a visual impact that cannot be obtained from any amount of text-based learning. Sections pertaining to the chest radiographs in heart disease, imaging in rheumatic heart disease, cardiac tuberculosis, prosthetic heart valve disease, advances in cardiovascular MRI imaging techniques, and application of artificial intelligence and deep learning add immense learning value to the atlas. Cases in the atlas range from those commonly encountered in clinical practice to many extremely rare conditions. The format of the atlas has allowed us to illustrate these cases generously, concentrating on multiple high-quality images. These will allow the reader to identify an unknown case fairly easily by simply comparing the images of his patient with the images in the atlas. v

vi

Preface

Across the atlas, each index case begins with a clinical scenario that ties radiological imaging to the real-world conditions encountered in our patients. A multimodality imaging approach is outlined for each clinical scenario with sequential imaging findings presented in a manner which replicates the way the patient would ideally undergo imaging-based investigations to arrive at the diagnosis in day-to-day practice. Cases that may pose as possible imaging differential diagnosis accompany the index case, followed by a variety of companion cases illustrating the possible spectrum of abnormalities that the reader may be confronted with while dealing with the index case. This concept of the index case, imaging appearances of various differential diagnoses, and companion cases showcasing the spectrum of imaging abnormalities in the index disease state as well as use of imaging to guide management options in each case is a unique first for this atlas. A short discussion accompanies each index case, but for more detailed and in-depth information, we advise the reader to refer to existing textbooks. The accompanying text is concisely presented with many easy-to-access bullets as a quick point-of-care reference focusing on the impact of imaging on management while highlighting the current controversies and future trends in imaging of the disease process under consideration. The optimized imaging protocols have been provided in the discussion, wherever relevant, to serve as a ready reckoner for readers who are new to cardiac imaging. Each section concludes with an assortment of structured multiple-choice questions. The stimulating cases should act as “teasers” intended to serve as self-assessment tools for the readers. The staff of the department of cardiovascular radiology and endovascular interventions at this Institute have joined hands to put together this treatise. The concepts, contents, and algorithmic approach presented in this text summarize the experience of this dedicated cardiac imaging center over the last 35 years. I fervently hope that it will positively influence the practice standards of the readers. Jaipur, Rajasthan, India

Sanjiv Sharma

Contents

1 Chest  Radiograph in Heart Disease �������������������������������������������������������������������������   1 Sanjiv Sharma, Niraj Nirmal Pandey, Pujitha Vidiyala, and Amarinder Singh Malhi Part I Congenital Heart Diseases 2 Algorithmic  Approach to Imaging Diagnosis of Congenital Heart Disease ���������  43 Sanjiv Sharma, Niraj Nirmal Pandey, and Aprateem Mukherjee 3 Imaging  in Tetralogy of Fallot�����������������������������������������������������������������������������������  47 Sreenivasa Narayana Raju 4 Imaging  in Double-Outlet Right Ventricle���������������������������������������������������������������  89 Rishabh Khurana 5 Imaging in Tricuspid Atresia�������������������������������������������������������������������������������������  97 Sravan Nagulakonda and Amarinder Singh Malhi 6 Imaging  in Transposition of Great Arteries������������������������������������������������������������� 113 Surya Pratap Singh 7 Imaging in Truncus Arteriosus ��������������������������������������������������������������������������������� 123 Amit Ajit Deshpande 8 Imaging  in Total Anomalous Pulmonary Venous Connections and Partial Anomalous Pulmonary Venous Connections��������������������������������������� 133 Amit Ajit Deshpande 9 Imaging in Ebstein’s Anomaly����������������������������������������������������������������������������������� 147 Amit Ajit Deshpande 10 Imaging  in Hypoplastic Left Heart Syndrome��������������������������������������������������������� 171 Vineeta Ojha 11 Imaging  in Ventricular Septal Defect ����������������������������������������������������������������������� 187 Manish Shaw and S. H. Chandrashekhara 12 Imaging  in Atrioventricular Canal Defects (AVSD) ����������������������������������������������� 215 Surya Pratap Singh 13 Imaging in Aortopulmonary Window����������������������������������������������������������������������� 221 Rishabh Khurana 14 Imaging  in Sinus of Valsalva Aneurysm������������������������������������������������������������������� 235 Manish Shaw

vii

viii

15 Imaging  of Patent Ductus Arteriosus ����������������������������������������������������������������������� 251 Mahesh Krishna Anne and Amit Ban 16 Imaging in Cor Triatriatum��������������������������������������������������������������������������������������� 267 Vasundhara Arora 17 Imaging in Vascular Rings����������������������������������������������������������������������������������������� 279 Neeraj Kumar and Amit Ban 18 Imaging  in Heterotaxy Syndromes��������������������������������������������������������������������������� 305 Avichala Taxak 19 Imaging  in Coarctation of Aorta������������������������������������������������������������������������������� 313 Rishabh Khurana Part II Cardiomyopathies 20 Cardiac  MR Imaging in Myocarditis ����������������������������������������������������������������������� 327 Rishabh Khurana 21 Imaging  in Cardiac Sarcoidosis��������������������������������������������������������������������������������� 341 Mansi Verma and Vineeta Ojha 22 Cardiac  MR Imaging in Non Ischemic Dilated Cardiomyopathy������������������������� 357 Rishabh Khurana 23 Cardiac  MR Imaging in Left Ventricular Noncompaction������������������������������������� 379 Vasundhara Arora 24 Cardiac  MR Imaging in Arrhythmogenic Ventricular Cardiomyopathy ������������� 387 Mansi Verma and Vineeta Ojha 25 Cardiac  MR Imaging in Hypertrophic Cardiomyopathy��������������������������������������� 395 Rishabh Khurana 26 Imaging in Cardiac Amyloidosis������������������������������������������������������������������������������� 411 Mansi Verma and Vineeta Ojha 27 Cardiac  MR Imaging in Restrictive Cardiomyopathy ������������������������������������������� 419 Amit Ajit Deshpande 28 Imaging in Cardiac Tuberculosis������������������������������������������������������������������������������� 441 Sravan Nagulakonda 29 Imaging  in Iron Overload Cardiomyopathy������������������������������������������������������������� 453 Surya Pratap Singh 30 Cardiac  MR Imaging in Ischemic Cardiomyopathy����������������������������������������������� 461 Mansi Verma and Amit Ban 31 Imaging  in Complications of Myocardial Ischemia������������������������������������������������� 469 Avichala Taxak 32 Imaging in Takotsubo Cardiomyopathy������������������������������������������������������������������� 481 Rishabh Khurana 33 Imaging  in Post Heart Transplant Patients ������������������������������������������������������������� 489 Vineeta Ojha Part III Cardiac Masses 34 Imaging  Approach to Cardiac Masses ��������������������������������������������������������������������� 497 Mumun Sinha and Mansi Verma

Contents

Contents

ix

Part IV Coronary Artery Anomalies 35 Imaging  in Anomalies of Coronary Artery Origin��������������������������������������������������� 521 Sreenivasa Narayana Raju 36 Imaging  in Anomalies of Coronary Artery Course������������������������������������������������� 535 Sreenivasa Narayana Raju 37 Imaging  in Anomalies of Coronary Artery Termination����������������������������������������� 567 Sravan Nagulakonda 38 Imaging in Coronary Artery Aneurysms ����������������������������������������������������������������� 581 Mansi Verma Part V Miscellaneous 39 Advances  in Cardiovascular MRI in Heart Failure������������������������������������������������� 591 Amit Ajit Deshpande and Manish Shaw 40 Artificial  Intelligence in Cardiac Imaging ��������������������������������������������������������������� 599 Rishabh Khurana and Vineeta Ojha

About the Editor

Sanjiv Sharma  (MD) is a former Professor and Head of the Department of Cardiovascular Radiology and Endovascular Interventions at the All India Institute of Medical Sciences, New Delhi, retired after a career spanning 41 years in radiology; and 35 years of them in a dedicated cardiac radiology department. He was president of the Asia-Pacific Society of Cardiovascular and Interventional Radiology (2004–2006), Indian Society of Vascular and Interventional Radiology (2003–2006), Indian national registry for Vascular and Interventional Radiology procedures (1999–2014), ISVIR Research and Education Foundation (2009–2015) and Society for Emergency Radiology (2017–2021). He is the Regional Editor of Cardiovascular and Interventional Radiology (CVIR, official journal of Cardiovascular and Interventional Radiology Society of Europe) since 2009, Associate Editor of Journal of Vascular and Interventional Radiology (JVIR, official journal of the North American Society of Interventional Radiology) since 2010 and Regional Editor of Asia Pacific Journal of Interventional Radiology. He has edited many publications, including, Update on Imaging and Intervention in Cardiovascular Diseases in 1994; Recent Advances in Imaging and Intervention in Cardiovascular Diseases (Springer-Verlag, Singapore, 1996); Syllabus: MCVIR’2000, Indian Society of Vascular and Interventional Radiology; Official publications of Indian Society of Vascular and Interventional Radiology (July 1997–December 2000); and Needs for development of Interventional Radiology in the Emerging World (International Atomic Energy Agency, 2015, Vienna, Austria), among others. In addition, he has 324 scientific papers in peer-reviewed international and national journals and 68 chapters in books. He has delivered 586 guest lectures and presentations in various meetings and teaching programs outside India (266) and within India (320). He has organized over 40 national and international scientific congresses and is a regular guest faculty at various national and international forums. He is a member of various national and international scientific societies and bodies and a visiting professor to various national and international universities. He is also the recipient of various awards and honours, including ML Wig Gold Medal for Clinical Research; lifetime achievement award from Indian Society of Vascular and Interventional Radiology; Gold Medals from Asia-Pacific Society of Cardiovascular and Interventional Radiology, Indian Society of Vascular and Interventional Radiology and Mongolian Society of Interventional Radiology; distinguished fellowships from the European Society of Cardiovascular and Interventional Radiology and Chinese Society of Interventional Radiology, as well as various national and international orations.

xi

1

Chest Radiograph in Heart Disease Sanjiv Sharma, Niraj Nirmal Pandey, Pujitha Vidiyala, and Amarinder Singh Malhi

Imaging algorithms in any form of heart disease should always begin with a chest radiograph. It is the cheapest and most widely available technique for imaging the heart. Careful interpretation of the radiograph provides useful diagnostic information for identifying morphologic abnormalities as well as assessing their impact on the function of the heart. Even though it often provides only inferential information, and despite the availability of advanced orthogonal imaging techniques for more comprehensive evaluation of the cardiovascular system, chest radiograph is the most widely used first examination in all patients with suspected heart disease. The information contained in it can be retained as a permanent record for reference in the future and for comparison during follow-up and for prognosticating the outcomes of treatment. It is essential to understand the normal anatomy in order to appropriately interpret the various cardiac pathologies. In elective settings, an optimal acquisition of the chest radiograph is essential for its suitability for interpretation. One should pay careful attention to the positioning of the patient, phase of respiration during exposure, appropriate KVP and mAs, and posture, among others, during acquisition. The radiograph should be obtained in an erect posture and in the postero-anterior (PA) projection; the front part of the chest in close proximity to the x-ray plate, with the x-rays entering the body from the back. This helps to minimize magnification of the cardiac structures. The patient should be positioned such that there is no rotation to any side (medial ends of the clavicle should be equidistant from the anatomic mid-line, identified by joining two or more oval shadows of the spinous processes), and the scapulae should have been removed from the area of interest by appropriate rotation and positioning of the arms. The film to x-ray focus distance should be optimized to prevent undue magnification and

S. Sharma (*) · N. N. Pandey · P. Vidiyala · A. S. Malhi Department of Cardio Vascular Radiology and Endovascular Interventions, AIIMS, Ansari Nagar, New Delhi, India

excessive radiation. The radiographic exposure factors should be adequate to show the inter-vertebral disc spaces up to tracheal bifurcation and not below it. This will prevent misreading of pulmonary vascularity. The x-ray exposure should be made in suspended deep inspiration to push the domes of diaphragm as low as possible. These basic steps will ensure that acquired radiograph is well suited for interpretation for heart disease (Fig. 1.1). In selected situations in sick patients in the emergency room and in the intensive care units (ICU), if radiographs in erect posture are not feasible, a semi-erect acquisition in an antero-posterior (AP) or PA view by raising the back support can be used to detect gross abnormalities in the pulmonary parenchyma, pulmonary vascularity, and pleural effusions. Even though such radiographs

Fig. 1.1  A normal chest radiograph in postero-anterior view showing the appropriate positioning (note the distance between oval-shaped spinous processes and the medial ends of clavicle, position of the scapulae), centering, exposure (clear disc spaces up to tracheal bifurcation and in a blur below this level), and phase of respiration during exposure (deep inspiration with low domes of diaphragm)

© The Author(s), under exclusive license to Springer Nature Singapore Pte Ltd. 2023 S. Sharma (ed.), Case-based Atlas of Cardiac Imaging, https://doi.org/10.1007/978-981-99-5620-3_1

1

2

provide limited information, they can be useful in differential diagnosis, assessing response to therapy such as those in the setting of pulmonary edema or lung consolidation and can help prognosticate the outcome of treatment in selected situations. The shape of the heart can vary widely with the body habitus. In tall thin individuals, it is long and narrow. In obese or short-statured individuals, the shape of the heart is often transverse, and in well-built individuals, it may be oblique or globular. Normally, the heart shadow is more toward the left than right (about two-thirds on the left and one-third on the right) in adults, but this is very variable and is, at best, a nebulous clue to the underlying disease. In infants, the shape of the heart is frequently transverse or globular and may be more in midline, equally on either side. Overlying thymic tissue in infants and young children frequently overshadows the pedicle of the heart and great vessels, limiting appropriate interpretation, at times.

1.1 Contour of the Heart 1.1.1 Postero-anterior (PA) View (Fig. 1.2) From above downward, the right heart border is formed by the superior vena cava (SVC) (it widens laterally in its upper part) and the lateral border of right atrium (RA). The SVC-RA junction is identified by a notch. The upper right heart border may become convex in old age due to dilatation

S. Sharma et al.

and elongation aorta, referred to as the unfolding of aorta. The convexity represents the outer margin of the ascending aorta and projects outside the shadow of SVC. This happens because the aorta is relatively fixed at its origin at the aortic valve and at its exit from the thorax at the aortic hiatus. When it dilates in response to age-related/atherosclerotic changes, it spreads sideways resulting in shadows along the right upper heart border (ascending aorta) as well in the left parasternal region behind the heart (descending thoracic aorta). A short portion of the inferior vena cava (IVC) can be seen as a triangular shadow in the lowermost part of the right border, adjacent to the right cardio-phrenic angle in tall thin individuals and is visible in up to 15% of chest radiographs. The left heart border is formed from above downward by the aortic knuckle (highest and posterior part of the aortic arch), a short segment of concavity that represents the region of ductus arteriosus (it becomes flat or bulges outward in patients with a patent ductus arteriosus (PDA), and an area of fullness or a small bulge just below this is represented by a combination of shadows from pulmonary trunk and the proximal left pulmonary artery and is called the pulmonary artery segment (PAS) or the pulmonary conus. It is flat or slightly concave. Prominent PAS is a normal variant in young females and children, but can also be seen in the setting of an idiopathic pulmonary trunk dilatation, valvular pulmonary stenosis (PS), pulmonary artery aneurysm, and pulmonary arterial hypertension (PAH), among others. Below the PAS, there is a small concavity represented by the left atrial appendage (LAA). In conditions with enlargement of LAA, this segment becomes flat or is replaced by a bulge. This is also known as the “third mogul.” The lowermost part of the left heart border is smoothly convex outward and is produced by the outer margin of the left ventricle (LV). This segment is relatively straight in tall thin individuals and can be convex and rounded in obese and those with short stature, largely due to the position of the diaphragm. It is important to remember that the left atrium (LA) and the right ventricle (RV) are midline structures, posteriorly and anteriorly respectively, that do not contribute to the heart borders in frontal projections.

1.1.2 Lateral View (Fig. 1.3)

Fig. 1.2  A normal chest radiograph in postero-anterior view showing normal cardiac borders (RA right atrium, LV left ventricle, SVC superior vena cava)

Detailed description of the normal anatomy in this projection is beyond the scope of this book. Among the pertinent points, the anterior margin of the cardiac shadow is formed by the RA appendage above and the RV below. There is normally a retrosternal area of lucency between the cardiac shadow and the sternum. This region gets obliterated in patients with an enlarged right heart. The posterior border of cardiac shadow is formed by LA above and by LV below. IVC may project in

1  Chest Radiograph in Heart Disease

Fig. 1.3 A chest radiograph in lateral view showing the normal retrosternal translucency (arrow) and the triangular lung lucency bounded by the vertebral column posteriorly, diaphragm below, and the posterior margin of LV antero-superiorly (arrowhead)

the lowermost segment. It is important to remember that there is a roughly triangular portion of translucent lung parenchyma that is visible in normal radiograph; bounded by the posterior margin of left ventricle antero-superiorly, vertebral column posteriorly, and the dome of diaphragm below. In patients with LV enlargement, this area is overshadowed by the enlarged heart. Proper understanding of a normal chest radiograph, as well as the appearances of normal variations, is essential for its gainful utilization in patient management. One should use a clearly defined algorithm for its interpretation so that useful information is sequentially obtained and used to establish a probable diagnosis. A proposed algorithm for interpretation of chest radiograph in heart disease should be as follows: 1. Assessment of situs 2. Cardiac size 3. Chamber enlargement 4. Aorta 5. Pulmonary vasculature 6. Lung parenchyma 7. Calcifications 8. Bones and soft tissues 9. Prosthesis and post-operative changes, if any

3

Fig. 1.4  Chest radiograph in PA view showing the normal viscero-­ atrial situs. Note the ascending aorta (arrowhead) and right atrium (arrow) on right side and opposite to the fundus of the stomach (long arrow). Note also the right bronchus more in line with the trachea

1.1.3 Situs Evaluation The evaluation of a chest radiograph, irrespective of the clinical indication, should always begin with the assessment of the situs. Basically, the situs refers to the relationship of various anatomic structures in the thorax to those in the abdomen. These include the location of the ascending aorta, systemic venous (morphological right) atrium, morphological LV and the tracheal bifurcation, and distribution of the bronchi in the thorax; and the location of the liver, spleen(s), and gastric fundus in the abdomen. Position of the abdominal aorta, IVC, and intestines, among others, is less reliable and is not routinely used for this decision-making. In situs solitus (Figs.  1.4 and 1.5) in the abdomen, the inferior vena cava (IVC) and liver are located on the right side and fundus of stomach (identified by air-fluid level in it), aorta, and spleen are located on the left side. The morphological right atrium (RA) is located on the right side, receives the systemic venous blood, and lies opposite to the gastric fundus. This relationship is known as the “viscero-atrial situs” (Figs. 1.6 and 1.7). Ascending aorta lies on the same side as the systemic venous atrium and on the side opposite to the gastric fundus. Whenever this relationship is breached, corrected transposition of great arteries is almost always present. It is important to re-emphasize that the location of the ascending aorta and the gastric fundus on the same side is virtually pathognomonic of corrected transposition of

4

Fig. 1.5  Chest radiograph PA view showing abdominal situs solitus with dextrocardia. Note that the cardiac situs is reversed with the cardiac apex on the right side. This is also known as dextroversion or isolated dextrocardia and is frequently associated with CHD

Fig. 1.6  Chest radiograph in PA view showing abdominal situs inversus with levocardia. This condition is almost always associated with complex CHD. Note that the bronchial and abdominal situs are inverted, the shorter and wider bronchus is present on the left, and long narrow more abruptly turning bronchus is present on the right side (arrow). The gastric fundus with air fluid level is present on the right side and the liver shadow is on the left side

great arteries (Fig. 1.8). The right main bronchus is shorter, wider, more vertically oriented, and in line with trachea when compared to the left main bronchus. The right lung is tri-lobed and the left lung is bi-lobed. This is called as “bron-

S. Sharma et al.

Fig. 1.7  Chest radiograph PA view showing an example of situs ambiguous. Note the midline liver, heart, and ambiguous gastric fundus. The bronchial situs is not well seen in this image

Fig. 1.8  Chest radiograph in PA view in a patient with abdominal situs solitus and corrected transposition of great arteries. Note the absence of ascending aortic shadow on the right side and a bulge overlying the pulmonary conus (the shadow of the ascending on the side of gastric fundus)

chial situs.” The dome of diaphragm on the side of cardiac apex is lower. It is important to remember that the position of the liver does not determine this relationship. In normal subjects, the left dome is lower because the cardiac apex is on the left side. It is also worth remembering that the presence of pulmonary conus in its normal location is an important

1  Chest Radiograph in Heart Disease

5

inferential point as it denotes that there are two ventricles and two great arteries and that these are normally connected.

1.1.4 Heart Size One should remember that chest radiograph in PA view provides only a rough estimate of the size of the heart and may show normal measurements in presence of cardiomegaly. Assessment of cardiac volumes from PA and lateral views provides a more accurate assessment but is cumbersome and has been largely abandoned in view of the availability of more accurate non-invasive methods, such as ECHO, to assess this. Assessment of the true heart size can also be done by orthodiagraphy, which is rarely used in day-to-day practice. Chest radiograph obtained at a focus-film distance (FFD) of 6 feet provides the minimum magnification (heart size is about 1 cm greater in width than that obtained with orthodiagraphic measurements). If the FFD is doubled to 12 feet, the measurements by both techniques are similar. But this is not practical due to various challenges, including that of increased scatter and radiation exposures. Therefore, the assessed heart size at 6 feet is taken as a reasonable representative for assessing cardiac enlargement despite minimal magnification. The dimensions of the heart on a chest radiograph can depend on many factors; including age, height, sex, occupation, race, body weight, phase of respiration, and the presence of lung and abdominal pathology, which cause elevation or depression of the diaphragm, among others. The easiest way to measure the size of the heart is to assess the transverse cardio-thoracic ratio (CTR), the most adopted practical method, even though it is, at best, a very rough estimate of the actual heart size. CTR represents the ratio between the widest transverse diameter of the heart and the widest intra-­ thoracic diameter above the level of the diaphragm in PA view. The widest diameter of the heart is measured by adding the distances from the outmost point on the right heart border and the distance from the outmost point of the left heart border to the anatomic mid-line. The sum of the above two measurements, which may not necessarily be in a straight line, provides the widest transverse diameter of heart. The widest intra-thoracic diameter is measured between the inner margins of the ribs at widest point above the diaphragm (Fig. 1.9). A CTR of >50% is considered as cardiomegaly. The normal upper limit is 60% in neonates and 55% in infants. Any condition that reduces the transverse thoracic diameter, among them, elderly and those with osteoporosis, may result in an increase in CTR, due to infolding of the ribs. Many different classification systems have been described for grading cardiomegaly as mild, moderate, or severe, but these classifications are inaccurate and not widely used. The one that we use frequently defines cardiomegaly as mild if the transverse diameter of the heart is within 2 cm beyond the widest trans-

Fig. 1.9  Measuring cardio-thoracic ratio (CTR) in chest radiograph PA view. The sum of a (distance between the outmost point on the right heart border and the anatomic midline) and b (the distance between the outmost point on the left heart border and the anatomic midline) divided by c (the widest intrathoracic diameter above the diaphragm). CTR = a + b/c

Fig. 1.10  Chest radiograph PA view showing severe cardiomegaly, as evidenced by the lateral border reaching up to the lateral chest wall

verse diameter on either side, severe if the cardiac border touches the lateral chest wall on either side (Fig. 1.10), and moderate for all other enlargements in between the above two categories (Fig. 1.11). This classification has served us well over the last many decades and corresponds roughly with estimates by other imaging techniques.

6

S. Sharma et al.

1.2 Radiological Signs of Enlarged Cardiac Chambers 1.2.1 Right Atrium (RA)

Fig. 1.11 Chest radiograph in PA view showing moderate cardiomegaly

Fig. 1.12  Measuring the height of right atrium (RA). This is the most reliable sign of RA enlargement where the distance between the arch and SVC-RA junction (yellow line) is less than that between SVC-RA

With the increase in its size, there is increased convexity of right heart border that extends more laterally to the right, away from the midline. The distance from outermost right heart border to the right lateral vertebral border of >3 cm or that from the anatomic mid-line of >4.5 cm may suggest an enlargement of the RA.  However, the above measurements provide only a nebulous clue to RA enlargement and are not reliable. The most reliable sign of RA enlargement in PA view is an increase in the height of RA shadow. Normally, the distance between the top of the arch and the SVC-RA junction (X) is more than the distance between the SVC-RA junction and right cardiophrenic angle (Y). RA is considered as enlarged if Y is more than X (Fig. 1.12). This sign is up to 85% specific for detecting an enlarged RA. Due to right-sided enlargement, the heart as a whole rotates medially along its long axis, resulting in the pedicle of the heart becoming less prominent and the ascending aortic shadow disappearing behind the mediastinum. This radiographic appearance is suggestive of an ASD.

junction and right cardiophrenic angle (black line). Normally, the reverse is true. Note that the inferior margin of enlarged RA is seen in continuation with the diaphragm

1  Chest Radiograph in Heart Disease

1.2.2 Left Atrium (LA) Due to its midline posterior location, this chamber does not normally contribute to any heart border in the PA projection. It is bounded above by the tracheal bifurcation. The enlargement of LA is at first superior, resulting in the upward displacement of left bronchus. But this sign is difficult to identify on a chest radiograph due to insufficient penetration for these structures. As the LA enlarges further in this direction, it results in the widening of the carina (Fig. 1.13). This is a useful sign for detecting LA enlargement. One should remember that carinal angle widening can be caused by a variety of other causes but LA enlargement is the most common cause. This angle normally ranges between 51 and 71 degrees. With further increase in its size, the LA enlarges in all directions other than anterior. The outer margin of an enlarged LA may be seen outside the RA shadow. This is referred to as a double density that is seen within or outside the cardiac shadow on the right side (Fig. 1.13). The posterior enlargement of LA can cause indentation and displace-

Fig. 1.13  Chest radiograph in PA view showing features of left atrial (LA) enlargement in a patient with mitral stenosis and tricuspid regurgitation. Note the widening carinal angle (obtuse), lifting up of left bronchus and a double density projected through the right heart border (red lines). Also note that the LA shadow always converges medial above the diaphragm and never touches it. In comparison, the lateral margin of RA (yellow line) touches the diaphragm. There is also evidence of PVH and PAH

7

ment of the esophagus as well as the descending thoracic aorta posteriorly. With further enlargement, LA can form the right heart border almost throughout up to the region of the SVC. In presence of double density, it is important to remember that LA margin never touches the right diaphragm and turns medially behind the heart shadow before reaching the diaphragm (Fig. 1.13), whereas the RA outer margin merges with the shadow of the right dome of diaphragm. LA enlargement can be accompanied by the enlargement of LA appendage and, when seen, this is virtually pathognomonic of rheumatic etiology (Figs. 1.14 and 1.15). The enlarged LA can also cause an extrinsic compression of the lower lobe bronchi on the left or right side, more often on left. This can result in radiographic changes in the lungs due to collapse, bronchiectasis, obstructive emphysema, or recurrent infection distal to the extrinsic compression of the airway (Fig.  1.16). An enlarged LA is termed as aneurismal if it touches the lateral chest wall on either side. In cases of chronic rheumatic heart disease, calcification of mitral valve can be appreciated on the radiograph. Its presence supplements the diagnosis of mitral stenosis (Fig. 1.17). However, annular calcification in the elderly can be an age-related phenomenon. It is important to remember that calcification in rheumatic heart disease, in contrast, is mainly in the valve leaflet, and can extend to the other structures in the valve apparatus in advanced disease. It is also possible to make a semi-quantitative assessment of the LA size on a frontal chest radiograph. A rough estimate of LA size can be

Fig. 1.14  Chest radiograph in PA view showing the straightening of left heart border in a patient with rheumatic mitral stenosis. Note the prominent pulmonary conus, enlarged LA, and LAA contributing to the straightening of the left border. There is PVH, PAH, and a right diaphragmatic hump as well

8

S. Sharma et al.

Fig. 1.15  Chest radiograph in PA view in a patient with rheumatic MD and MR showing a typical bulge in left mid heart border caused by enlarged LAA. This is also referred to as the “third mogul.” Also note cardiomegaly with biventricular configuration, double density of enlarged LA, PVH, and PAH Fig. 1.17  Chest radiograph in PA view in a patient with rheumatic MS and MR showing calcification of the mitral valve. Note that the calcification is below a line drawn from the right cardiophrenic angle to the base of the PAS. Also note the double density in right heart border, the widening of carina, and bulge of enlarged LAA

obtained by measuring the distance between the outmost point of the double density in right heart border and the midpoint of left main bronchus. By following this measurement in serial chest radiographs, a change in the size of LA can be documented to prognosticate the outcome of treatment (Fig. 1.18).

1.2.3 Left Ventricle (LV)

Fig. 1.16  Chest radiograph in PA view in a patient with ASD and MR showing left lower lobe collapse (green arrows) due to compression of left lower lobe bronchus secondary to LA enlargement. Note the widening of carina with obtuse angle between right and left bronchi (red lines)

It forms the left lower heart border and its enlargement is largely in the left and posterior directions. Whereas its hypertrophy produces rounding of the apex, its dilatation causes elongation in the left and downward direction. This appearance along with rounding of the apex results in an exaggeration of the smooth concavity of the left mid heart border (Fig.  1.19). In the lateral view, this is best seen as an encroachment upon the lucency of the lung in pre-vertebral space.

1  Chest Radiograph in Heart Disease

a

Fig. 1.18  Serial chest radiographs in PA view in a patient with rheumatic MS before (a) and after (b) surgery showing regression of LA size. In this projection, the LA size is measured by drawing a line from the point of maximum convexity of double density on right side to the

9

b

mid-point of left main bronchus. This distance can be a rough estimate of LA size in serial chest radiographs and can be used for assessing its regression after treatment. Note also the regression of heart size, disappearance of PVH, and mitral valve prosthesis in (b)

1.2.4 Right Ventricle (RV) Due to its mid-line anterior location, this chamber does not normally contribute to the formation of any heart border in the PA projection. When enlarged, it grows in an anterior and leftward direction. The RV outflow tract enlargement is best seen as a bulge adjacent to the region of LAA overshadowing it in frontal view and as an obliteration of the retrosternal lucency in the lateral view. When enlargement is significant, PAS may become prominent in the frontal projection. With the enlargement of RV, the heart tends to rotate on its long axis, shifting and rotating the apex wherein a portion of the left heart border may be formed by this ventricle by causing rotation of LV to the left with elevation of the apex. This results in a typical radiographic appearance due to lifting of left lower heart border producing a double convexity contributed by the enlarged RV, and displaced and rotated LV (“double arc” sign) (Fig. 1.20). The above rotation also shifts the aorta to the right, such that the aortic knuckle becomes less prominent. In the lateral view, RV enlargement causes an obliteration of the normal lung lucency in the retrosternal space. Fig. 1.19  Chest radiograph in PA view in a patient with Marfan’s syndrome showing features of LV enlargement. Note the rounded shape of cardiac apex that is also shifting downward and exaggeration of the concavity in the left mid heart border. There is also dilatation of the ascending aorta and arch, and mild PVH due to the underlying disease

10

S. Sharma et al.

Fig. 1.20  RV enlargement showing double arc sign

1.2.5 Aorta As described earlier, the ascending aorta is seen through the shadow of SVC in normal subjects but does not form the border on right side. Unfolding of aorta, in old age and in disease states, results in its dilatation and elongation. This is seen as a bulge in the right heart border beyond the SVC. As described earlier, aortic knuckle forms the uppermost bulge in left upper cardiac border. In infants and young children, this is not well seen and becomes more prominent as the age advances. It may show curvilinear calcification along its lateral and inferior margins. The descending aorta travels in left parasternal location within the mediastinal shadow as an oblique line that runs downward from the left parasternal location toward mid-line. While reading a chest radiograph, a comment on the following should be included in the report:

Fig. 1.21  Chest radiograph in PA view in a patient showing typical features of right aortic arch. Note the indentation of aortic knuckle on the right margin of trachea above its bifurcation, lateral displacement of the SVC shadow, and right-sided descending aorta for a variable distance

1. Is it a D-loop or L-loop (Fig. 1.21)? 2. Location of the aortic knuckle-on right or left side. 3. Normal or enlarged (if enlarged, is it uniform or there is a disproportionate enlargement of the ascending, arch or descending aorta?) (Figs. 1.22 and 1.23). 4. Calcification: If present, its location (knuckle, descending thoracic or ascending aorta), pattern (curvilinear, punctuate or plaque like), and extent should be commented on (Fig. 1.24). 5. Contour abnormalities of the thoracic aortic outline, such as retraction of descending thoracic aorta in Takayasu’s aortitis (Fig. 1.25). Evaluation of the aorta provides useful clues for localization of various congenital and acquired heart diseases.

Fig. 1.22  Chest radiograph in PA view in a patient with valvular AS showing a typical focal post-stenotic dilatation of the proximal ascending aorta (arrow)

1  Chest Radiograph in Heart Disease

Fig. 1.23  Chest radiograph in PA view in a patient with PDA showing a typical disproportionate dilatation of the arch with an unremarkable ascending aorta. Also note LV configuration of heart, dilated PAS, plethora, and kyphoscoliosis

11

Fig. 1.25  Chest radiograph in PA view in a patient with Takayasu’s arteritis with uncontrolled hypertension showing typical retraction of the descending thoracic aorta (arrows). Note also moderate cardiomegaly with LV configuration, dilated ascending aorta and the arch, and mild PVH.  There are also multiple calcific lesions in right lung and hilar nodes, due to old tuberculosis

Information on all of the above helps develop an algorithmic approach to diagnosis in view of their localizing value. The respective individual abnormalities and their contribution to building algorithms will be discussed with illustrations in the subsequent section.

1.2.6 Lung Vasculature The chest radiograph is useful for comprehensive evaluation of the pulmonary vessels. The pulmonary blood flow reflects the pathophysiology of the heart in a reasonably accurate manner as these vessels are clearly seen on the radiograph and the abnormal patterns of blood flow can be identified. It has been appropriately suggested that the pulmonary vasculature is the “eye” through which one looks at the heart!

1.2.7 Normal Radiographic Anatomy

Fig. 1.24  Chest radiograph in a 68-year-old man with calcifications in the aortic knuckle which is the most common site of atherosclerotic calcifications in the aorta

The pulmonary valve is the starting point of pulmonary circulation. PAS on a chest radiograph is produced by the confluence of overlapped segments of MPA and left pulmonary artery (LPA). The floor of PAS is formed by the left border of main pulmonary artery (MPA). Normal PAS is slightly concave or straight. The enlargement of MPA causes bulging of the PAS. A short distance above the pulmonary valve, slightly

12

to the left of midline but within the mediastinum, the MPA bifurcates into the right (RPA) and left pulmonary arteries. Both these branches travel laterally through the mediastinum and divide in each lung hilum into their respective branches. The intrapulmonary arteries run parallel to the bronchi, divide in an orderly manner, and gradually taper toward the periphery as they travel distally. The LPA and left hilum are approximately 1 cm above the RPA because of the course above or below the corresponding bronchus, resulting in the nomenclature of ep-arterial or hyp-arterial bronchus, accordingly. The veins have larger diameter, less distinct margins, are less dense, and branchless often than the companion arteries. In terms of their location, these are medial to the companion artery in upper zone, overlap each other in mid-zone, and are lateral to the artery in lower zone. Distension of upper lobe vessels is said to be present when the upper lobe pulmonary veins measure >3 mm in diameter opposite the first rib anteriorly. This is referred to as “cephalization” secondary to redistribution of blood flow in presence of pulmonary venous hypertension (PVH). The pulmonary veins converge into the left atrium (LA) in the region below the hilum. This confluence can sometimes mimic a double density but is a normal variation and does not cause widening of carina. It can sometimes be difficult to differentiate between the artery and vein due to variations in the arterial supply and venous drainage. Normally, as the pulmonary arteries branch, they gradually lose diameter, and at each degree branching, this loss of diameter is less than 50% of the proximal measurement. Any loss of diameter above this is categorized as pruning, a reliable sign of the presence of pulmonary artery hypertension (PAH). In a normal chest radiograph, the vessels become sparse as they reach the periphery and only up to 4–6 vessels are seen in the lateral 1/3rd of the lung. This number may be increased whenever there is a regional or generalized increase in pulmonary vascularity. The confluence of right upper lobe pulmonary vein and right descending pulmonary artery (RDPA) produces the shadow of the right hilum. These two vessels usually form a right angle at the confluence. The pulmonary blood flow determines the width of the pulmonary vessels. Normally, the RDPA has a straight or slightly concave lateral border. A convexity in this region suggests an enlargement, as in the presence of plethora or PAH, and a deep concavity suggests a decreased size, as in pulmonary oligemia. Whereas the width of RDPA is taken as a measure of its dilatation in adults (measured 1.25 cm distal to the origin of right upper lobe branch; normal range is 10–16 mm in men and 9–15 mm in women), the ratio of RDPA to trachea is used for this assessment in children (normally this ratio is 2 times the diameter of accompanying end-on bronchus (arrowhead) (a), enlarged PAS and dilated central Pas, and generalized increase in vessels in both lungs, (a

& b). In (a), aorta is normal, there is biventricular shape with LA enlargement suggesting an intracardiac shunt at ventricular level; (b) shows a large ascending aorta with LV configuration suggesting a shunt at the aortic root level

4. A generalized increase in the lung vascularity in all zones. 5. Presence of more than 6 vessels in peripheral lungs (normally up to 6 vessels are seen in the peripheral one-third). 6. Differential increase in the right upper zone vascularity may be seen in selected patients with an isolated VSD or D-TGA due to an abnormal orientation of the RV outflow tract toward the right.

evaluating it for pulmonary vascularity; an underexposed film can superficially resemble an increased vascularity and an overexposed film may give an erroneous impression of a decreased vasculature. In addition, normal individuals can, at times, have small pulmonary vessels. Pulmonary oligemia can be produced by a variety of disease states, including those caused by a pulmonary outflow tract and/or valvular obstruction, PAH from any cause, obstructing lesions in the pulmonary vasculature due to congenital, thrombo-embolic or inflammatory causes, tumors and vasculitis, among others. The findings on the chest radiograph may include reduction in the size of the pulmonary arteries and veins, small central vessels, and difficult to visualize peripheral arteries. The radiological signs include (Fig. 1.28):

On a chest radiograph, the radiological signs of plethora correlate better with the shunt size than the heart size. The assessment of plethora should be comprehensive, including overall clinical and radiographic features, and should not be based on measurements alone. Measurements should be used in conjunction with an overall clinical perspective.

1.2.9 Decreased Pulmonary Blood Flow (Pulmonary Oligemia) Its diagnosis is easy when severe but can be challenging if the decrease in pulmonary vascularity is minimal. It is important to ensure that the radiograph is optimally exposed before

1. Concave or absent MPA: the presence of pulmonary bay in the setting of Tetralogy of Fallot (TOF) or conditions superficially resembling it. 2. If the radiograph appears over-penetrated, despite an optimal exposure. 3. The size of RDPA is smaller than that for respective age, as described above. 4. 50% diameter and peripheral oligemia (a & b). Note the location of pruning in a relatively lateral position as well as calcification on the central PAs (arrows) (b)

1.2.11 Pulmonary Venous Hypertension (PVH)

Fig. 1.30  Chest radiograph in PA view in a patient with primary PAH showing normal heart size, RV configuration, dilated central PAs, peripheral pruning that is more medial as compared with that seen in L-R shunt (Fig. 1.29), no features of increased vascularity and peripheral oligemia

The normal pulmonary capillary wedge pressure (PCWP) ranges between 8 and 12 mmHg. When the pressure increases beyond this range, PVH is considered to be present. It produces characteristic radiographic changes that roughly parallel the increase in PCWP.  PVH has been graded as mild, moderate, and severe, depending on the degree of increase in PCWP.  Before interpreting the radiographic abnormalities, one must remember that the radiographic signs provide only a nebulous clue to the severity of PVH as their appearance is influenced by various factors, including the acute versus chronicity of disease, compensated versus decompensated cardiac status, underlying lung disease, and presence, duration and severity of PAH, among others. Mild PVH (PCWP, 13–19 mmHg): This is characterized by redistribution of blood flow in the lungs and is also referred to as cephalization of the blood flow (Fig. 1.31). The radiographic changes may be visible before the clinical signs and symptoms emerge. The radiographic signs include narrowing of lower zone vessels, loss of their distinct outline, effacement of the hilar angle, and dilatation of upper lobe vessels. With a further rise in PCWP, interstitial lung water is increased with a resultant rise in the pulmonary lymphatic drainage. This results in “cuffing” of fluid around the small

16

a

S. Sharma et al.

b

Fig. 1.31  Chest radiographs in PA view in two patients with rheumatic MS showing features of mild PVH, including cephalization as evidenced by spasm of lower lobe vessels (arrow), and distension of upper lobe vessels (long arrow) (a & b), haziness of hilum (arrowhead), and

indistinct vessel outline (b). Note also mild cardiomegaly with RV configuration, straightening of left heart border (a), and enlargement of LA and LAA (more pronounced in b)

bronchi. The excess interstitial water gravitates to dependent areas, and causes an increase in the interstitial pressure as well as the vascular resistance in lung bases. These changes result in diversion of blood to the upper lobes, leading to the radiographic appearances related to redistribution. At times, these typical features may not be seen in the presence of an underlying parenchymal lung disease, including basal emphysema, pulmonary fibrosis, and inflammatory parenchymal diseases of the upper lobe, especially pulmonary tuberculosis in this geographic location. Moderate PVH (PCWP, 20–24 mmHg): Interstitial edema (Fig.  1.32): This is caused by accumulation of fluid in the interstitial spaces and manifests radiographically by the presence of Kerley’s lines (septal and interstitial edema), peribronchial cuffing, and background haze in the visualized lung parenchyma subtly replacing the normal translucency of lungs, and pleural effusion. Kerley described the radiographic signs associated with thickening of various septae and cuffing of normal structures. Among them, the B lines, produced by thickening of the interlobular septae, are most reliably detected and most frequently seen as well. These appear as thin white lines, 1–2 mm thick and 1–2 cm long, in the lateral lung fields typically perpendicular to the costo-­phrenic angles, more so on the right side. The A lines, produced by thickened interlobar septae, are seen less frequently and appear as oblong thin white lines, 2–6 cm long and 25  mmHg): Alveolar edema (Fig.  1.33): As the plasma oncotic pressure is breached (>25 mmHg), and because by this stage the interstitial fluid starts accumulating much faster than it can be removed by the lymphatics, it starts accumulating within the alveoli and produces typical radiographic abnormalities. These are seen as semi-confluent shadows in the medial and paramedial parts of mid and lower zones on both sides. A typical “Bat’s Wing appearance” may be seen on the chest radiograph. This is caused by a combination of the effect of squeezing of the edema fluid toward the hilum by the expanding lung and the posture of the patient (Figs.  1.34 and 1.35). At times, an atypical pattern of radiographic appearance may be seen due to an underlying chronic parenchymal disease and a change in posture (for example, dependent position). In this stage, fluid also accumulates in the pleura if fluid formation by visceral pleura is faster than its absorption by the parietal pleura. In addition to the above, the other typical but inconsistent radiographic abnormalities associated with PVH include: 1. Secondary pulmonary hemosiderosis (Fig.  1.36): These are seen in patients with chronic long standing severe PVH and appear as bilaterally distributed military shadows superficially resembling military tuberculosis but showing subtle difference in the form of distribution (more dense in mid and lower zones and in the medial and paramedial locations as compared with tuberculosis that

1  Chest Radiograph in Heart Disease

17

Fig. 1.32  Chest radiograph in PA view in a patient with rheumatic MS showing features of PVH, including the presence of Kerley’s B lines best seen in right costo-phrenic angle (arrow). Also note other features including cephalization of blood flow, hazy hila, indistinct vessel outlines, and PAH, besides mild cardiomegaly and LA enlargement. In addition, bilateral miliary shadows are present suggesting secondary hemosiderosis

a

Fig. 1.33  Chest radiographs in PA view in two patients with rheumatic MS in failure showing features of severe PVH, including cephalization of blood flow, hazy hila, and vessels (a), bilateral pleural effusions, fluffy ill-defined alveolar shadows in medial and paramedial locations

may be more dense in the upper zones and is uniformly distribute in medial as well as lateral locations; shadows are smaller, more discrete, and less dense in ­hemosiderosis as compared to military tuberculosis that is more dense, less discrete, and larger in diameter). 2. Pulmonary ossification (Fig. 1.37): These appear as dense nodules, higher in radiographic density than the adjacent bone, are of varying sizes with no specific distribution pattern, appear as heterotopic bone formation following organization of intra-alveolar edema and are not related to either the chronicity or severity of PVH. These can be seen in mitral valve disease, LA myxomas, and left ventricular failure from any cause. 3. Pulmonary venous varix: Localized venous dilatation of pulmonary vein(s), typically at confluence of veins adjacent to LA. This can be caused by mitral valve disease but can also be seen as a part of hereditary hemorrhagic telangiectasia or in patients with an advanced liver failure. The varix can have different image morphologies, including sacular, tortuous, or confluent. Among them, the sacular type is typically seen on a chest radiograph as a nodular or rounded radiodense lesion(s), depending on its size, near the hilum and with vessels leading to and/or from it. In patients with mitral valve disease, the tell-tale signs of the underlying heart disease and features of PVH are evident. Sacular type most commonly involves the left upper lobe whereas the other two types are commonly seen in the right lower lobe.

b

of mid and lower zones (a & b), and loss of normal lung translucency (b). Also note mild cardiomegaly with RV configuration, LA and LAA enlargement in (b), and absence of bulge in the region of LAA (a) (arrow), suggestive of fibrosis/thrombus in it

18

Fig. 1.34  Chest radiograph in PA view in a patient with uncontrolled hypertension and acute myocardial infarction in failure showing features of acute pulmonary edema and a typical bat’s wing appearance.

S. Sharma et al.

Note the central and paracentral location of fluffy ill-defined alveolar shadows. Also note cardiomegaly, LV configuration, and an unfolded aorta

Fig. 1.35  Chest radiograph in PA view in a patient with acute non-­ cardiogenic alveolar pulmonary edema produced by over-hydration and showing typical semi-confluent shadows in both lungs in a characteristic distribution and showing air bronchogram in an alveolar consolidation. Note the central venous line in SVC and a relatively normal heart size

1.2.12 Asymmetrical Pulmonary Blood Flow or Pleonemia (Fig. 1.38) This refers to a differential vascularity among the two lungs and is can be seen a variety of diseases states, including: 1. Localized decrease: Obstruction to regional pulmonary blood flow from any cause, including thrombo-embolism, obstructive airway disease (COAD), vasculitis, or an arterial obstruction from any cause.

Fig. 1.36  Chest radiograph in PA view in a patient with rheumatic MS showing bilateral miliary shadows due to secondary pulmonary hemosiderosis. Note the typical distribution of discrete opacities in both lungs in central and paracentral location, more in mid and lower zones and relatively sparing the apices. The heart size is normal with RV configuration, LA and LAA enlargement, PVH, and PAH

2. Decrease in one lung: Produced by an obstruction to arterial flow, hemi-truncus, in isolation or in combination with an underlying CHD.

1  Chest Radiograph in Heart Disease

19

3. Increase in one lung: After shunt surgery (any lateralized shunt such as Blalock-Taussig shunt or Glenn shunt), PDA, or an anomalous origin of a PA from the aorta. Many disease states also produce specific bone abnormalities. As an example, rib notching in typical locations is associated with coarctation of the aorta at various anatomic sites. This subject will be discussed with illustrative examples in a subsequent section. Post-operative changes can help identify the type of surgery done (Fig. 1.39). Some examples may include, among others: • • • • • • • •

Fig. 1.37  Chest radiograph in PA view in a patient with rheumatic MS showing ossific nodules in both lower zones (arrows). Also note mild cardiomegaly with RV configuration, LA and LAA enlargement, PVH, and PAH

a

Fig. 1.38  Chest radiograph in PA view showing pleonemia in two patients; (a) a patient with TOF with absent pulmonary valve syndrome and LPA atresia showing large RPA and central arteries in right lung and absent vasculature in left lung. Also note cardiomegaly with right

Soft tissue disparity Rib regeneration Asymmetry of intercostal space Depressed outer rib sign Pleural thickening Wires or band Surgical clip, prosthetic valve Regional or unilateral alteration in lung vascularity

The presence of specific bone defects has been identified by specific CHD. Some pertinent examples include:

b

heart configuration, large aorta and pulmonary bay. (b) a patient with tricuspid atresia with increased flow in right lung and atresia of LPA. Also note mild cardiomegaly with globular shape, large aorta, and pulmonary bay

20

S. Sharma et al.

a

b

Fig. 1.39  Chest radiograph in PA view in two patients post left lateral thoracotomy for a shunt surgery showing loss of parallelism of the left 4th and 5th ribs posteriorly (a) and rib regeneration in same location (b) in another patient

• Pectus excavatum –– Mitral valve prolapse –– Marfan’s syndrome • Hyper segmented sternum –– Down’s syndrome • Pink patient with plethora –– 11 or 13 ribs—Down’s syndrome –– Kyphoscoliosis—ASD Whereas the above description on a step-by-step guide to reading a chest radiograph for heart disease is not exhaustive, it is intended to provide the reader a working approach in handling day-to-day real world clinical challenges. The chest radiograph mirrors the events produced by heart disease. A systematic step-by-step interpretation provides reliable clues to the underlying disease, including the hemodynamic alterations resulting from it. The radiographic appearances are useful for assessing the response to therapy. These also serve as a record for comparative assessment during follow-up after treatment in most situations. The following narrative has been structured to provide the readers with a systematic approach to integrate the chest radiographs in the diagnostic evaluation and management planning of various disease states affecting the heart.

1.3 Algorithmic Approach to Localization of Congenital Heart Disease by Chest Radiograph All algorithms for imaging of the heart in congenital heart disease (CHD) should begin with a chest radiograph. It is a simple, widely available, quick, and cheap test that yields useful information about the heart size, aorta, lung vasculature, and

parenchymal pathology that can be stored and reproduced reliably for serial comparison for establishing the diagnosis, prognosticating outcomes, and follow-up after therapy. Chest radiograph can be used to build an algorithm for step-by-step identification of the underlying heart defects, irrespective of whether the patient is pink or blue, in compensated or decompensated state, or with or without clinically manifest PAH. Information on the clinical signs suggesting presence or absence of cyanosis, PAH, and congestive failure, as well as the age of the patient, should be available before interpreting a chest radiograph for CHD as these can impact the radiographic interpretation, as discussed below. The appropriate acquisition requirements as well as the approach to reading a radiograph have been extensively described earlier in this narrative. A proposed algorithm for the classification of CHD, based on chest radiographic interpretation, is described below. Careful interpretation of the aortic shadow and lung vasculature is central to building this algorithm.

1.3.1 Left-to-Right (L-R) Shunt Assessment of the aorta is the key for localization of the level of the shunt as intra or extra-cardiac. If the chest radiograph shows a large aorta, the shunt is caused by an extra-cardiac lesion. If the aortic size is normal, the shunt is intra-cardiac. Based on the location and uniformity of aortic enlargement, these shunts can be further sub-categorized in the extra-­cardiac group.

1.3.2 Large Aorta The extracardiac shunt can be at various levels. If the enlargement of the aortic knuckle is more prominent and is disproportionate to the ascending aorta, the L-R shunt is caused by

1  Chest Radiograph in Heart Disease

a

21

b

Fig. 1.40  Chest radiograph in PA view in two patients with small (a) and large (b) PDA. (a) shows a normal heart size with LV configuration, a large aorta with disproportionate enlargement of knuckle, fullness in the region of ductus just below the knuckle, dilated PAS, and a subtle

increase in pulmonary vascularity. (b) shows moderate cardiomegaly with LV and LA enlargement, large aorta with enlargement of knuckle, a bulge in the region of ductus, dilated PAS, and features of pulmonary plethora

a shunt lesion at the level of the knuckle, a patent ductus arteriosus (PDA) (Fig. 1.40). In this defect, the heart is usually normal in size or may occasionally show a mild cardiomegaly with an LV configuration. The aortic knuckle and the PAS are enlarged and pulmonary vasculature is increased, producing features of plethora. As mentioned earlier, the region of concavity between the aortic knuckle and the PA conus becomes full or is replaced by a bulge due to the dilated ductus. Some evidence of LA enlargement may be seen. In addition, the radiograph may often show features of PAH.  Occasionally, there may be calcification at the PDA site. In patients in whom the enlargement is most conspicuous in the ascending aorta, the shunt is localized to the level of the aortic root. The pertinent examples include a ruptured aneurysm of the sinus of Valsalva (RSOV), an aorto-­ pulmonary window, or a coronary arteriovenous fistula.

1.3.3 RSOV [2] There is cardiomegaly with a left or bi-ventricular configuration. The radiograph will often show an abnormal shadow of the aneurysm at the level of the aortic root, usually seen as an abnormal density on the right side and occasionally with a typical curvilinear calcification in it. The aorta is enlarged with a disproportionate dilatation of the aortic root. The PAS is prominent and pulmonary vasculature is increased with resultant features of plethora. When RSOV is clinically symptomatic, radiographic features of PVH are also present. The radiographic picture is influenced by the location of RSOV, depending on which sinus is involved, whether rup-

Fig. 1.41  Chest radiograph in PA view in a patient with an RSOV (right sinus to RV) showing moderate cardiomegaly, LV configuration, enlarged aorta with disproportionate dilatation of the ascending aorta, pulmonary plethora, and PVH

tured or not, and if so into which cardiac chamber (Fig. 1.41). Most commonly, the right coronary sinus is involved. Typically, it ruptures into the right ventricle. Less commonly, it may rupture into the RA or the flow may be bidirectional, into the RA and RV.  It is important to remember that the

22

S. Sharma et al.

a

b

Fig. 1.42  Chest radiograph in PA (a) and lateral (b) views in a patient with an unruptured aneurysm of noncoronary sinus showing a pathognomonic appearance of mild cardiomegaly, large aorta with dispropor-

tionate dilatation of ascending aorta, curvilinear calcification within the aneurysm in a characteristic location, and presence of a pacemaker lead in the RV

presence of both pulmonary plethora and PVH is typically seen in RSOV. Occasionally, the sinus of Valsalva aneurysm may be unruptured at the time of presentation. Such lesions when located in the nonocoronary sinus present with a complete heart block due to the aneurysm burrowing into the interventricular septum and disrupting the bundle of His. The pathognomonic chest radiographic appearance consists of a normal heart size to mild cardiomegaly, an abnormal shadow of the aneurysm in the region of the aortic root, especially if calcified, normal lung vascularity, and presence of a pacemaker lead (Fig. 1.42).

1.3.5 Coronary Arteriovenous Fistula

1.3.4 Aorto-pulmonary Window Usually, there is a mild to moderate cardiomegaly, with left or biventricular configuration (depending on the severity of PAH). The ascending aorta shows a disproportionate dilatation. The PAS is enlarged, pulmonary vasculature is increased, and features of PAH are seen (Fig. 1.43). Lungs also show features of mosaic perfusion.

This defect is usually associated with a small L-R shunt and the radiograph is often normal. If the shunt is large, there may be mild cardiomegaly with LV configuration and a focal dilatation of the ascending aorta. The PAS may be enlarged and the pulmonary vasculature is normal to increased. The abnormal shadow of the aneurysm or the dilated proximal coronary artery in the region of coronary artery calcification (CAC) triangle may be seen (Fig. 1.44). Such shadows overlie the region of the LAA and should be included in its differential diagnosis.

1.3.6 Normal Aorta A normal aortic shadow in a frontal chest radiograph in the presence of increased lung vascularity in a pink patient should suggest an intra-cardiac L-R shunt. This shunt can be located before the tricuspid valve or beyond the tricuspid valve.

1  Chest Radiograph in Heart Disease

23

1.3.7 Pre-tricuspid Shunt The shunts before the tricuspid valve include an atrial septal defect (ASD) or a partial anomalous pulmonary venous connection (PAPVC).

1.3.8 ASD

Fig. 1.43  Chest radiograph in a patient with an AP Window showing moderate cardiomegaly with LV configuration, LA enlargement, prominent aorta, plethora, and PAH

Fig. 1.44  Chest radiograph in PA view in a patient with coronary AV fistula showing a normal heart size, disproportionate dilatation of ascending aorta, an abnormal shadow in the region of coronary artery calcification triangle overlying the region of LAA, and mildly increased pulmonary vascularity

Usually, the heart is not enlarged; mild cardiomegaly may be seen in about 1/4th patients. Typically, RA is enlarged and the aorta is normal; and rotation of the heart on its long axis due to enlargement of the right heart results in medial displacement of the ascending aortic shadow. As a result, it is not seen in its normal location within the upper right cardiac border. This sign is typically seen in presence of an ASD. In addition, there is a leftward shift of the heart. The pulmonary vasculature is increased and there may be evidence of PAH (Fig. 1.45). The radiographic appearance may be modified if associated anomalies are present. In the setting of ASD, one should suspect an associated mitral valve disease if there is LA and LA appendage enlargement, presence of PVH (especially if Kerley’s lines), or a calcified mitral valve. “Lutembacher’s syndrome” refers to the association of ASD with mitral stenosis of rheumatic etiology (Fig. 1.46) [3]. LA enlargement is uncommon in an ASD and, if present, should

Fig. 1.45  Chest radiograph in PA view in a patient with an ASD showing a typical appearance including mild cardiomegaly with RV configuration, RA enlargement, absence of ascending aorta from the right upper cardiac border, pulmonary plethora, and PAH

24

Fig. 1.46  Chest radiograph in PA view in a patient with an ASD with Lutembacher’s syndrome showing moderate cardiomegaly with RA and LA enlargement, normal aorta, PAH, and PVH

S. Sharma et al.

Fig. 1.47  Chest radiograph in PA view in a patient with PAPVC showing mild cardiomegaly, RA enlargement, dilatation of SVC (arrows), and pulmonary plethora

make one suspect a primum type of defect or an associated mitral regurgitation, in addition to the above-described association. The presence of LV configuration and/or distended upper lobe vessels due to LV dysfunction may be seen in the setting of myocardial ischemia, hypertensive heart disease, or elderly age.

1.3.9 PAPVC The chest radiograph may superficially resemble that of an ASD. One should suspect the diagnosis of PAPVC if there is an enlarged superior vena cava (SVC) in this setting (Fig. 1.47) or if an abnormally directed pulmonary vein, typically seen as the “scimitar” sign in an infra-diaphragmatic drainage, is seen (Fig. 1.48). Sometimes, an abnormal intrapulmonary course of the anomalous vein may also be evident.

1.3.10 L-R Shunt Beyond the Tricuspid Valve This is caused by a VSD and is the most common form of acyanotic CHD.  The radiographic findings depend on the shunt size, number, and size of VSD and the direction of predominant flow. In the presence of a small 2:1

Fig. 1.48  Chest radiograph in PA view in a patient with PAPVC of right pulmonary veins to the IVC showing a typical “scimitar sign” (arrows)

shunt, there may be mild to moderate cardiomegaly with a left or a bi-ventricular configuration, LA enlargement, a normal aorta, increased pulmonary vasculature, and features of PAH may be seen (Fig. 1.49). In about 3–5% patients, a differential increase in lung vascularity in the right upper zone may be seen due to rightward direction of pulmonary blood flow (Fig. 1.50a, b). Also, 50%) from one ventricle. This may be a double-outlet LV (DOLV) or a double-outlet RV (DORV). • Single outlet: –– Only aorta connected, PA atresia –– Only PA connected, aortic atresia –– Truncus arteriosus During development in embryonic life, the ventricles form a normal D-loop or a reversed L-loop. Since this does not depend on the visceral situs development, either L-loop or D-loop may be discordant or concordant relative to the viscero-atrial situs. Concordant: D-loop pattern with situs solitus/L-looping pattern with situs inversus. Discordant: D-loop pattern with situs inversus/L-looping pattern with situs solitus. Spatial position of the great vessels: The great arteries are identified by the branches they give off and not by their relationship to the ventriculo-arterial valves. Normally, the ascending aorta is positioned posterior and to the right of the pulmonary artery (PA); it gives rise to at least one coronary artery and many branches for systemic circulation. The main pulmonary artery (MPA) travels to the left and divides into the left (LPA) and right (RPA) pulmonary arteries. It is also important to remember the spatial relationship of the aortic and pulmonary valves. In a D-loop anatomy, the aortic valve is positioned inferior, posterior, and to the right; and the pulmonary valve is positioned anterior, superior, and to the left of the aortic valve. This relationship is disturbed in the presence of abnormal great vessel relationships, such as in the transposition of great arteries as well as double outlet syndromes. This will be discussed in detail in a later section. The location of the conus also helps in identifying the pulmonary artery—normally it is sub-pulmonic in  location. There are four different types of anatomy that can be identified in the conus: • Normal—this morphology has a subpulmonic conus. • When the great arteries are transposed, the conus is sub-­ aortic with the presence of mitral-pulmonary continuity.

45

• In a DORV, a bilateral conus is present. • In a DOLV, there is deficient conus beneath both semi-­ lunar valves. Assessment of associated abnormalities: After sequential chamber localization and assessment of connections, including their relationships in a step-wise manner, the associated intracardiac defects and malformations should be identified. This includes assessments at three levels; a) at the level of cardiac chamber-presence of atrial and ventricular septal defects, sizes of the chambers, ventricular outflow obstructions, among others, b) at the level of great vessels— presence of variations of major vessel branching, patency of ductus arteriosus, presence of significant aorto-pulmonary collaterals, hypoplasia of the arch, pulmonary artery anatomy and size, and the coronary arterial anatomy and c) any variations—these should be noted and described as part of any pre-surgical evaluation. Any shunts, stents, implants, and closure devices should be reported as well.

46

The above algorithm using the sequential approach has the potential to provide comprehensive information that can be used as a guide in decision-making for appropriate customized management in each patient. Careful interpretation and judicious utilization of above knowledge helps optimize management planning. The structured report of any imaging technique used for evaluation of a patient with a suspected CHD should include a comment on all of the following: • Are there two adequately sized ventricles? This helps determine a univentricular versus bi-ventricular approach in surgery. Accurate assessment of function helps prognosticate the outcome of surgery. • Are the pulmonary arteries confluent and good sized? This helps determine a single-stage versus two-stage correction (defines the need for shunt surgeries) and prognosticates outcomes. • Are there two normally located and functioning AV valves? AV regurgitation influences the surgical choice. • Are the semi-lunar valves and outflow tracts normal? This helps determine the surgical approach as well as decision-making in terms of infundibular resection, valvotomy, or replacement. • Is the coronary anatomy normal?—Variations, if present and not reported, may lead to damage to the artery during surgery and subsequent myocardial ischemia and complicated recovery. It can also influence the selection of surgical approach as well as the type of surgery planned—as an example, an anomalous coronary artery crossing the RVOT can influence plans for infundibular resection. Current-generation orthogonal imaging techniques have largely replaced conventional catheter angiography for the diagnosis of CHD. The advances in MRI and CT have the potential to provide reliable and reproducible hemodynamic assessment in most clinical situations, including shunt, gradient and flow measurements, volumetric assessment, and regurgitant volumes, among others. In addition, their large field of view, sequential high-resolution imaging capability, acquisition of continuous 3-D data sets, their reconstruction in any imaging plane, and objective evidence documentation

S. Sharma et al.

relatively free of intra- and interobserver variation provide huge advantages over echocardiography. There is also superior depiction of situs, central pulmonary arteries, A-V and V-A connections, systemic and pulmonary veins, and aorta. There is little doubt that in the imaging evaluation of a child with CHD today, orthogonal imaging is the way to go. A sequential approach can provide us with all the necessary information desired from the imaging techniques for best management practices. Both CT and MRI have common strengths, in terms of their ability for multiplanar, gated acquisitions, as well as the ability to acquire continuous data sets on an out-patient basis, among others. The various factors that influence the selection between them may include the speed of acquisition (need for breath-hold, immobilization of small children, sick patients), image resolution (imaging of coronary arteries, as an example), propensity for motion artifacts, need for functional assessment, radiation exposure, iodinated contrast medium usage, and some MRI specific challenges including claustrophobia and in situ metallic implants and pacemakers. Broadly, if only anatomic information is desired and speed of acquisition is the key, we generally prefer CT (TOF and similar clinical states as well as depiction of coronary arteries are a pertinent example); if functional information is essential for treatment planning, MRI may be preferred (Ebstein’s anomaly is a pertinent example). Both techniques can provide diagnostic quality images. It is just that MRI takes longer for image acquisition but has the advantage of circumventing the need for radiation exposure and use of iodinated contrast medium for those at risk. Overall, these techniques complement each other, are rapidly changing the way we manage patients with CHD, and have been responsible for rapid strides in improving the outcomes of treatment. Newer advances in MRI technologies, including 3-D cine cardiac MR and interactive MR fluoroscopy of heart without need for ECG gating or breath-hold, real-time cardiac MRI at a frame rate of 41 frames/sec (examination in real-time like ECHO), faster imaging techniques with high-­ quality sequences and parallel imaging, and single breath-­ hold 3-D sequences for function analysis (CT like imaging outcome), may be the potential game changers in the near future with the potential to rewrite the imaging algorithms.

3

Imaging in Tetralogy of Fallot Sreenivasa Narayana Raju

3.1

Case 3.1

3.1.1 Clinical Presentation Three-month-old girl with cyanosis since second week of birth and episodic worsening of cyanosis for 1 month, usually after feeding (XRAY—Fig. 3.1). In cyanotic child, CXR with oligaemia and large aorta— consider differentials of right-to-left shunt lesion as follows: • • • •

TOF Pulmonary atresia with VSD DORV, VSD with severe pulmonary stenosis Tricuspid atresia with VSD and severe pulmonary stenosis • D-TGA, VSD with severe pulmonary stenosis • Single ventricle with severe pulmonary stenosis • AVSD with severe pulmonary stenosis

3.1.2 Echo VSD, with aortic override, severe infundibular stenosis, and RVH with AR.

3.1.3 What Investigation Will You Do Next? 3.1.3.1 Goals of Imaging in CHD Catheter angiography can provide anatomic information to a large extent; however, it is invasive and no longer necessary to obtain the desired anatomic information. Hence, it is reserved for patients requiring intervention for PA growth such as stenting of patent ductus arteriosus or RVOT, and in occasional patients requiring preoperative closure of signifiS. N. Raju (*) Department of Cardio Vascular Radiology and Endovascular Interventions, AIIMS, Ansari Nagar, New Delhi, India

Fig. 3.1  “Boot-shaped” heart (upturned apex) due to right ventricular hypertrophy, concave pulmonary arterial segment (pulmonary bay), pulmonary oligaemia

cant aortopulmonary collaterals. Orthogonal imaging provides comprehensive evaluation of the disease, including following morphologic and functional evaluation in a reproducible and operator-independent manner.

3.1.4 Morphologic Features  • Ventricular (right and left) morphology including the wall thickness, size, and systolic function (contraction, volume), and size and location of VSD(s)  •  Morphology and size of both atria  • Anatomy, size, and degree of obstruction(s) of the RVOT, pulmonary valve, main and branch pulmonary arteries  • Assessment of additional sources of pulmonary blood flow (MAPCAs/PDA)  •  Morphology of the overriding aorta (size and degree)  •  Origins of coronary arteries and their relation to RVOT  •  Anatomy of the cardiac valves  •  Aortic arch sidedness  • Presence of coexisting anomalies including ASD, PDA, and persistent LSVC

© The Author(s), under exclusive license to Springer Nature Singapore Pte Ltd. 2023 S. Sharma (ed.), Case-based Atlas of Cardiac Imaging, https://doi.org/10.1007/978-981-99-5620-3_3

47

S. N. Raju

48

3.1.5 Functional Parameters  • Measure end-diastolic and end-systolic ventricular volumes, stroke volumes, and ejection fraction of both ventricles.  • Ventricular mass calculated by subtracting endocardial from epicardial volume for the RV and the LV.  •  Wall motion abnormality in RV and RVOT.  •  Quantify pulmonary or other valve regurgitation.  •  Qp/Qs ratio.

In view of the above, we use orthogonal imaging techniques to derive the relevant information by Gated CT angiography and cardiac MR. Although both CT and cardiac MR share advantages as being multi-planar, outpatient-based and gated acquisition each have their advantages and limitation. Advantages

Gated CTA Allows for rapid acquisition of scan with superior temporal and spatial resolution

Cardiac MR Allows for shunt quantitation, gradient measurement, flow, and regurgitant volume quantitation which is not possible by CTA

Limitations

Gated CTA Associated with radiation exposure and risk of allergy to iodinated contrast media

Cardiac MR Disadvantage of MRA include long breath holds (hence impracticable in young children), longer acquisition times with contraindication in claustrophobia, and MR-incompatible pacemakers

3.1.6 What Are the Imaging Findings? See Fig. 3.2.

3.1.7 Final Diagnosis Classical Tetralogy of Fallot with infundibular stenosis, confluent pulmonary arteries.

3  Imaging in Tetralogy of Fallot

49

a

b

c

d

e

Fig. 3.2  Cardiac CT findings: (a) (Coronal), (b) (sagittal): infundibular hypertrophy and stenosis (yellow stars) with confluent pulmonary arteries. (c) Axial images showing confluent good sized pulmonary arteries. (d) Subaortic ventricular septal defect (VSD) with aortic over-

f

ride 50% overriding (yel-

low arrow), (c) subaortic conus (yellow curved arrow) with aortomitral discontinuity, (d) aorta and main PA from right ventricle, (e) dilated aorta and small, confluent pulmonary arteries, (f) infundibular PS with confluent pulmonary arteries

54

3.5 Case 3.5 Five-year-old boy with exertional dyspnoea and cyanosis with echocardiographic finding of membranous VSD and pulmonary stenosis, what are the imaging findings (Fig. 3.6)?

3.5.1 Final Diagnosis Single ventricle PS with VSD and anomalous left circumflex artery.

S. N. Raju

3.5.2 Differentiating Features from Classical TOF Functionally single ventricle of right ventricular morphology (presence of moderator band and septal trabeculation) has a rudimentary LV with abnormal atrioventricular connection. Seen in the setting of mitral atresia, hypoplastic left heart syndrome, and double inlet right ventricle which can be identified by orthogonal imaging.

3  Imaging in Tetralogy of Fallot

55

a

b

c

d

e

f

Fig. 3.6 (a) CXR—cardiomegaly with RV configuration apex and pulmonary oligaemia and enlarged RA. CT (b) morphology RV showing coarse trabeculation (straight arrow) with VSD (curved arrow) (yellow arrow), (c) short axis view showing dilated morphologic right ventricle with hypoplastic LV, (d) subaortic conus (yellow curved arrow) with

aortomitral discontinuity (e) aorta and main PA from morphologic right ventricle and valvular PS (f) anomalous left circumflex artery from posterior PA facing sinus (arrow head). Normal left anterior descending artery from anterior PA facing sinus

56

3.6 Case 3.6 Five-year-old boy with dyspnoea and cyanosis with echocardiographic finding of membranous VSD and pulmonary stenosis, what are the imaging findings (Fig. 3.7)?

3.6.1 Final Diagnosis VSD with pulmonary atresia.

S. N. Raju

3.6.2 Differentiating Features from Classical TOF Non-rotation of the aortic root sinuses (line along the interatrial septum bisects the non-coronary sinus) with lack of continuity between RVOT and PA s/o atresia of PA.

3  Imaging in Tetralogy of Fallot

57

b

a

c

d

e

Fig. 3.7 (a) Cardiomegaly with RV configuration apex and pulmonary oligaemia. CT (b) subaortic VSD with absent overriding (yellow arrow), (c) short axis view through aortic sinus showing normal orientation of the posterior facing non-coronary sinus with respect to the interatrial septum (yellow dotted line), (d) confluent PA, long segment

pulmonary atresia, and small RPA and LPA (seagull PAs) (yellow arrow head and curved arrow respectively), (e) VRT image showing confluent PA, MPA atresia and small RPA and LPA (yellow arrow head and curved arrow respectively)

58

3.7 Case 3.7 Four-years-old boy with dyspnoea and cyanosis with echocardiographic finding of small right ventricle, what are the imaging findings (Fig. 3.8)?

3.7.1 Final Diagnosis Tricuspid atresia with VSD with PS.

S. N. Raju

3.7.2 Differentiating Features from Classical TOF Absent communication with RA and right ventricle resulting in a small right ventricle. Obligatory ASD.

3  Imaging in Tetralogy of Fallot

a

c

59

b

d

e

Fig. 3.8 (a) CXR—straightened right heart border with RV configuration apex (BUT IT IS LV apex) and pulmonary oligaemia and enlarged RA. CT (b) tricuspid atresia (yellow arrow), with absent communication between RA and right ventricle, a small right ventricle with large

ASD (curved arrow), (c) oblique sagittal view showing restrictive VSD (yellow arrow head), (d) Infundibular PS yellow arrow), (e) confluent pulmonary arteries

60

3.8 Case 3.8 Seven-years-old boy with dyspnoea with echocardiographic findings suggestive of TOF, what are the imaging findings (Fig. 3.9)?

3.8.1 Final Diagnosis Double-chambered right ventricle.

a

S. N. Raju

3.8.2 Differentiating Features from Classical TOF DCRV is characterized by hypertrophied muscle bands incorrectly positioned too high. The obstruction is only located in the proximal portion of the infundibulum, whereas in TOF the entire infundibular region is affected. The right ventricular cavity is divided into two parts: the high-pressure chamber proximal to the muscle bands and the low-pressure distal chamber.

b

Fig. 3.9  CT (a) confluent PAs, with normal infundibulum (yellow curved arrow). (b) Hypertrophied abnormal muscle bundle proximal to the RVOT/infundibulum with spotty calcification and hypertrophied wall of right ventricle

3  Imaging in Tetralogy of Fallot

61

3.9 Tetralogy of Fallot

3.9.4 Associated Anatomical Abnormalities

3.9.1 Introduction

• TOF is associated with coronary artery abnormalities in 5% of patient’s population. The most common being an anomalous origin of right coronary artery (RCA) from the left anterior descending (LAD) artery traversing anterior to right ventricular infundibulum. The next common coronary anomaly is a prominent conal branch of the RCA also traversing anterior to right ventricular infundibulum. It is very pertinent to identify these coronary anomalies as it changes the surgical management, such as introduction of a conduit between the RV and pulmonary arteries (RV–PA conduit) rather than RV infundibulectomy because of the possible associated myocardial ischemic damage due to injury to crossing coronary. • Up to 25% of cases of TOF can show right aortic arch with mirror images branching. • ASD and persistent left SVC may be seen in 5% and 11% of cases, respectively.

Tetralogy of Fallot (TOF) is commonest of all the cyanotic congenital heart diseases (CHD) and constitutes 5–7% of CHD spectrum with an incidence of 0.5/1000 live births. It is characterized by four specific congenital defects: (1) right ventricular outflow tract (RVOT) obstruction, (2) malaligned (perimembranous) VSD (VSD), (3) biventricular origin of the aorta (overriding aorta), and (4) right ventricular hypertrophy [1–3].

3.9.2 Embryology and Pathoanatomy of TOF • TOF is a developmental anomaly of conotruncal septum, where there is malalignment (antero-superior deviation) and underdevelopment of the infundibular septum leading to narrowing and obstruction of the RVOT.  As a result, right ventricular hypertrophy ensues, in addition to the presence of perimembranous VSD and the aortic root overriding the right and the LVs. • The RVOT obstruction is often typically subvalvular at pulmonic infundibulum. However, it can be valvular/subvalvular. The valvular obstruction can be either in the form of hypoplastic valve ring, thickening, and fusion of the valve leaflets leading to stenosis or atresia. • The main PA may be showing varying degrees of stenosis, hypoplastic, or atretic. In severe cases, pulmonary flow is maintained by a PDA when the main PA and its branches are intact and/or by MAPCAs when the pulmonary arteries are absent or very hypoplastic. MAPCAs usually arise from the descending thoracic aorta but can also have their origins from subclavian, innominate, vertebral, thyrocervical tank, internal mammary artery, coronary arteries, and the abdominal aorta and anastomose with hilar pulmonary arteries.

3.9.3 Genetic Abnormalities • 25% of patients with TOF show genetic abnormalities either in the form of trisomy 21 or a 22q11 deletion with the latter being more common. Patients with a 22q11 deletion have associated a right aortic arch with an absent thymus. In trisomy 21, coexistence of TOF with common atrio-ventricular channel can be seen.

3.9.5 Imaging in Tetralogy of Fallot (a) Chest radiographs: Classic signs as described below: 1. “Boot-shaped” heart with an upturned cardiac apex due to right ventricular hypertrophy 2.  Concave pulmonary arterial segment 3.  Pulmonary oligaemia 4.  Aortic enlargement 5.  Right-sided aortic arch in 25% of cases

(b) Echo: It is the first-line investigation to diagnose and monitor CHD, particularly in paediatric practice where the acoustic window is good. Its role, however, is limited in assessing extra-cardiac vessels. (c) Computed tomography: In preoperative imaging of TOF, CT [4] can demonstrate all the following morphologic findings needed to plan patient management as given in Appendix B. (d) Cardiac MRI: Cardiac MRI [5] evaluates all the morphologic features and volumetric and flow analysis features described in Appendix B. (e) Catheter angiography: It is a minimally invasive technique. Although not primarily used in diagnosis, catheter angiography still plays a significant role in the management of TOF as described below.

62

S. N. Raju  •  Assessment of routability of VSD  •  Preoperative embolization of MAPCAs  • Assessment of pulmonary arteries after palliative procedure—Modified BT shunt  • In patients with complex pulmonary atresia for the detailed characterization of lung segmental pulmonary vascular supply, especially when non-invasive imaging methods incompletely define PA anatomy Catheter angiogram techniques to demonstrate confluence of PAs:     1. Aortic angiogram in presence of PDA to identify filling of Pas     2. Selective angiogram of the MAPCA with long acquisition in delayed angiographic runs identify retrograde filling of Pas     3. Pulmonary venous wedge angiogram: Retrograde filling of the PAs after selective angiograms of the pulmonary vein with proximal occlusion  • In determination of coronary circulation in pulmonary atresia with intact septum.

3. TOF with large aorto-pulmonary collaterals 4. TOF with absent pulmonary valve 5. TOF with spectrum of branch PA atresia, hypoplasia, and stenosis 6. TOF with unilateral RPA/LPA PA atresia 7. TOF with crossed pulmonary arteries 8. TOF with isolated left subclavian artery 9. TOF with anomalous coronaries 10. TOF with ASD ➔ Pentalogy of Fallot

3.10 Case 3.9 See Fig. 3.10.

3.10.1 Final Diagnosis TOF with valvular pulmonary stenosis.

3.9.6 Spectrum of Imaging Abnormalities in TOF 1. TOF with valvular pulmonary stenosis 2. TOF with pulmonary atresia (Type I, II, III, IV)

3  Imaging in Tetralogy of Fallot

63

a

b

d

Fig. 3.10  CXR (a) cardiomegaly with RV configuration apex and pulmonary oligaemia. CT (b) subaortic VSD, (c) thickening and doming of pulmonary vale leaflets, (d) confluents PAs with dilated left PA, (e)

c

e

VRT image showing confluent PAs with dilated left PA, pulmonary annular hypoplasia

64

S. N. Raju

3.11 Case 3.10

3.11.1 Final Diagnosis

See Fig. 3.11.

TOF with pulmonary atresia (Type I).

a

c

Fig. 3.11  CXR (a) cardiomegaly with RV configuration apex, pulmonary bay, and pulmonary oligaemia. CT (b) subaortic VSD (yellow arrow), (c) confluents PAs with discontinuity between the RVOT

b

d

(curved yellow arrow) and the main pulmonary artery. (d) VRT image showing confluents PAs with discontinuity between the RVOT (curved yellow arrow) and the main pulmonary artery

3  Imaging in Tetralogy of Fallot

65

3.12 Case 3.11

3.12.1 Final Diagnosis

See Fig. 3.12.

TOF with pulmonary atresia (Type II).

a

b

c

e

d

f

Fig. 3.12  CXR (a) cardiomegaly with RV configuration apex, pulmonary bay, and pulmonary oligaemia. CT (b) subaortic VSD, (c) confluents PAs with discontinuity with the RVOT and absent main PA

g

segment. (d, e) Large aortopulmonary collateral from the medial wall of the descending thoracic aorta. (f, g) Hilar left PA below the level of the left main bronchus

66

S. N. Raju

3.13 Case 3.12

3.13.1 Final Diagnosis

See Fig. 3.13.

TOF with pulmonary atresia (Type IV). a

b

d

c

e

Fig. 3.13  CXR (a) cardiomegaly with RV configuration apex, pulmo- segment and reformed hilar pulmonary arteries. (d, e) Large nary bay and pulmonary oligaemia. CT (b) subaortic VSD, (c) non-­ Aortopulmonary collateral from the descending thoracic aorta confluents PAs with discontinuity with the RVOT and absent main PA

3  Imaging in Tetralogy of Fallot

67

3.14 Case 3.13

3.14.1 Final Diagnosis

See Fig. 3.14.

TOF with hypoplastic RPA.

a

b

d

c

e

Fig. 3.14  CXR (a) cardiomegaly with RV configuration apex, dilated left PA and pulmonary oligaemia, right sided aortic arch. CT (b) subaortic VSD, (c) confluents PAs with hypoplastic RPA from the ostia up to the PDA insertion. (d) Dilated RPA distal PDA insertion site. (e)

VRT image showing confluents PAs with hypoplastic RPA from the ostia up to the PDA insertion and post-stenotic dilatation of RPA distal PDA insertion site

68

S. N. Raju

3.15 Case 3.14

3.15.1 Final Diagnosis

See Fig. 3.15.

TOF with RPA atresia.

a

b

c

d

e

Fig. 3.15  CXR (a) opaque right haemothorax with diaphragmatic hernia and hypoplastic right lung and upturned right main bronchus. CT (b) subaortic VSD, (c) infundibular PS, (d) VRT image showing MPA

and LPA with absent RPA, (e) lung window in coronal projection showing upturned right main bronchus with hypoplastic right lung with right diaphragmatic hernia

3  Imaging in Tetralogy of Fallot

69

3.16 Case 3.15

3.16.1 Final Diagnosis

See Fig. 3.16.

TOF with crossed PA.

a

b

c

d

e

Fig. 3.16  CXR (a) cardiomegaly with RV configuration apex, pulmonary oligaemia. CT (b) subaortic VSD, (c) infundibular PS, (d) confluent PAs with crossing of PAs. (e) Aberrant left subclavian artery from

f

distal aortic, coursing posterior to the trachea. (f) Lung window sagittal plane showing posterior tracheal wall indentation

70

S. N. Raju

3.17 TOF: Spectrum of Branch PA Involvement 3.17.1 Case 3.16: LPA Atresia

3.17.2 Case 3.17: LPA Ostial Stenosis See Fig. 3.18.

See Fig. 3.17. a

b

Fig. 3.17  CT (a) axial CT showing MPA and RPA with absent LPA, (b) VRT images showing MPA and RPA with absent LPA

a

b

Fig. 3.18  CT (a) axial CT showing confluent PAs with ostial stenosis of the LPA and post-stenotic dilatation, (b) VRT images showing confluent PAs with ostial stenosis of the LPA and post stenotic dilatation

3  Imaging in Tetralogy of Fallot

71

3.17.3 Case 3.18: Diffuse LPA Stenosis Proximal to PDA Insertion Site with Post-stenotic Dilatation

3.17.4 Case 3.19

See Fig. 3.19.

3.17.4.1 Final Diagnosis TOF with absent pulmonary valve

a

b

See Fig. 3.20.

c

Fig. 3.19  CT (a) axial CT showing confluent PAs with diffuse LPA stenosis proximal to PDA insertion site with post-stenotic dilatation. Coronal and sagittal CT in (b) and (c) respectively showing PDA (yellow star) connecting to LPA (yellow arrow)

72

S. N. Raju

a

b

c

d

e

f

Fig. 3.20  CXR (a) Cardiomegaly with enlarged pulmonary arteries and widening of vascular pedicle width and pulmonary oligaemia. CT (b) subaortic VSD, (c) absent pulmonary valve with annular hypoplasia and confluent dilated MPA, RPA, and LPA. (d) Rudimentary ridge of

pulmonary valve leaflet. (e) VRT images confluent dilated MPA, LPA with annular hypoplasia. (f) Lung window in coronal projection showing downward displacement of the left main bronchus

3  Imaging in Tetralogy of Fallot

73

3.17.5 Case 3.20

3.17.5.1 Final Diagnosis TOF with LPA atresia and absent pulmonary valve.

See Fig. 3.21. a

b

d

Fig. 3.21  CXR (a) cardiomegaly, RV configuration apex with enlarged right PA and pulmonary oligaemia. CT (b) subaortic VSD, (c) absent pulmonary valve with annular hypoplasia and dilated MPA, RPA and

c

e

absent LPA. (d) VRT images annular hypoplasia and dilated MPA, RPA and absent LPA. (e) Lung window in sagittal plane showing anterior wall indentation of the right main bronchus by the dilated RPA

74

S. N. Raju

3.17.6 Case 3.21

3.17.6.1 Final Diagnosis TOF with anomalous origin of right pulmonary from ascending aorta.

See Fig. 3.22.

a

b

c

d

Fig. 3.22  CXR (a) cardiomegaly, RV configuration apex with enlarged right PA and pulmonary oligaemia and pulmonary bay. CT (b) subaortic VSD, (c) dilated ascending aorta with dilated RPA from the posterior

e

wall of the ascending aorta, hypoplastic LPA from the RVOT. (d, e) VRT images dilated ascending aorta with dilated RPA from the posterior wall of the ascending aorta hypoplastic LPA from the RVOT

3  Imaging in Tetralogy of Fallot

3.17.7 Case 3.22

75

3.17.7.1 Final Diagnosis TOF with isolated left subclavian artery.

See Fig. 3.23. a

b

c

d

e

Fig. 3.23  CXR (a) cardiomegaly, RV configuration apex and pulmonary oligaemia, right-sided aortic arch. CT (b) subaortic VSD, (c) confluent pulmonary arteries, (d) sagittal oblique view showing aortic arch with origin of the innominate (yellow arrow) and left carotid artery

(yellow curved arrow) and absent left subclavian artery. (e) Stump of the left SCA (yellow curved arrow) reformed from the left vertebral artery (yellow star) s/o isolation of left SCA

76

3.17.8 Case 3.23: Pentalogy of Fallot (TOF with ASD)

S. N. Raju

3.17.8.1 Final Diagnosis Pentalogy of Fallot (TOF with ASD).

See Fig. 3.24.

a

b

c

d

Fig. 3.24  CXR (a) cardiomegaly, RV configuration apex and pulmonary oligaemia. CT (b) subaortic VSD, (c) thickened pulmonary valve leaflets with fish-mouth appearance, (d) ASDs

3  Imaging in Tetralogy of Fallot

3.18 TOF with Coronary Anomalies 3.18.1 Case 3.24: Hypertrophied Conal Artery Crossing the Infundibulum

77

3.18.2 Case 3.25: LAD—Common Origin with RCA from Right Coronary Sinus See Fig. 3.26.

See Fig. 3.25.

a

b

c

d

Fig. 3.25  CT (a) subaortic VSD, (b) infundibular PS with confluent PAs, (c) conal artery direct origin front the anterior PA facing sinus, (d) conal artery traversing anterior to the hypertrophied muscular infundibulum

78

S. N. Raju

a

b

c

d

Fig. 3.26  CT (a) subaortic VSD, (b) common origin of the Left anterior descending artery and right coronary artery from anterior PA facing sinus, (c) left anterior descending artery traversing anterior the hyper-

trophied infundibulum and RVOT, (d) VRT image showing left anterior descending artery traversing anterior to RVOT in the anterior interventricular groove

3  Imaging in Tetralogy of Fallot

79

3.18.3 Case 3.26: LM—Common Origin with RCA from Right Coronary Sinus See Fig. 3.27.

a

b

c

d

Fig. 3.27  CT (a) subaortic VSD, (b) common origin of the Left main artery and right coronary artery from anterior PA facing sinus, (c) ft. main coronary traversing anterior the hypertrophied infundibulum and

RVOT, (d) VRT image showing Left main coronary artery traversing anterior to RVOT in the anterior interventricular groove

80

3.19 TOF with Aortopulmonary Collaterals In response to reduced O2 saturation, numerous systemic to pulmonary collaterals develop, from descending thoracic aorta, intercosto-bronchial trunk, bronchial arteries, ­subclavian arteries and uncommonly from vertebral, lateral thoracic, celiac, renal arteries, inferior phrenic artery, and abdominal aorta. Prior to intracardiac corrective surgical repair of TOF, only significant systemic APCs should be preoperatively embolized to reduce the intra-operative blood loss and postoperative endotracheal bleed, haemoptysis, difficult extubation, and pulmonary oedema. Emergency embolization of significant APCs is also indicated when the patients present with significant haemoptysis.

3.19.1 Criteria for Significant APC

S. N. Raju

3. PA filling 4. Pulmonary arteries venous shunting 5. Aneurysm/pseudoaneurysm of APC 6. Active contrast extravasation

3.19.2 Prerequisites for Embolization of APC 1. Good sized native pulmonary arteries (Nakata Index >1.2). 2. Confirm dual supply of the pulmonary segments by the native pulmonary arteries and the APC.

3.19.3 Case 3.27: Significant Systemic to PA Collaterals See Fig. 3.28.

1. Hypertrophied (2–3 mm) and tortuous 2. Pulmonary parenchymal blush

3  Imaging in Tetralogy of Fallot

a

81

b

d

f

Fig. 3.28  Catheter angiographic demonstration of significant systemic to PA collaterals form different locations: (a) descending thoracic, (b) common bronchial trunk, (c) right internal mammary artery (d) right bronchial artery, (e) left intermammary artery and vertebral artery, (f) left intercostal artery with early filling of the pulmonary veins and

c

e

g

parenchymal blush suggestive of microfistulae, (g) right lateral thoracic artery (h, i) from left intercostal artery showing parenchymal blush, (j, k) from innominate artery showing early filling of the hilar pulmonary arteries and pulmonary parenchymal blush, (l, m) from the juxta renal abdominal aorta

82

S. N. Raju

h

i

j

k

l

m

Fig. 3.28 (continued)

3  Imaging in Tetralogy of Fallot

3.20 How Imaging Impacts Management?

83

Long segment atresia—BT shunt ➔ RV to PA conduit at 2–3 years (size issue) The natural history of the condition and the “timing” of surIf both PAs are small—RVOT stenting or central shunt gery depends on the degree of obstruction of the RVOT and (morbid) 3 adequacy of pulmonary arteries. It is done as single-stage PV Balloon angioplasty: If very sick child (surgery total repair or sequential repair (palliative followed by cor- expertise not available)—as a bailout ➔corrective repair if rective total repair).1 good PAs, If PAs not good ➔ BTS 4 *How to decide adequacy of PAs Total correction for anomalous coronary crossing RVOT: Total correction with RV–PV conduit in age > 2 years 1. McGoon ratio (Fig.  3.29): (Diameter of RPA  +  LPA)/ and wt > 10 kg (Fig. 3.31) 1 Descending thoracic aorta at diaphragm Infundibulectomy: It consists of resection of the hyper• Normal 2.1 trophic myocardium, usually through a longitudinal incision • Adequate for VSD closure ≥1.5 in the RA and proximal PA.  The pulmonary valve is sub• Inadequate for VSD closure 200 mm /m polytetrafluoroethylene (Gore-Tex; Gore, Newark, Del) • Adequate for total repair >150 mm2/m2 patch is then sutured to both margins of the incision, increas(Not usable pre-operatively when major aorto-­ ing the diameter of the outflow region and the main PA. The pulmonary collaterals are the major source of pulmonary patch can be extended to the branch pulmonary arteries if blood flow and one-stage unifocalization + full repair is concomitant stenosis of these vessels is present. 3 planned) (Fig. 3.30) RV–PA conduit placement: A synthetic or biologic 1 Shunts: Mostly modified BT shunt—more reliable (usually allograft or xenograft) conduit is inserted in the source of blood flow (McGoons anterior). –– Dilatation of the atrialized portion of the right ventricle –– Redundant, tethered, or fenestrated anterior leaflet • Apically displaced septal or the posterior leaflet often forms a sail-like curtain in the right ventricular outflow causing obstruction. • As a result, the right heart consists of RA, atrialized right ventricle, functional right ventricle, and right ventricular outflow. The tricuspid annulus dilates progressively. • The atrialized portion of the ventricle often becomes disproportionately dilated and may amount to more than half of right ventricular volume. Eventually dilated right heart compresses the left heart with paradoxical motion of the interventricular septum [4].

Table 9.4  Carpentier classification of Ebstein’s anomaly Type A Type B Type C Type D

Adequate volume of functional right ventricle Free movement of anterior leaflet of tricuspid valve with large atrialized RV Restricted anterior leaflet with obstruction of right ventricular outflow Almost complete atrialization of RV

Table 9.5 Extended Glasgow outcome scale/great ormond street (GOSE) score for prognostication of Ebstein’s anomaly Grade 1 2 3 4

Ratio < 0.5 0.5 to 0.99 1 to 1.49 >1.5

Table 9.5. It is used for prognostication of patients with Ebstein’s anomaly, mainly in the neonatal age group. A ratio > 1.5 (grade 4) has the worst prognosis with nearly 100% mortality. A ratio of 1–1.5 (grade 3) has better prognosis with an early mortality of 10%, reaching up to 45% in childhood. A ratio of 90%. Celermajer extended Glasgow outcome scale [(RA  +  aRV)/ (fRV + LA + LV)]

9.9.4 Etiology • It is a multifactorial disease with environmental and genetic risk factors. • Maternal consumption of benzodiazepines and lithium (rare) is seen to be associated with Ebstein’s anomaly. • Most of the cases are sporadic; however, it is also common in twins and in patients with family history of congenital heart disease.

9.9.3 Classification

9.9.5 Associated Anatomical Abnormalities

• Carpentier et al. [5] has proposed a classification system based on the volume of atrialized right ventricle and the mobility of anterior leaflet of tricuspid valve as given in Table 9.4. • Celermajer et  al. [6] have given an Extended Glassgow Outcome Scale/Great Ormond Street (GOSE) Score on the basis of ratio of combined area of RA and atrialized right ventricle and combined area of functional right ventricle and left heart. It is divided in grade 1 to 4 as given in

• ASD is the most common associated cardiac abnormality in Ebstein’s anomaly, described in 80–94% of cases. • Other associated anomalies described are VSDs, pulmonary stenosis, bicuspid aortic valve, coarctation of aorta, or hypoplastic pulmonary arteries. • Patients of congenitally corrected transposition of great arteries (ccTGA) often have ebstentoid deformity of left atrio-ventricular valve, reported in 15–50% of cases.

A. A. Deshpande

158

9.9.6 Left Ventricular Involvement • Left ventricular involvement is fairly common in patients of Ebstein’s anomaly. • Attenhofer et al. [7]. have reported an incidence of 39% in one series with the most common abnormality detected was left ventricular non-compaction. • Other structural abnormalities include mitral valve prolapse, mitral valve dysplasia, and bicuspid aortic valve. • Extreme dilatation of the atrialized RV causes compression of left heart. This along with structural abnormalities lead to left ventricular dysfunction. • Left ventricular dysfunction is an important risk factor for the development of major adverse cardiac events along with right ventricular dysfunction [8].

Table 9.6  Classical CXR signs of Ebstein’s anomaly  1.  Cardiomegaly with globular heart  2.  Narrow pedicle  3.  Right atrial enlargement (elongation of right heart border, >50% of mediastinal cardiovascular shadow)  4.  Normal or decreased pulmonary vascularity Table 9.7  Role of CMR in evaluation of Ebstein’s anomaly Diagnosis

Treatment planning (Carpentier classification)

Prognostication (goose score)

9.9.7 Imaging 1. Goals of Imaging (a) Diagnosis (b) Treatment planning (c) Follow-up (d) Detection of complications 2. Various Imaging modalities (a) Chest radiographs: Classic signs as described in Table 9.6. (b) Echo: It is the first-line investigation to diagnose and monitor CHD, particularly in pediatric practice where the acoustic window is good. It can evaluate the tricuspid valve leaflets, degree of apical displacement, and tricuspid regurgitation. The volume of atrialized right ventricle and functional right ventricle can be measured as well. (c) Computed tomography: CT is not indicated in the pre-operative evaluation of Ebstein’s anomaly. However, it can be done in patients with sub-optimal Echo and if MRI is cannot be performed. CT can demonstrate the morphologic findings needed to plan patient management as discussed previously; however, the functional imaging may not be accurate. (d) Cardiac MRI: Cardiac MRI is the investigation of choice in patients with sub-optimal Echo. It evaluates volumetric and flow analysis features in addition to morphologic features (Table  9.7). CMR is recommended as a class II indication to determine RV functions and anatomy [9].

Follow-up (post-operative)

Accurate assessment of apically displaced tricuspid valve Functional and volumetric assessment of aRV, fRV and LV Status of anterior leaflet The status of anterior leaflet (mobile/redundant/fenestrated) Volume and function of fRV Volume of fRA Volume of RA, aRV, fRV, LA, and LV Status of tricuspid valve (TR) Functional analysis of fRV

(e) Catheter angiography: It is not routinely required for pre-operative evaluation anymore. However, it can be performed in the absence of cross-sectional imaging for diagnosis and also to measure right atrial and ventricular pressure in select cases. (f) Electrophysiological Study: The patients of Ebstein’s anomaly presenting in adulthood frequently come with arrhythmias. Electrophysiological study can be done to identify the accessory pathways before the surgery or during the surgery. It has been given a class IIa recommendation in AHA 2018 guidelines [9]

9.9.8 Variations of Ebstein’s Anomaly 1. Ebstein’s with good-sized fRV and normal anterior leaflet 2. Ebstein’s anomaly with muscle band in aRV 3. Ebstein’s with near complete obliteration of fRV and redundant anterior leaflet 4. Ebstein’s with deranged LV function with LV clot 5. ccTGA with Ebstentoid anomaly of left AV valve 6. Ebstein’s anomaly with pulmonary atresia 7. Ebstein’s with ASD

9  Imaging in Ebstein’s Anomaly

159

9.10 Case 9.9

9.10.1 Final Diagnosis

Forty-one-year-old man, with gradually progressing exercise intolerance and palpitations for last 2  years. ECG showed tachyarrhythmias and prolonged PR interval. Echo revealed dilated RA and RV with apical displacement of tricuspid valve leaflets and TR (Fig. 9.10).

Ebstein’s anomaly with good-sized fRV and normal anterior leaflet.

D

E

F

G

H

I

Fig. 9.10  Chest X ray (a) borderline cardiomegaly with mildly dilated RA. (b, c) Four chamber systole and diastole images show mildly dilated RA with apically displaced septal leaflet of tricuspid valve (red arrow) with respect to mitral leaflets (black arrow), normal anterior

leaflet of tricuspid valve (green arrow), and tricsupid regurgitation (*). T2 short axis (d), four chamber (e), and T1 short axis (f) images do not show abnormal myocardial signal intensity

LV Functions Absolute Indexed EF 60.55 (56–78%) – EDV 86.65 (77-­195 mL) 47.27 (47–92 ml/m2) Atrialized RV end-diastolic volume: Absolute: 19.55 mL; normalized 10.67 mL/m2

fRV Absolute 41.65 (47–74%) 228.07 (88–227 mL)

Indexed – 124.42 (55–105 mL/m2)

A. A. Deshpande

160

9.11 Case 9.10

9.11.1 What Are the Imaging Findings?

Thirteen-year-old boy presented with gradually progressing dyspnea of exertion for last 3 years. No h/o palpitations cyanosis/edema/pallor. Echo revealed dilated RA and RV with apical displacement of tricuspid valve leaflets.

See Fig. 9.11.

9.11.2 Final Diagnosis Ebstein’s anomaly with good-sized fRV, elongated anterior leaflet, and muscle bundle in aRV.

D

E

F

G

H

I

J

K

Fig. 9.11  Chest X ray (a) cardiomegaly with globular heart. RA appears enlarged with narrow pedicle. Lung vascularity appears mildly reduced. (b, c) Four chamber diastole and systole images show a muscle bundle in aRV. (d) Short axis T1 map shows normal native myocardial T1 values. (e, f) Short axis systole and diastole images show

apically displaced septal leaflet (red arrow) and elongated anterior leaflet (green arrow). The muscle bundle is evident in aRV. (g, h) Four chamber and short axis late gadolinium enhanced images do not show abnormal enhancement

LV fRV Functions Absolute Indexed EF 54.39 (56–78%) – EDV 107.05 (77–195 mL) 63.93 (47–92 mL/m2) Atrialized RV end-diastolic volume: Absolute: 45.06 mL; indexed: 26.91 mL/m2

Absolute 35.86 (47–74%) 233.92 (88–227 mL)

Indexed – 139.7 (55–105 mL/m2)

9  Imaging in Ebstein’s Anomaly

161

9.12 Case 9.11

9.12.1 What Are the Imaging Findings?

Three-year-old girl presented with progressive cyanosis since birth and failure to thrive. No e/o edema/pallor. Echo revealed dilated RA and RV with apical displacement of tricuspid valve leaflets.

See Fig. 9.12.

9.12.2 Final Diagnosis Ebstein’s anomaly with ASD, redundant anterior leaflet, and near complete obliteration of fRV.

D

E

F

G

H

I

J

K

Fig. 9.12  Chest X ray (a) cardiomegaly with globular heart. RA appears enlarged with narrow pedicle. Lung vascularity appears mildly reduced. (b, c) Four chamber systole and diastole images dilated aRV, ASD and small fRV. (d) Apical section of four chamber image shows apically displaced septal leaflet of tricuspid valve (red arrow) and

redundant anterior leaflet (green arrow). (e, f) Four chamber T2 and T1 image do not show abnormal myocardial signal intensity. (g, h) Short axis diastole and systole images show dilated aRV and apically displaced septal leaflet

LV RV Functions Absolute Indexed EF 46 (56–78%) – EDV 51.06 (52–141 mL) 102.4 (41–81 mL/m2) Atrialized RV end-diastolic volume: Absolute: 180.05 mL; indexed: 95.33 mL/m2

Absolute 34.03 (47–74%) 29.78 (58–154 mL)

Indexed – 59.73 (48–87 mL/m2)

162

A. A. Deshpande

9.13 Case 9.12

with apical displacement of tricuspid valve leaflets, LV systolic dysfunct, and TR (Fig. 9.13).

A 34-year-old man with the progressive dyspnea on exertion for last 3 years, palpitations and bilateral lower limb edema. No h/o cyanosis. ECG revealed tall and broad P waves and fragmented QRS complex. Echo revealed dilated RA and RV

9.13.1 Final Diagnosis Ebstein’s anomaly with deranged LV function and LV clot.

D

E

H

I

F

G

J

Fig. 9.13  Chest X ray (a) cardiomegaly with globular heart. Right atrial enlargement, narrow pedicle and normal to reduced lung vascularity. (b–d) Four chamber systole and diastole images show apically displaced septal leaflet of tricuspid valve with dilated RA and aRV. ASD is noted (yellow arrow). Systole and diastole images show reduced LV

K

contractility. (e, f) Short axis diastole and systole images show dephasing jet of tricuspid regurgitation (*) and reduced LV contractility. (g, h) Four chamber and short axis late gadolinium enhanced image show enhancement along RV side of interventricular septum (arrowheads). Small clot is noted at LV apex (arrow)

LV Functions Absolute Indexed EF 25 (56–78%) – EDV 179.45 (77–195 mL) 132.64 (47–92 mL/m2) Atrialized RV end-diastolic volume: Absolute: 121.06 mL; indexed: 67.69 mL/m2

fRV Absolute 19.11 (47–74%) 243.57 (88–227 mL)

Indexed – 180.04 (55–105 mL/m2)

9  Imaging in Ebstein’s Anomaly

163

9.14 Case 9.13

9.14.2 What Are the Imaging Findings?

A 28-year-old woman with complaints of dyspnea on exertion for 2 years started during pregnancy.

See Fig. 9.14.

9.14.3 Final Diagnosis

9.14.1 Echo Dilated RA and RV with apical displacement of tricuspid valve leaflets.

ccTGA with ebstentoid anomaly of left AV valve and dilated LA.

D

E

F

G

H

I

J

K

L

M

N

O

Fig. 9.14  Chest X ray (a) LA enlargement (double right heart border), mild cardiomegaly. (b, c) Four chamber systole and diastole images A-V discordance. Moderator band (yellow arrow). Apical displacement of the MV leaflet (white arrow) with mild left AVVR (*). (d) Axial section shows malposed great vessels suggestive of V-A discordance. Short

Functions EF EDV

LV Absolute 54.23 (56–78%) 100.31 (52–141 mL)

axis (e, f) & four chamber (g, h) T1 and T2 images do not show abnormal myocardial intensity. (h) Sagittal section shows aorta anterior to main PA. (i, j) Short axis T1 and T2 maps do not show altered native T1 and T2 values. (k, l) Short axis and four chamber LGE images do not show abnormal enhancement fRV

Indexed – 78.69 (41–81 mL/m2)

Absolute 48.19 (47–74%) 126.47 (58–154 mL)

Indexed – 99.21 (48–87 mL/m2)

164

A. A. Deshpande

9.15 Case 9.14

9.15.2 Final Diagnosis

Thirty-year-old woman with DOE and palpitations for last 3 years.

Ebstein’s anomaly with massive RA and aRV. Osteoproximal narrowing of RPA and atretic/occluded LPA.

9.15.1 ECG Tall & broad P wave and RBBB. Echo-dilated RA and RV, apically displaced of tricuspid valve leaflets (Fig. 9.15).

D

E

F

G

H

I

J

K

L

M

Fig. 9.15  Chest X ray (a) massive cardiomegaly with enlarged RV and RA, narrow pedicle. (b, c) Four chamber systole and diastole images show apically displaced septal leaflet of tricuspid valve (red arrow) with dilated RA and aRV. Tricuspid regurgitation(*). (d) Trufi axial image shows massively dilated RA and aRV. (e, f) Two chamber diastole and systole images show dephasing jet of tricuspid regurgitation (*) with

apically displaced septal and posterior leaflets. (g, h) Short axis T1 and T2 images do not show abnormal myocardial signal intensity. (i) TWIST angiogram showing atretic LPA and ostial narrowing of RPA. (j) Short axis late gadolinium-enhanced image does not show abnormal enhancement

LV Functions Absolute Indexed EF 58.55 (56–78%) – EDV 32.53 (52–141 mL) 23.98 (41–81 mL/m2) Atrialized RV: Absolute: 135.18 mL; indexed: 99.66 mL/m2

fRV Absolute 34.03 (47–74%) 29.78 (58–154 mL)

Indexed – 59.73 (48–87 mL/m2)

165

9  Imaging in Ebstein’s Anomaly

9.16 Case 9.15

9.16.2 Final Diagnosis

Fifteen-year-old girl with the cyanosis and easy fatiguability for last 2 years. H/o of stroke 3 months back.

Ebstein’s anomaly, elongated anterior leaflet, near obliteration of fRV and ASD.

9.16.1 Echo Dilated RA and RV, apically displaced tricuspid valve leaflets, ASD and TR (Fig. 9.16). D

E

F

G

H

I

J

K

L

M

N

O

Fig. 9.16  Chest X ray (a) cardiomegaly with enlarged RA and RV, narrow pedicle. (b–d) Four chamber systole (b) and diastole (c) images dilated RA and aRV. The fRV appears small. The ASD is also evident (arrow in d). (e, f) Two chamber diastole and systole images show dephasing jet of TR. (g, h) Short axis diastole and systole images show

apically disaplced septal leaflet (red arrow), elongated anterior leaflet (green arrow) and dephasing jet of tricuspid regurgitation (*). (i, j) Short axis T1 and T2 images do not show abnormal myocardial signal intensity. (k, l) Four chamber and short axis LGE images show enhancement along the RV side of IVS

LV Functions Absolute Indexed EF 53.88 (56–78%) – EDV 111.08 (52–141 mL) 88.13 (41–81 mL/m2) Atrialized RV: Absolute: 169.71 mL; indexed: 134.65 mL/m2

fRV Absolute 36.26 (47–74%) 62.33 (58–154 mL)

Indexed – 49.68 (48–87 mL/m2)

166

9.17 How Imaging Impacts the Management

A. A. Deshpande

The optimal management plan depends on the age of presentation, symptoms, volume of functional RV, and the status of anterior leaflet of the tricuspid valve.

• The arrhythmia in Ebstein’s anomaly is mainly AV nodal re-entrant tachycardia or accessory pathways. It is frequently associated with atrial flutter or fibrillation. • The ablation of accessory pathway can be done with catheter ablation or surgical excision using electroanatomic mapping either during or after the surgery.

9.17.1 Neonates

9.17.5 Management of Index Case

The societal guidelines for medical treatment of Ebstein’s anomaly are given in Table 9.8. The surgical options of the Ebstein’s anomaly are given in Fig. 9.17.

9.17.2 Children and Adults

• The index case was 17-year-old girl with Ebstein’s anomaly with progressive dyspnea. • The CXR, Echo, and CMR confirmed the reduced RV function. • This patient should undergo a corrective surgery— Tricuspid valve repair with biventricular repair.

Societal Guidelines for surgical treatment of Ebstein’s Anomaly in adults are given in Table 9.9.

9.17.6 Controversies and Future Directions

9.17.3 Medical Management Optimum heart failure management in form of diuretics and antiarrhythmic drugs. Asymptomatic, or mild cardiomegaly with no right to left shunt and normal exercise tolerance are managed conservatively. In doubtful cases, surgery (Fig.  9.18) may be chosen if there is echo evidence of high probability of TV repair.

9.17.4 Adjuvant Procedures 9.17.4.1 Anti-arrhythmic Measures • Patients who present with the tachyarrhythmias undergoing tricuspid valve repair generally show resolution of symptoms. However, the anti-arrhythmic measures are important in resistant cases.

• Cardiac MR can be very useful in arrhythmia management during catheter ablation of the accessory pathway and to predict the risk of recurrence as well as sudden cardiac death in the patients (Table 9.10).

Table 9.8  Societal guidelines for medical treatment of Ebstein’s anomaly

Guidelines—medical treatment Neonates with significant cyanosis—IV prostaglandin infusion Neonates with congestive heart failure—IV diuretics Neonates with arrhythmias—anti-arrhythmic drugs Surgery if not stabilized with medical treatment

Level of evidence Indian guidelines [10] I I I IIa

9  Imaging in Ebstein’s Anomaly

167

Neonates

Choice of Repair Depends upon Volume and function of fRV Status of Anterior leaflet

Biventricular Repair

Univentricular Repair (RV Exclusion)

Tricuspid valve repair Reduction of aRV/RA Fenestrated closure of ASD* SOS Delayed Sternal Closure

Enlarging ASD Fenestrated Closure of Tricuspid Orifice# Systemic to Pulmonary Shunt RA/aRV Reduction

Transplant

BDG and Fontan Pathway

Fig. 9.17  Surgical options for Ebstein’s anomaly in Neonates. *ASD Closure: A small defect is left intentionally to facilitate the right to left flow through the ASD in view of anticipated elevated RV and PA pressure in setting of RV Dysfunction. #Closure of Tricuspid Orifice: A small defect is left intentionally in tricuspid valve to decompress the

right ventricle getting filled from thebesian vein. If Significant Pulmonary Regurgitation is present, either PA or Pulmonary valve needs closure. In some cases, complete RV is excluded by plication and non-fenestrated TV Closure

A. A. Deshpande

168 Table 9.9  Societal guidelines for surgical treatment of Ebstein’s anomaly in adults Guidelines—surgical management Surgery if one or more following symptoms are present:  •  Symptomatic heart failure  •  Worsening of exercise capacity  •  Worsening of RV systolic dysfunction Catheter ablation in adults with multiple accessory pathways Surgery if one or more following symptoms are present:  •  Progressive RV enlargement  •  Progressive cyanosis  •  Paradoxical embolism

Level of evidence AHA 2018 [9] I

ESC 2010 [11] I

Indian guidelines 2019 [10] I

I IIa

– IIaa

IIa I

ESC 2010 guidelines have recommended isolated catheter closure of ASD if the primary problem is paradoxical embolism or cyanosis.

a

Fig. 9.18  Surgical options for Ebstein’s anomaly in Children and Adults. * TV Repair: Most Commonly done as Cone’s Repair—Surgical delamination, mobilization, and annular reattachment. Annuloplasty is done to support the repair. Annular plication/leaflet augmentation/neo chordae formation may be added as deemed appropriate. # Sometimes the tricuspid valve is not repairable (muscularized anterior leaflet,

absent septal leaflet, massive annulus) and undergoes replacement with preferably a bioprosthetic valve. $ Bidirectional Cavo-Pulmonary Shunt: Suitable in Rv dysfunction and also in valves not having perfect repairs. Essential requirement: LV end-diastolic pressure 50% override and conus below both arterial trunk consistent with DORV

206

M. Shaw and S. H. Chandrashekhara

11.18 Case 11.17 A 12-year-old boy with dyspnea and lower limb pain while playing. Both the femoral pulse was weak. Echo revealed ASD, VSD; however, arch and DTA were not adequately assessed. What are the imaging findings in Xray and CTA (Fig. 11.18)?

Fig. 11.18  X-ray revealed cardiomegaly with dilated RA and pulmonary plethora. CTA shows VSD and CoA (black arrow). Associated ASD also noted in axial image

11  Imaging in Ventricular Septal Defect

207

11.19 VSD 11.19.1 Definition An abnormal communication between the RV and LV with Left → Right shunt.

11.19.2 Classification VSDs are classified based on anatomic components where the VSD is located (Soto classification) (Fig. 11.19) [1]. • Perimembranous (80%) • Muscular (5–20%)—subtyped into mid-muscular, apical, marginal, or multiple • Outlet or subarterial doubly committed 5–7% • Inlet: 5–8%

1. Perimembranous VSD • PM VSD may extend surrounding muscular septum, further categorized according to muscular part involved. • PM defect may have inlet, trabecular, or outlet extension. Perimembranous defect may extend far posteriorly beneath both AV valves and reach crux cordis-atrioventricular canal type of VSD. 2. Muscular VSD • Trabecular muscular septal defect—any part of trabecular septum, multiple (Fig. 11.19) • Separated from membranous septum by trabecula septomarginalis • Inlet, trabecular, outlet types

11.19.3 Can Also Be Classified as Follows (a) Kirklin Classification: • TYPE I: Conal, subarterial

supracristal,

infundibular,

• TYPE II: Perimembranous • TYPE III: Inlet/AV canal type • Type IV: Muscular (b) VSD classification as per size: • Large: 2/3rd of aortic annular size or > 15 mm • Moderate: Half of aortic annular size 5 to 15 mm • Small: One third of aortic annular size (c) VSD classification as per lesion size: • Restrictive VS –– B)

11.20.4.2 Factors Precluding VSD Routability 1. Tensor apparatus of AV valves in pathway from LVTO the aorta. Chances of significant subaortic stenosis post-operatively 2. Straddling of tricuspid valve/abnormal tricuspid chordae 11.20.4.3 Subpulmonic VSD VSD closure with LV-PA routing + arterial switch 11.20.4.4 Non-committed VSD • Distance b/w defect and either semilunar valve—more than aortic annulus diameter • Anatomic constraints –– Adequacy and position of VSD –– Abnormalities of AV valve attachment –– Ventricular imbalance 11.20.4.5 Imaging Algorithm [5, 6] See Fig. 11.22.

11  Imaging in Ventricular Septal Defect

211

Table 11.1  Comparison between different imaging modalities for assessment of VSD Parameters Diagnosis of VSD Additional VSDs (particularly apical VSDs) Types of VSD Associated CHD Ventricular function Shunt quantification PAH Reversibility of PAH VSD routability

X-ray ++ −

ECHO ++++ ++

CATH ANGIO ++++ ++++

CT ++++ ++++

MRI ++++ ++++

+ ++ + − + − −

++ +++ +++ ++ ++ − +++

++++ +++ ++ +++ ++++ ++++ +++

+++ ++++ +++ − +++ − +++

+++ ++++ ++++ +++ +++ − ++++

Fig. 11.22  Management algorithm for suspected case of VSDs Fig. 11.21  Pictorial diagram shows measurement to be assessed in the pre-operative evaluation of the VSD

11.20.4.6 How Imaging Impacts Management [7–9] • Surgery for VSD involves patch closure. The location of VSD determines route of closure. • Device closure can be performed for mid-muscular VSD, anterior muscular VSD, post-operative residual VSD. It is

usually performed on patients with weight > 8 kg and left-­ to-­right shunt more than 1.5:1. See Figs. 11.23 and 11.24.

212

M. Shaw and S. H. Chandrashekhara

Fig. 11.25  CTA showing VSD closure device in place Fig. 11.23  Catheter angiography revealing mid muscular VSD

Fig. 11.24  Catheter angiography showing muscular VSD with subsequent insertion of closure device

11  Imaging in Ventricular Septal Defect

11.20.4.7 Role of Cross-Sectional Imaging in Post-operative Period • Residual VSD • Calculated Qp/Qs if residual shunt is present • Assessment of Ventricular function by Echo/MRI • CTA after device closure of VSD to evaluate the presence of residual VSD (if any) 11.20.4.8 Conclusion • VSD is the one of the most common CHD. • Majority of VSDs require only close follow-up because of tendency to close spontaneously. • Echo remains central and most important tool for evaluation. • Large VSDs should be effectively managed for heart failure and early surgery to be considered. Multiple Choice Questions Question 1: All are true about restrictive VSD except: (a) VSD size is  RA (c) Differential increase in lung vascularity in LUL (d) Right-sided aorta can be seen in 3–5% of cases. Question 7: Imaging criteria for routable VSD are all except (a) VSD size more than aortic valve diameter (b) IVS-aortic valve length  diameter of aortic valve annulus (d) IVS-aortic valve length  8 kg (d) Performed when L➔R shunt 5 mm larger than the defect size Question 9: Which MRI sequence will be most useful to quantify shunt in case of RSVA? (a) ECG—Gated CINE MRI (b) Gadolinium Contrast imaging (c) Phase contrast imaging (d) White blood sequence Question 10: All are false related to management of SVA with regard to device closure except (a) Performed for unruptured SVA (b) Can be performed when VSD is present (c) Performed when chances of infections are high at the defect site (d) Performed for congenital RSVA. Answer 1 b

2 b

3 b

4 a

5 a

6 c

7 a

8 c

9 c

10 d

References 1. Larsen WJ. Essentials of human embryology. New York: Churchill Livingstone; 1993. 2. Smith WA.  Aneurysm of the sinus of Valsalva with report of two cases. JAMA. 1914;62:1878. 3. Wang ZJ, Zou CW, Li DC, et al. Surgical repair of sinus of Valsalva aneurysm in Asian patients. Ann Thorac Surg. 2007;84:156–60. 4. Bricker AO, Avutu B, Mohammed TL, et  al. Valsalva sinus aneurysms: findings at CT and MR imaging. Radiographics. 2010;30:99–110. 5. Sakakibara S, Konno S.  Congenital aneurysm of the sinus of Valsalva: anatomy and classification. Am Heart J. 1962;63:405–24. 6. Xin-Jin L, Xuan L, Bo P, et  al. Modified Sakakibara classification system for ruptured sinus of Valsalva aneurysm. J Thorac Cardiovasc Surg. 2013;146:874–8. 7. Taguchi K, Sasaki N, Matsuura Y, et al. Surgical correction of aneurysm of the sinus of Valsalva. A report of forty-five consecutive patients including eight with total replacement of the aortic valve. Am J Cardiol. 1969;23:180–91. 8. Hijazi ZM.  Ruputured sinus of valsalva aneurysm: management options. Catheter Cardiovasc Interven. 2003;58:135–6. 9. Arora R, Trehan V, Rangasetty UM, et al. Transcatheter closure of ruptured sinus of valsalva aneurysm. J Interv Cardiol. 2004;17:53–8.

Imaging of Patent Ductus Arteriosus

15

Mahesh Krishna Anne and Amit Ban

15.1 Case 15.1

15.1.3 Echo

15.1.1 Case Presentation

Dilated LV and LA with patent ductus arteriosus with left to right shunt (Qp:Qs-2).

Five-month-old acyanotic girl with breathing difficulty and history of recurrent respiratory infections since birth. O/e— tachypnoea, tachycardia, raised JVP, and continuous cardiac murmur (Fig. 15.1).

15.1.2 Differential Diagnosis • Patent ductus arteriosus • Aorto-pulmonary window • Truncus arteriosus

15.1.4 What Is the Next Investigation? 1. Usually, patent ductus arteriosus is diagnosed and evaluated on transthoracic Echo. 2. Cross-sectional imaging may be needed in the following scenarios: • Detection of associated arch anomalies (interruption/ coarctation of aorta)

M. K. Anne (*) · A. Ban Department of Cardio Vascular Radiology and Endovascular Interventions, AIIMS, Ansari Nagar, New Delhi, India © The Author(s), under exclusive license to Springer Nature Singapore Pte Ltd. 2023 S. Sharma (ed.), Case-based Atlas of Cardiac Imaging, https://doi.org/10.1007/978-981-99-5620-3_15

251

252

M. K. Anne and A. Ban

3. Cardiac catheterization is considered for evaluation of pulmonary vascular resistance and operability in older children and in cases where percutaneous closure of patent ductus arteriosus is considered. 4. CT and MRI are available options for evaluation of these parameters. 5. Contrast CT angiography is preferred over MRI with regard to fast acquisition and lesser dependence on patient sedation. In this case, CT angiography was done to rule out aortic arch anomalies (Fig. 15.2).

15.1.5 What Are the Imaging Findings?

Fig. 15.1  Cardiomegaly with left ventricular enlargement, wide vascular pedicle, prominent main PA segment, bilateral pulmonary hilar segments, increased pulmonary vascular markings

• Tortuous course/PDA not completely evaluated on TTE • Evaluation of branch pulmonary arteries along with lung parenchyma

15.1.5.1 Diagnosis Patent ductus arteriosus with left to right shunt [Krichenko type A PDA—conical]. Differentiation between various aorto-pulmonary vascular channels on imaging: (Table 15.1). 1. Patent ductus arteriosus 2. Persistent fifth arch 3. Aorto-pulmonary collaterals

15  Imaging of Patent Ductus Arteriosus

253

a

b

c

d

ŽƌƚĂ

ŽƌƚĂ

W

W

e

f

W

Fig. 15.2 (a, b) Axial CT images—the PDA (star) communicating the PA and DTA. (c) Oblique sagittal CT image—the PDA (arrow) communicating the PA and DTA. (d) Oblique coronal CT image—the PDA

(star) communicating the PA and DTA. (e) VRT image—the conical PDA (blue arrow) with narrow end towards the PA. (f) CT lung window axial image—mosaic attenuation involving bilateral lower lobes

254

M. K. Anne and A. Ban

Table 15.1  Differentiation between various aorto-pulmonary vascular channels on imaging Patent ductus arteriosus Persistent fifth arch

Aorto-­pulmonary collaterals

Persistence of fetal communication between aorta and left PA or bifurcation of main PA Persistence of fetal fifth aortic arch Can be either, beneficial as in systemic outflow tract obstructions or cause hemodynamic compromise when associated with a significant left to right shunt Usually associated with tetralogy of Fallot with severe pulmonary stenosis, pulmonary atresia with VSD

Imaging Arising from undersurface of arch of aorta and draining into left PA A persistent fifth arch can connect the ascending aorta to the descending aorta (systemic-to-­systemic connection) or the ascending aorta to a derivative of the sixth arch, usually the left PA (systemic-to-pulmonary connection) Vascular channels usually arising from undersurface of arch or descending aorta and reaching up to posterior aspect of hilum

15.2 Case 15.2

15.2.2 Diagnosis

15.2.1 Clinical Presentation

Systemic-to-systemic communicating persistent fifth aortic arch in case of tetralogy of Fallot.

One-year-old boy presenting with cyanotic spells (Fig. 15.3).

15  Imaging of Patent Ductus Arteriosus

a

Fig. 15.3 (a) Axial CT image shows fourth aortic arch (left sided arch). (b) Axial CT image at lower level as figure (a) showing communicating channel between ascending aorta and descending thoracic aorta. (c) MIP axial image shows infundibular and valvular pulmonary stenosis with small caliber main PA, RPA & LPA. (d) Oblique sagittal CT image shows abnormal vascular channel (blue star) communicating

255

b

ascending aorta and descending aorta (anomalous channel noted below level of normal aortic arch). (e, f) Oblique coronal CT image and 3D-reconstructed image show abnormal vascular channel (blue star) communicating ascending aorta proximal to origin of innominate artery (black star) and descending aorta

M. K. Anne and A. Ban

256

c

d

d ƐĐ͘ ŽƌƚĂ

e

f

d

d

Fig. 15.3 (continued)

15  Imaging of Patent Ductus Arteriosus

257

15.3 Case 15.3

15.3.2 Diagnosis

15.3.1 Case Presentation

Tetralogy of Fallot with severe pulmonary stenosis, right aortic arch, confluent pulmonary arteries, small caliber bilateral pulmonary arteries, and persistent fifth aortic arch. Few significant aorto-pulmonary collaterals.

Five-year-old boy with cyanotic spells and recurrent respiratory infections. Clinical examination shows tachypnoea, cyanosis (Fig. 15.4).

a

b

Aortic arch

Asc.A

DTA

Fig. 15.4  CXR—RV cardiac apex with right-sided aortic arch and absent pulmonary conus. Small caliber right and left pulmonary hilar segments. Pulmonary vascularity appears reduced. (a) Axial CT image shows origin of persistent fifth arch from arch. (b) Axial CT image showing pulmonic stenosis with small caliber main and both right & left main pulmonary arteries. (c, d) Coronal CT images showing persistent fifth arch origin, coursing to left hilum and draining into left PA at

hilum. (e) Oblique sagittal CT image showing origin of persistent fifth arch proximal to the origin of innominate artery (black star). (f) 3D reconstructed images showing persistent fifth arch, its origin proximal to innominate artery from aorta and tortuous course to drain into hilar part of left PA (blue star denotes large aorto-pulmonary collateral from DTA to left lung)

258

M. K. Anne and A. Ban

c

d

e

f

Arch

Arch

LPA

D T A

Fig. 15.4 (continued)

15  Imaging of Patent Ductus Arteriosus

259

15.4 Case 15.4

15.4.2 Diagnosis

15.4.1 Case Presentation

Pulmonary atresia with VSD, confluent pulmonary arteries, large peripherally calcified aorto-pulmonary collateral from undersurface of arch to right lung.

Thirty-six-year-old male with DOE, cough since last 2–3 months. Clinical examination showed tachypnoea, cyanosis, clubbing, and continuous cardiac murmur (Fig. 15.5).

a

b

MPA

Fig. 15.5  CXR—cardiomegaly with prominent ascending, arch and descending aorta, prominent bilateral pulmonary arteries. Curvilinear calcification noted in the region of aorto-pulmonary window region. Soft tissue radio-opacity is noted involving left upper lung zone. (a) Axial CT images show the origin of the anomalous vascular channel from aortic arch. (b) Axial MIP CT image shows confluent pulmonary arteries. (c, d) Oblique sagittal CT images showing origin of the col-

lateral channel from undersurface of arch with multiple wall calcifications and supplying the right lung. (e) Oblique coronal image showing significant large tortuous aorto-pulmonary collateral supplying the right lung. (f) 3D reconstructed images showing significant aorto-pulmonary collateral from under surface of arch and traversing into the right hilum along with right PA

260

M. K. Anne and A. Ban

c

d

Arch of aorta Arch

e

f

Aortic arch

Fig. 15.5 (continued)

15  Imaging of Patent Ductus Arteriosus

261

15.5 Patent Ductus Arteriosus

15.5.4 Clinical Features

15.5.1 Introduction

1. Hemodynamic effects of an isolated PDA depend on the ductal resistance/restriction and on the pressure gradient between the systemic and pulmonary circuits. (a) Small and restrictive PDA • No significant hemodynamic effect • Patients are asymptomatic (b) Moderate to large PDA • Pulmonary over-circulation and left heart volume overload • Chronic volume overload of the left heart can lead to onset of congestive heart failure in adulthood • Long-standing left to right shunting → pulmonary hypertension • Persistently increased pulmonary vascular resistance → Pulmonary resistance > systemic resistance → Eisenmenger phenomenon 2. A rare late mode of presentation of a PDA is infective endocarditis. The presence of a PDA may alter the local vascular immune response mechanisms or cause endothelial damage, providing a nidus for bacterial colonization.

The ductus arteriosus is encountered as a normal vascular connection between the PA and thoracic aorta forming an essential part of the fetal circulation physiology. It facilitates right-to-left shunting of oxygenated blood from placenta to the systemic circulation by bypassing the high-resistance pulmonary bed. Following birth, increased oxygen tension and reduced pulmonary vascular resistance promote spontaneous closure of the ductus. Patent ductus arteriosus (PDA) is the persistent vascular structure beyond the early neonatal period, causing left-to-right shunt between the aorta and PA with subsequent pulmonary over-circulation.

15.5.2 Epidemiology Patent ductus arteriosus predominates in females, with a gender ratio of 2 or 3 to 1 [1]. Isolated PDA accounts for 5–10% of congenital heart diseases with an incidence of 0.5 in 1000 live-term births [2].

15.5.3 Embryology The normal ductus arteriosus develops from the dorsal portion of the left sixth arch. However, because of the bilateral symmetry of the arterial system, arterial ducts can be bilateral or absent, right- or left-sided. The medial smooth muscle is particularly sensitive to PGE1 causing relaxation/dilatation and PGI2 promoting ductal constriction [3, 4]. The first stage of ductal closure occurs within 15 to 20 h after birth. This is triggered by a rapid decrease in plasma concentration of prostaglandins. The second stage of ductal closure is completed within 2 to 3 weeks of birth. This occurs by fibrous proliferation in the intima. Because of the immaturity of the ductal tissue, preterm infants respond poorly to increases in oxygen tension and therefore become the patient population at highest risk of developing PDA persistence. Inability of the immature lungs to clear circulating vasodilators may also contribute to ductal persistence. Patent ductus arteriosus beyond 3 months of age is considered abnormal.

15.5.5 Associations • ASD and VSD are common associations [5, 6]. • Other associated cardiovascular anomalies include: –– Coarctation of aorta –– Interrupted aortic arch –– Hypoplastic left heart –– Pulmonary atresia with VSD –– Tetralogy of Fallot –– Transposition of great vessels

15.5.5.1 Ductus-Dependent Circulations Certain congenital heart diseases have hemodynamics such that a patent ductus arteriosus is essential for the survival of the patient. This can be related to maintaining pulmonary blood flow in cases of pulmonary atresia with VSD or Tricuspid atresia or maintaining systemic blood flow in cases of arch interruption or coarctation of aorta. In these scenarios, it is imperative to keep the ductus patent.

M. K. Anne and A. Ban

262

15.6 Goals of Imaging 15.6.1 Anatomic Features  •   •   •   • 

Identification of complete course and morphology of PDA. Presence of arch anomalies. Evaluation of pulmonary arteries. Detection of associated cardiac anomalies.

15.6.2 Hemodynamic Evaluation  •  Quantification of shunt.  •  Evaluation of associated anomalies.  •  Pulmonary vascular resistance and reversibility.

15  Imaging of Patent Ductus Arteriosus

263

15.7 Imaging Modalities

15.7.1 Imaging in Management Planning

(a) Chest radiographs: Initial investigation. May be normal in small shunt. Cardiomegaly with left ventricular and left atrial enlargement of varying degree depending on the amount of shunting present. Pulmonary plethora due to the left to right shunt. Peripheral pruning of pulmonary vasculature. (b) Cross-sectional imaging (CT/MRI): Magnetic resonance imaging and computed tomography may be useful in defining the anatomy in patients with unusual PDA geometry and in patients with associated abnormalities of the aortic arch. Important assessment areas through cross-sectional imaging include:

Important features include:

Location and size (length and width) Presence and extent of calcifications, constrictions, or aneurysms Presence of vegetations Size and morphology of the great arteries for the presence of pulmonary hypertension Morphology, size, and function of the cardiac chambers, particularly the LA and LV

Krichenko et al. classified the PDA based on its morphology [7] (Table 15.2). (c) Cardiac catheterization: It is indicated • for assessing the pulmonary vascular resistance, • reversibility of pulmonary hypertension. • It is particularly important in older children and adults.

Table 15.2  Angiographic classification of PDA Type A (conical) B (window) C (tubular) D (complex) E (elongated)

Morphology Well-defined aortic ampulla and constriction near the PA Very large and very short Without constrictions Multiple constrictions Constriction remote from the anterior edge of trachea

• • • •

The minimum diameter The largest diameter (usually at the aortic ampulla) The length The relationship of the ductus to the anterior border of the tracheal shadow, which helps guide device positioning • Pulmonary vascular resistance • Reversibility of pulmonary hypertension This information is essential so that the proper device and device size can be chosen for the intervention.

15.7.2 Guidelines for Closure of PDA According to Indian Guidelines [8] 15.7.2.1 Age for Closure of PDA • Large/moderate PDA, with congestive heart failure, PA hypertension: Early closure (by 3 months) (Class I) • Moderate PDA, no congestive heart failure: 6 months–1 year (Class I). If failure to thrive present, closure can be accomplished earlier (Class IIa) • Small PDA: At 12–18 months (Class I) • Silent PDA: Closure not recommended (Class III) 15.7.2.2 Closure of PDA Weight > 6 kg—Can be individualized. Device closure (preferred as less invasive), coil occlusion, or surgical ligation (Class I). Weight  right) with few areas of mosaic attenuation (air-trapping). (h) 3D reconstructed image of trachea and main bronchi shows indentation on left (due to arch), right (due to aberrant course) and posterior aspects (due to aberrant course and dilatation of origin of artery) of lower trachea

290

N. Kumar and A. Ban

17.5 Case 17.4 One-year-old male child presenting with complaints of noisy labored breathing. Clinical examination revealed wheezing and stridor. No cyanosis. See Fig. 17.10.

17.5.1 Imaging Findings See Fig. 17.11.

Fig. 17.10  CT scout image shows right sided aortic arch with indentation of trachea on the right

17  Imaging in Vascular Rings

291

a

d

b

e

c

f

g

Fig. 17.11 (a) CTA axial image shows right aortic arch with retro-­ esophageal course of left subclavian artery; (b) CTA sagittal image shows abnormal vascular structure (arrow) posterior to trachea causing compression over it. (c) CTA coronal image shows right aortic arch with the aberrant left subclavian artery. (d, e) 3D reconstructed images showing right aortic arch with retroesophageal course of left subclavian

artery and left PDA from left subclavian artery to origin of left pulmonary artery forming a complete vascular ring. (f) CT lung window coronal image showing no areas of atelectasis/consolidation/mosaic attenuation. (g) 3D reconstructed image of trachea and main bronchi shows indentation on right and posterior aspects of lower trachea

292

17.6 Case 17.5 Two-year-old female child presenting with acute onset breathlessness. Clinical examination revealed presence of stridor and wheezing.

N. Kumar and A. Ban

17  Imaging in Vascular Rings

293

17.6.1 Chest X-Ray See Fig. 17.12. a

b

d

g

j

c

e

f

h

i

k

Fig. 17.12  Chest X-ray showed multiple areas of air space opacification involving bilateral lung zones with endo-tracheal intubation tube. (a, b) CTA axial images show circumflex aortic arch (left aortic arch with right descending aorta) with PDA from the right descending aorta to right pulmonary artery forming complete vascular ring; (c) CTA sagittal image shows abnormal vascular structure (arrow) posterior to trachea, endotracheal tube (star) noted within the trachea. (d, e) CTA coronal images show left aortic arch which arches posterior to trachea

and forms right descending aorta with presence of aberrant right subclavian artery (star). (f, g) 3D reconstructed images showing circumflex aortic arch (left aortic arch with right descending aorta) with PDA from the right descending aorta to right pulmonary artery forming complete vascular ring, blue arrow shows aberrant right subclavian artery. (h, i) CT lung window axial & coronal images show areas of consolidation involving bilateral lung parenchyma. Pre-operative (j, l) & post-­ operative (k, m) images

294

N. Kumar and A. Ban

l

Fig. 17.12 (continued)

m

295

17  Imaging in Vascular Rings

17.7 Case 17.6

Chest x-ray did not reveal significant abnormality (Fig. 17.13).

One-year-old female child presenting with recurrent respiratory infections since birth. History of noisy breathing since birth. On examination stridor was noted. No cyanosis ­present. a

d

b

e

c

f

g

Fig. 17.13 (a) CTA axial image shows normal round configuration of cross-section of trachea (star) (b) CTA axial image shows narrowing of trachea at the level of innominate artery (red arrow) anterior to the trachea (star); (c) CTA sagittal image shows anterior compression of trachea due to the innominate artery (star) (d) CTA coronal image shows common origin (star) of innominate (red arrow) and left common

carotid (blue arrow) arteries (e) 3D reconstructed image showing common origin of innominate and left common carotid arteries with innominate artery arising from a more left position than normal (f) CT lung window coronal image shows no areas of atelectasis/consolidation/ mosaic attenuation. (g) 3D reconstructed images show anterior compression of trachea

296

N. Kumar and A. Ban

17.8 Case 17.7

cardiomegaly with left atrial, right atrial and right ventricular enlargement, pulmonary plethora, and narrow vascular pediA 32-year-old male presenting with dyspnea on exertion cle. Echo revealed presence of a large ASD. Incidental findsince 5–6 months. No history of coronary artery disease/dia- ing of left pulmonary artery sling on CT angiography betes/hypertension. Clinical examination suspected intra-­ (Fig. 17.14). cardiac Left to right shunt. No cyanosis. Chest X-ray showed

a

d

b

e

Fig. 17.14 (a) CTA axial image shows left pulmonary artery arising from the right pulmonary artery and coursing posterior to trachea to reach left hilum; (b) CTA sagittal image shows posterior compression of trachea due to the left pulmonary artery (star); (c, d) 3D reconstructed images showing course of left pulmonary artery and formation

c

f

of pulmonary sling; (e) CT lung window coronal image shows mosaic attenuation pattern involving bilateral lung parenchyma (possibly due to perfusion disturbances); (f) 3D reconstructed images of trachea show posterior indentation of lower trachea

17  Imaging in Vascular Rings

297

17.9 Case 17.8 Three-year-old male child presenting with dysphagia on intake of solid food and occasional noisy breathing. Clinical examination showed presence of wheezing. No cyanosis.

17.9.1 Chest X-Ray See Fig. 17.15 and CT (Fig. 17.16).

Fig. 17.15  Chest X-ray (rotated view) showed prominent impression over trachea on the left side

298

N. Kumar and A. Ban

a

b

d

Fig. 17.16 (a) CTA axial image shows left aortic arch (red star) with retro-esophageal course of right subclavian artery (blue star); (b) CTA sagittal image shows abnormal vascular structure (arrow) posterior to trachea causing compression over it. (c) CTA coronal image shows left aortic arch (red star) with the aberrant right subclavian (blue star). (d, e)

c

e

3D reconstructed images showing left aortic arch with retroesophageal course of right subclavian artery, no PDA present. (f, g) CT lung window axial and coronal images show no areas of atelectasis/consolidation/mosaic attenuation

17  Imaging in Vascular Rings

f

299

g

Fig. 17.16 (continued)

17.10 Case 17.9 A 23-year-old male with complaints of dysphagia and occasional cough for 6 months.

17.10.1 Chest X-Ray See Fig. 17.17 and CT (Fig. 17.18).

17.10.2 Specific Features of Vascular Ring Morphologies 1. Double aortic arch: (a) Most common symptomatic vascular ring (b) Ring formed by both patent arches and/atretic segment (c) Usually one arch is dominant, and the other arch is smaller or may be atretic (in up to 25–34% of cases) (d) The right-sided (posterior) arch—dominant in 75% (e) The left-sided (anterior) arch—dominant in 18% (f) Arches were equal in size in 7% of the patients of double aortic arch (g) Determination of the non-dominant arch important for surgical planning (h) Not associated with any major cardiac anomaly (i) Cross-sectional imaging: • Characteristic appearance of the double arch is the “four-artery sign” • Symmetrical trapezoidal or square appearance of the four arch branch vessels at the level of the thoracic inlet • In case of atretic segment, imaging clues that need to be looked for to correctly differentiate from right aortic arch variants include:

Fig. 17.17  Chest X-ray showed right-sided aortic arch with indentation of trachea on right side

–– Ductus/diverticulum/descending aorta on opposite side –– “Four vessel” sign at thoracic inlet –– Subclavian artery taking more inferior course 2. Right aortic arch (RAA) vascular ring variants (Table 17.5) (a) Right aortic arch with isolated left subclavian artery-rare form. • The left subclavian artery is not attached to either the arch or the left common carotid artery, but instead connects to the pulmonary artery via a ductus arteriosus that may be patent or closed.

300 Fig. 17.18 (a) CTA axial image shows right aortic arch (red star) with retro-esophageal course of left subclavian artery with dilated origin (blue star) (Kommerell’s Diverticulum); (b) CTA sagittal image shows abnormal vascular structure (arrow) posterior to trachea causing compression over it. (c) CTA coronal MIP image shows right aortic arch (red star) with the aberrant left subclavian (blue arrows) and Kommerell’s diverticulum (blue star). (d, e) 3D reconstructed images showing right aortic arch with retroesophageal course of left subclavian artery with no PDA present. (f) CT lung window coronal image shows no areas of atelectasis/ consolidation/mosaic attenuation. (g) 3D reconstructed image of trachea and main bronchi shows indentation on right (due to arch) and posterior aspect (due to aberrant course and dilatation of origin of artery) of trachea

N. Kumar and A. Ban

a

b

d

c

e

17  Imaging in Vascular Rings

301

f

g

Fig. 17.18 (continued) Table 17.5  Vascular ring variants with right sided aortic arch (a) RAA with aberrant left subclavian and left ductus Incomplete ring:  •  No ligamentum  • May be related to Kommerell diverticulum causing mild compression Complete ring:  • Retroesophageal course of the left subclavian artery  • Ligament attaching the descending aorta (posterior) to the pulmonary artery (anterior)

3. 4.



5.

(b) RAA with mirror branching and ductus/ligamentum to descending aorta. Incomplete ring:  • Ligamentum is usually between innominate artery and pulmonary artery Complete ring:  • When the ligamentum arises from the aorta behind the esophagus and connects with the left pulmonary artery

• This anomaly can result in congenital subclavian steal syndrome and vertebrobasilar insufficiency. Left aortic arch (LAA) vascular ring variants (Table 17.6) Circumflex aorta (a) Occurs when a portion of the aortic arch (either right or left) extends behind the esophagus while the ascending and descending thoracic aortic segments are located on either side of the spine (b) The ductus or ligamentum extends from the descending aorta to the pulmonary artery, completing the ring. (c) Side of the arch determines the type of circumflex aorta. (d) Right circumflex aorta is more common than left. Innominate artery compression syndrome (a) Brachiocephalic artery has an anomalous course, originating more posterior and to the left, from the aortic arch.

Table 17.6  Vascular ring variants with left aortic arch LAA with aberrant right subclavian  •  Most common vascular anomaly  •  Not a complete ring  • Can produce dysphagia due to dilatation of the origin of subclavian artery in elderly population

LAA with right subclavian and right ductus  • Uncommon to have right ductus  • If present, complete ring formed

(b) Compression over anterior surface of trachea is caused by the anomalous brachiocephalic artery. (c) Bronchoscopy shows a pulsatile mass compressing the anterior trachea from the left to right at the level of the vocal cords, which is much higher than the other vascular rings. 6. Pulmonary sling (a) Left pulmonary artery arises anomalously from the right pulmonary artery, passing over the right main bronchus and coursing between the trachea and esophagus to reach the left pulmonary hilum. (b) A pulmonary sling can be classified according to  the associated airway anomalies (Well’s Classification) • Type I pulmonary sling with normal branching pattern. –– Type IA—absence of pre-eparterial tracheal bronchus. –– Type IB—presence of pre-eparterial tracheal bronchus. • Type II pulmonary sling with abnormal branching pattern. –– Type IIA—presence of bridging bronchus from left bronchus supplying the right middle lobe. Right bronchus is present.

302

N. Kumar and A. Ban

–– Type IIB—No right bronchus present, right lung aerated by bridging bronchus arising from left bronchus. (c) Complete tracheal rings are associated with pulmonary sling in 35–50% which give characteristic “inverted T” configuration to the carina. (d) Other associated anomalies include—pulmonary artery atresia with ipsilateral lung agenesis; hypoplastic left pulmonary artery.

17.10.3 Management • Surgical correction of the compressive ring. • Preoperative cross-sectional imaging guides towards the surgical approach. Table 17.7 shows thoracotomy approach for various vascular ring anomalies and Table 17.8 shows surgical approach for vascular rings.

17.10.3.1 Role of Cross-Sectional Imaging in Post-Operative Period • No further imaging is indicated in asymptomatic patients following surgery. • Patients with persistent symptoms after surgery: –– Incomplete resection of compressing structure. –– Tracheo-bronchomalacia.

Table 17.7  Thoracotomy approach for vascular ring anomalies Left thoracotomy Right thoracotomy

Median sternotomy

 •  Double aortic arch with right dominant arch  •  Right aortic arch with left ligamentum/ ductus  •  Double aortic arch (left dominant arch)  •  Left aortic arch, aberrant right subclavian artery, right descending aorta and right ductus arteriosus is present  •  Reserved for the correction of associated intracardiac anomalies  •  Repair of pulmonary artery slings with or without sliding tracheoplasty

Table 17.8  Surgical approach for vascular ring anomalies Double aortic arch

Right aortic arch, aberrant left subclavian artery and left ligamentum Kommerell diverticulum

 •  Non-dominant aortic arch (which may be patent or atretic) and the ligamentum arteriosum are divided  •  Ligamentum alone is divided

 •  Excision in the primary operation, in order to avoid aneurysmal dilatation and recurrence of symptoms

• CT/MR imaging may provide additional or confirmatory anatomical detail of the trachea to determine whether a reoperation is warranted. • Tracheomalacia improves in majority of cases of vascular rings, post-operatively. • Severe tracheomalacia can be present in 5–10% cases where its management is initially conservative. • Surgical options including aortopexy, tracheostomy, external splinting, and internal stenting can be considered depending upon individual case scenario.

17.10.4 Future Prospective 17.10.4.1 3D Printing • The clinical applications of 3D printing in CHD can be summarized into five main areas: –– Preoperative planning, –– Pre-surgical simulation, –– Intra-operative orientation, –– Medical education and –– Communication in medical practice. • 3D-printed models serve as a useful and effective tool to improve understanding of complex cardiac structures in relation to the CHD [7]. • These models have shown to help pre-operative planning and achieve better operative success rates. • With respect to medical education, Jones et al. [8] studied the use of 3D models of vascular rings and slings to improve resident education. They concluded that a measurable gain in knowledge was noted about vascular rings and pulmonary artery slings with the addition of 3D-printed models of the defects [8]. • Few hurdles regarding this prospect that need to be addressed are: accuracy and cost. • Only few studies have shown the accuracy of the 3D models. Larger studies need to be conducted to evaluate its accuracy. • Feasibility of low-cost techniques needs to be considered for before widespread utilization. Multiple Choice Questions Question 1: All are complete vascular rings except: (a) Right aortic arch with mirror branching and ligamentum to innominate artery. (b) Right aortic arch with aberrant left subclavian artery with ligamentum to descending aorta. (c) Double aortic arch. (d) Right aortic arch with left descending aorta and left ligamentum. Question 2: Which of the following is the most common complete vascular ring configuration? (a) Double aortic arch

17  Imaging in Vascular Rings

(b) Right aortic arch with aberrant left subclavian artery. (c) Pulmonary artery sling. (d) Left aortic arch with right aberrant subclavian artery. Question 3: Cono-truncal anomalies are commonly related to which of the following: (a) Right aortic arch with aberrant left subclavian artery. (b) Right aortic arch with mirror branching. (c) Left aortic arch with aberrant right subclavian artery. (d) Left circumflex aorta. Question 4: Which of the following statements are NOT true with respect to Pulmonary artery sling? (a) It is commonly associated with abnormal airway branching. (b) It is associated with Tracheo-bronchomalacia. (c) Ring-sling complex denotes pulmonary artery sling + double aortic arch. (d) Unilateral pulmonary agenesis has been reported to be an association of pulmonary artery sling. Question 5: All are true with respect to clinical presentation in vascular rings except: (a) Onset of symptoms depends on the degree of compression. (b) Usual age of presentation for double aortic arch is first few weeks of life. (c) Kommerell’s diverticulum is usually seen in neonates. (d) Usual age of presentation for innominate artery compression is 3–6 months of life. Question 6: Excessive dynamic airway collapse (EDAC) refers to (a) Dynamic form of central airway obstruction characterized by a decrease of ≥10% in the cross-sectional area of the tracheobronchial lumen on expiratory phase compared to inspiratory phase. (b) Dynamic form of central airway obstruction characterized by a decrease of ≥20% in the cross-sectional area of the tracheobronchial lumen on expiratory phase compared to inspiratory phase. (c) Dynamic form of central airway obstruction characterized by a decrease of ≥30% in the cross-sectional area of the tracheobronchial lumen on expiratory phase compared to inspiratory phase. (d) Dynamic form of central airway obstruction characterized by a decrease of ≥50% in the cross-sectional area of the tracheobronchial lumen on expiratory phase compared to inspiratory phase. Question 7: Well’s classification for Pulmonary artery sling depends on (a) Airway branching. (b) Pulmonary artery morphology. (c) Tracheo-bronchomalacia. (d) Tracheal cartilage morphology.

303

Question 8: Which of the following statement/s is/are true related to Double aortic arch? (a) Right dominant arch seen in 75%. (b) Left dominant arch seen in 18%. (c) Co-dominant arch seen in 7%. (d) All the above. Question 9: Which of the following aspect is NOT useful in differentiating between double aortic arch with atretic segment from right aortic arch with mirror branching variant? (a) Four vessel sign. (b) Subclavian artery taking more inferior course. (c) Diverticulum noted at descending aorta. (d) None of the above. Question 10: All are true related to management of vascular rings except: (a) Left thoracotomy done in right aortic arch ring morphology. (b) Right thoracotomy done in double aortic arch with left dominant arch. (c) Median thoracotomy done in cases associated with intra-cardiac lesions. (d) Symptomatic Kommerell’s diverticulum is usually treated conservatively. Answers 1 a

2 a

3 b

4 c

5 c

6 d

7 a

8 d

9 d

10 d

Appendix 1: Imaging Protocol As discussed in Chap. 3.

Appendix 2: Reporting Template As discussed in Chap. 3.

References 1. Backer CL, Mavroudis C.  Congenital heart surgery nomenclature and database project: vascular rings, tracheal stenosis, pectus excavatum. Ann Thorac Surg. 2000;69(3):308–18. https://doi. org/10.1016/s0003-­4975(99)01279-­5. 2. Bonnard A, Auber F, Fourcade L, Marchac V, Emond S, Révillon Y. Vascular ring abnormalities: a retrospective study of 62 cases. J Pediatr Surg. 2003;38:539–43. 3. Menaissy YM, Elgamal M-AF, Amin S, Zaki AF. Vascular rings and slings: a challenging diagnostic and therapeutic rare disease entity. J Egypt Soc Cardiothorac Surg. 2017;25:349–55. 4. Hanneman K, Newman B, Chan F. Congenital variants and anomalies of the aortic arch. Radiographics. 2017;37(1):32–51.

304 5. Zhong YM, Jaffe RB, Zhu M, Gao W, Sun AM, Wang Q. CT assessment of tracheobronchial anomaly in left pulmonary artery sling. Pediatr Radiol. 2010;40(11):1755–62. 6. Su SC, Masters IB, Buntain H, Frawley K, Sarikwal A, Watson D, et  al. A comparison of virtual bronchoscopy versus flexible bronchoscopy in the diagnosis of tracheobronchomalacia in children. Pediatr Pulmonol. 2016;52(4):480–6. https://doi.org/10.1002/ ppul.23606.

N. Kumar and A. Ban 7. Lau I, Sun Z.  Three-dimensional printing in congenital heart disease: a systematic review. J Med Radiat Sci. 2018;65(3):226–36. https://doi.org/10.1002/jmrs.268. 8. Jones TW, Seckeler MD.  Use of 3D models of vascular rings and slings to improve resident education. Congenit Heart Dis. 2017;12(5):578–82. https://doi.org/10.1111/chd.12486.

Imaging in Heterotaxy Syndromes

18

Avichala Taxak

18.1 Case 18.1 18.1.1 Clinical Presentation A 22-year-old female, who had dyspnea since her childhood, presented with worsening of dyspnea and chest pain 4 months ago. See Fig. 18.1: Chest Radiograph

18.1.2 Echo Showed PAPVC, dilated MPA, RVH and dilated RA, RV.

18.1.3 What Will Be the Next Investigations? 1. Issues that need to be addressed include: (a) Anatomic evaluation: morphological anatomy, vascular anatomy, orientation of organs, and heart chambers. (b) Functional evaluation: to assess for systolic as well as diastolic dysfunction. (c) Hemodynamic evaluation: evaluation of EF, gradient quantification. 2. Available imaging options include: (a) Catheter angiography. (b) Orthogonal imaging (CT and MRI). 3. The information provided by catheter angiography is limited. 4. Orthogonal imaging provides superior spatial and temporal resolution as well as comprehensive evaluation with

Fig. 18.1  CXR shows normal cardiac shadow

respect to anatomic, functional, and hemodynamic assessment. 5. CT was performed in this case.

18.1.4 Orthogonal Imaging See Fig. 18.2: CTA.

18.1.5 Final Diagnosis Left isomerism with PAPVC, SV-ASD, interruption of intrahepatic IVC with hemiazygos continuation into LSVC.

A. Taxak (*) Department of Cardio Vascular Radiology and Endovascular Interventions, AIIMS, Ansari Nagar, New Delhi, India © The Author(s), under exclusive license to Springer Nature Singapore Pte Ltd. 2023 S. Sharma (ed.), Case-based Atlas of Cardiac Imaging, https://doi.org/10.1007/978-981-99-5620-3_18

305

306

A. Taxak

a

b

c

d

e

f

g

h

Fig. 18.2  CTA short axis and axial contrast-enhanced, cardiac-gated images demonstrating (a) four-chamber view showing enlarged RA, RV; (b) PAPVC; (c) SV-ASD (asterisk); (d) bronchial left isomerism;

18.2 Case 18.2 18.2.1 Clinical Presentation A 14-year-old female child, who had a history of on and off breathing difficulty, presented with easy fatigability and dyspnea since 1.5 months.

(e) polysplenia; (f) interruption of intrahepatic IVC with hemiazygos (asterisk) continuation into LSVC (blue arrow) (aorta: white arrow); (g) PAPVC; (h) B/L SVC with inter-communicating vein

See Fig. 18.3: CXR and CTA.

18.2.2 Final Diagnosis Right isomerism with AVSD, TAPVC (supracardiac), DORV.

18  Imaging in Heterotaxy Syndromes

a

307

b

c

e

f

CXR

d

Fig. 18.3  CXR (a): shows dextrocardia, gastric bubble on the right side. Contrast-enhanced, cardiac-gated images demonstrate (b) left-sided liver and gastric fundus with absent spleen; (c) bronchial right isomerism; (d) AVSD (asterisk); (e) DORV; (f) TAPVC

18.3 Case 18.3 18.3.1 Clinical Presentation A 7-year-old female child, who had a history of on and off breathing difficulty, presented with easy fatigability since 6 months.

See Fig. 18.4: CTA.

18.3.2 Final Diagnosis Right isomerism with dextrocardia, AVSD, asplenia, and systemic venous anomalies.

308

a

d

A. Taxak

b

e

c

f

Fig. 18.4  Contrast-enhanced, cardiac-gated images demonstrate (a) midline liver with absent spleen; (b) bronchial right isomerism; (c) dextrocardia; (d) double SVC; (e) b/l morphological right atria (asterisk); (f) LHC & MHV draining into IVC→ LA, and RHV → RA (arrows)

18.4 Case 18.4

18.4.2 Final Diagnosis

18.4.1 Clinical Presentation

Left isomerism with abdominal situs inversus, interruption of intrahepatic IVC with azygos continuation into LSVC

A 6-year-old female child, who had a history of breathing difficulty, presented with easy fatigability since 4 months. See Fig. 18.5: CTA.

18  Imaging in Heterotaxy Syndromes

309

a

b

c

d

e

f

Fig. 18.5  Contrast-enhanced, cardiac-gated images demonstrate (a) abdominal situs inversus; (b) bronchial left isomerism; (c) dextrocardia with morphological RA on the left side; (d) morphological LA on the

right side; (e) all pulmonary veins draining into morphological LA on the right side (*); (f) interruption of intrahepatic IVC with azygos (aorta-asterisk) continuation into LSVC (arrow)

18.5 Heterotaxy Syndromes

• Gastrointestinal abnormalities • Neurological abnormalities

18.5.1 Introduction • “Heterotaxy” is a laterality defect characterized by isomerism of the thoracic organs and random arrangement of the abdominal organs. • Polysplenia syndrome (PS) and asplenia syndrome (AS) are very frequently accompanied by congenital heart disease (CHD), situs ambiguous, and various thoracic and abdominal visceral malformations and malrotation. • A variety of cardiovascular malformations can be expected in patients known to have heterotaxy syndrome. Cardiac abnormalities are present in 50% to 100% of patients and, generally, such abnormalities are accountable for the severity and mortality of patients with such a syndrome [1]. • The incidence of CHD in patients with situs ambiguous is 50% to nearly 100%.

18.5.2 Components • Bronchial and pulmonary abnormalities • Cardiovascular malformations • Visceral abnormalities

18.5.3 Clinical Features • In 91.7% of patients, clinical manifestation occurs in the neonatal period, cyanosis being the main presenting feature, as compared to heart failure. • As clinical presentation in this condition may be similar to other common respiratory condition, high index of clinical suspicion is necessary for early detection. • Few of the cases are asymptomatic and are detected as incidental finding during evaluation of another condition.

18.5.4 Embryology Although the exact timing is not known, most of the abnormalities in the asplenia syndrome can be linked to horizon XIII, a developmental stage of the embryo that corresponds to approximately 28 days gestation and 28 somites. During the period of 20–30 days of gestation, the primitive heart and venous connections form.

310

Disruption of this early embryologic event, when the cardiac chambers are incompletely septated, helps explain the preponderance of common atria, single ventricles, abnormal pulmonary venous connections, and conotruncal anomalies observed in heterotaxy syndrome [2].

18.6 Imaging 18.6.1 Goals of Imaging 18.6.1.1 Anatomic Features   •  Number of and location of the spleen   •  Location of the stomach, liver, and gallbladder   •  Short pancreas and gastrointestinal malrotation   •  Venous anomalies   •  Tracheobronchial trees   •  Associated CHDs 18.6.1.2 Functional and Hemodynamic Evaluation   •  Dynamic evaluation of heart chamber for global/ RWMA, gradients   •  Assessment of percentage and presence of mitral regurgitation, LV function

18.6.2 Imaging Modalities 1. Chest radiographs: • Initial investigation. • The cardiac apex typically appears discordant from that of the stomach and liver. The stomach may also be midline and small (microgastria). • Few features may be inferred from the postero-­anterior (PA) radiograph, such as: Apex deviation Enlarged cardiac silhouette/chamber enlargement Pulmonary plethora/oligemia 2. Echo: • Initial investigation. • Echo will help define the intracardiac anatomy and cardiovascular connections. • It can detect left atrial isomerism and dextrocardia, interruption of the inferior vena cava, atrioVSD (partial presentation), patent arterial duct with aortopulmonary shunt quantification

A. Taxak

3. Ultrasonography (USG): Abdominal US will help identify the spleen(s) and position of the aorta and inferior vena cava in most cases. 4. Catheter angiography: Can provide anatomic details regarding vessel anatomy. However, in addition to being invasive, it cannot provide a detailed 3D evaluation that is essential for pre-­ operative planning. 5. Computed tomography (CT): CT can provide useful additional information, however, and can help exclude presentations that mimic complications such as pulmonary embolus and acute dissection. (Table 18.1) • CT may be preferred with regard to fast acquisition and lesser requirement of patient sedation [3, 4]. • In view of the above, we use the orthogonal imaging techniques to derive the relevant information by CT angiography. • Although both CTA and MRI share advantages as being multi-planar, out-patient based, and acquisition, each have their advantages and limitation (Table 18.2). 6. Cardiac MRI (CMR) (Table 18.3): • Can help identify associated findings and give functional information. • Prognostication • To quatify the gradients across the shunts in pre- and post-operative status. • CS, including morphologic, functional evaluation, and tissue characterization in a reproducible and operator-independent manner.

Table 18.1  Information provided by computed tomography angiography (CTA)  •  Anatomy of the disease process  •  Relationship with surrounding structures  •  Configuration of heart chambers (MIP, multiplanar reformats (MPR) and volume reconstructions)  •  Orientation of abdominal viscera  •  Lung parenchymal changes related to primary pathology  •  Detection of coexisting anomalies  •  3D analysis for pre-operative planning

311

18  Imaging in Heterotaxy Syndromes Table 18.2  Comparison of CTA and MRI with points highlighting their advantages and limitations Advantages

Limitations

CT  •  Volumetric images which can be viewed on multiple planes  •  3D images (volume rendering)  •  High spatial resolution  •  Ability to provide extraluminal information  •  Use of ionizing radiation  •  Needs iodinated contrast material

MRI  •  Ability to provide multiplanar and 3D images  •  Functional information  •  Ability to provide extraluminal information  •  Excellent soft tissue contrast  •  Ability to provide information without use of contrast material  •  Expensive  • Time-consuming modality which may limit use in sick patients or may warrant sedation  •  Contraindications: Metallic foreign body, claustrophobia  • Non-contrast MRA may be degraded by artefacts and often provides limited information

Table 18.3  Information provided by cardiac MRI (CMR) Morphologic features

Functional information

Extracardiac findings

Cardiac MR Ventricular (right and left) morphology including the size Ventricular and septal wall thickness (thickening/thinning) Concordance (A-V, V-A) Measures end-diastolic and end-systolic ventricular volumes Stroke volumes and ejection fractions for each ventricle Regional wall motion abnormalities/global abnormalities Lymphadenopathy, lung parenchymal changes

Table 18.4 Palliative procedures possible in heterotaxy cardiac diseases Single ventricular palliation Right atrial isomerism

Left atrial isomerism

 •  Glenn shunt is usually planned at the age of 15 months  •  SVC is anastomosed with the PA to increase pulmonary blood flow  •  Systemic-pulmonary shunt  •  Cavo-pulmonary bi-directional shunt  •  Completion of fontan procedure  •  Cardiac transplantation  •  Systemic-pulmonary shunt  •  Norwood procedure  •  Kawashima procedure in patients having interrupted IVC with azygos/hemiazygos continuation  •  Bi-ventricular repair  •  Cardiac transplantation

18.7 How Imaging Impacts Management • Initial management is usually conservative, so as to preserve oxygenation and tissue perfusion. • Surgical correction or palliation is indicated in these cases based on ventricular physiology. • Preoperative cross-sectional imaging guides towards the surgical approach. • Palliative procedures (Table 18.4) are usually reserved for patients with right atrial isomerism, owing to more severe cardiac defects. Left atrial isomerism patient undergoes biventricular repair more often.

18.7.1 Role of Cross-Sectional Imaging in Post-­ operative Period • Follow-up imaging is dependent on the stage of palliative procedure done and the clinical status of patient. • Neonates with palliative shunting need to have frequent follow-ups.

• For cavo-pulmonary bi-directional shunt, follow-up at 3–4 month interval is warranted. • After fontan, patient needs to be monitored for any arrhythmias and conduction defects, along with ventricular function.

References 1. Lafuente M, Villalba CN, Mouratian M, Forte AV, Sciegata A, Delucis PG, Capelli H. Clinical presentation and outcome of right isomerism. Rev Argent Cardiol. 2015;83(5):400–5. 2. Yoneyama H, Kondo C, Yamasaki A, Nakanishi T, Sakai S.  Comparison of situs ambiguous patterns between heterotaxy syndromes with polysplenia and asplenia. Eur J Radiol. 2015;84(11):2301–6. 3. Loomba R, Shah PH, Arora Y.  Radiologic considerations in heterotaxy: the need for detailed anatomic evaluation. Cureus. 2016;8:e470. 4. Applegate KE, Goske MJ, Pierce G, Murphy D. Situs revisited: imaging of the heterotaxy syndrome. Radiographics. 1999;19(4):837–52.

Imaging in Coarctation of Aorta

19

Rishabh Khurana

19.1 Case 19.1

19.1.5 Differential Diagnosis

19.1.1 Clinical Presentation

Interrupted aortic arch (type A)

A 10-year-old obese boy with elevated BP for age in both upper limbs on his regular school medical check-up. BP in both lower limbs was lower with a difference of 30 mmHg between upper and lower limbs. Radio-femoral delay was present. Murmur heard over the chest wall posteriorly. Echocardiography was suboptimal due to inadequate acoustic window. See Fig. 19.1: Chest radiograph

19.1.5.1 In Aortic Coarctation • Ascending aorta retains its normal curvature. • Distal aortic arch usually extends beyond the origin of the left subclavian artery over several centimeters. • Aortic arch cannot be visualized in its entirety in a true sagittal plane.

19.1.2 Diagnosis Coarctation of aorta

19.1.3 Next Step in Management Orthogonal imaging will be performed for morphological evaluation, determine aortic dimensions (exclude aortic arch hypoplasia), look for any associated cardiac anomalies, and treatment planning. See Fig. 19.1b–f: CTA

19.1.4 Final Diagnosis Interruption of aorta likely acquired due to coarctation progression. Normal LV function. No associated anomalies.

19.1.5.2 In Patients with Aortic Arch Interruption • Ascending aorta has usually a smaller caliber and a straight course to its branches. • The branches may show a V configuration on coronal imaging. • Visualization of a single complete thoracic vascular arch on a single sagittal image (resembling a “normal” aortic arch). As a rule, the aortic arch cannot be seen in its entirety on a single sagittal image, because it typically courses within the thorax from anterior and right to posterior and left. In patients with IAA, the PDA may mimic a “normal” aortic arch [oriented in an anteroposterior direction, best seen on sagittal imaging communicating between the pulmonary artery and aorta. Therefore, on a true sagittal plane, presence of an intact aortic arch may be evident in the setting of interruption]. However, a visualized PDA typically lacks the classic morphologic appearance of a normal aortic arch as it appears flat.

R. Khurana (*) Department of Cardio Vascular Radiology and Endovascular Interventions, AIIMS, Ansari Nagar, New Delhi, India © The Author(s), under exclusive license to Springer Nature Singapore Pte Ltd. 2023 S. Sharma (ed.), Case-based Atlas of Cardiac Imaging, https://doi.org/10.1007/978-981-99-5620-3_19

313

314

b

c

DTA

a

R. Khurana

d

e

f

Fig. 19.1 (a) Heart size is normal (LV configuration). Aorta shows the characteristic “fig. 3” sign. Lung vasculature is normal. Bilateral rib notching seen involving inferior border of posterior ends of fifth to ninth ribs (orange arrows). (b, c) Left-sided bovine aortic arch with interruption of proximal DTA ~1.4  cm distal to origin of LSCA (red arrow). Rest of the DTA is reformed via the collaterals from the bilat-

eral intercostal arteries (blue arrows). Length of interrupted segment: 8 mm. Distal Arch (proximal to interruption): 8.5 mm. Proximal DTA distal to the interruption: 11.2 mm. DTA at the diaphragm: 14.7 mm. (d–f) Collateral vessels seen in bilateral periscapular region (blue circles). Bilateral IMA are hypertrophied (Yellow arrows)

19.1.6 NSAA

19.1.7 Pseudocoarctation

In NSAA, there will be circumferential mural thickening involving aorta, arch vessels, visceral branches with luminal narrowing.

There is presence of elongation/narrowing or kinking. However, there is no pressure gradient, no collateral formation, and no rib notching.

19  Imaging in Coarctation of Aorta

19.2 Coarctation of Aorta 19.2.1 Introduction [1] Narrowing of proximal descending thoracic aorta typically located at the insertion of the ductus arteriosus just distal to the LSCA

19.2.1.1 Types Infantile and Adult Infantile (Preductal): characterized by diffuse hypoplasia or narrowing of the aorta from just distal to the brachiocephalic artery to the level of ductus arteriosus along with a more discrete area of constriction just proximal to the ductus but distal to the origin of the LSCA. The blood supply to the descending aorta is via PDA. Adult (Juxta-ductal): characterized by a short segment discrete stenosis of postductal aorta. It typically occurs just distal to the ligamentum arteriosum.

19.2.2 Etiology [2] Precise pathogenesis is unknown. Two main theories for development of congenital Coarctation of aorta (CoA) are 1. Reduced antegrade intrauterine blood flow leads to underdevelopment of fetal aortic arch. 2. Migration or extension of ductal tissue into the wall of the fetal thoracic aorta.

19.2.3 Genetic Predisposition Approximately 5–15% girls with CoA have Turner’s syndrome. Up to 30% cases of Turner’s syndrome have CoA. It is suggested to do genetic testing in girls who are diagnosed with coarctation of aorta.

19.2.4 Pathophysiology [2] 19.2.4.1 In Utero CoA does not cause hemodynamic problem in utero as two-­ thirds of the combined cardiac output flows through PDA → DTA and constriction site is bypassed. 19.2.4.2 In Neonate PDA and foramen ovale begin to close, and the fetal parallel circulation transitions to series circulation postnatally. Hence, there is steady increase in cardiac output that must

315

cross the narrowed aortic segment to reach lower extremities. The hemodynamic change ranges from mild systolic hypertension to severe heart failure. It depends upon severity of the coarctation and presence of other associated lesions. In neonates with severe lesions, heart failure may develop, because there is insufficient time for the development of myocardial hypertrophy or collateral blood flow.

19.2.4.3 Adult There is development of LV myocardial hypertrophy, which maintains normal systolic function and EF. Collateral blood flow develops involving the intercostal, internal mammary, and scapular vessels, which circumvents the stenotic lesion.

19.2.5 Associations See Table 19.1.

19.2.6 Natural History • In untreated patients, average survival age of individuals has been reported to be approximately 35 years with only 25% survival by 46 years of age • Common complications in unoperated patients or in those operated on during later childhood or adulthood are systemic hypertension, accelerated CAD, stroke, aortic dissection, and heart failure. • Causes of death include heart failure, aortic rupture, aortic dissection, endocarditis, endarteritis, intracerebral hemorrhage, and myocardial infarction • Patients with an associated bicuspid aortic valve may also develop significant AS/AR and ascending aortic aneurysm

Table 19.1  Associations of coarctation of aorta Cardiac  1.  Bicuspid aortic valve (75–80% cases)  2.  VSD  3.  Truncus arteriosus  4.  TGA  5.  Mitral valve defects (hypoplastic mitral valve, parachute mitral valve, abnormal papillary muscle)  6.  PDA  7.  Supravalvular PS Non-cardiac  1.  Intracranial berry aneurysms (adults)  2.  Spinal scoliosis Syndromic associations: Shone syndrome, Turner’s syndrome, PHACE syndrome

316

19.2.7 Goals of Imaging [2, 3] • Establish diagnosis: Site and severity of obstruction (Anatomical information), Gradient across the stenosis (Hemodynamic assessment), Cardiac chamber assessment: Ventricular hypertrophy, size and function, Collateral vessels. • Detect associated lesions. • Treatment planning. • Follow-up. • Prognostication. • Detection of complications/recurrence. • Brain imaging to detect aneurysms.

19.2.8 Plain Radiograph • Figure of 3 sign • Rib notching (Roesler sign) (Fig. 19.1a, b, f) –– Bilateral rib notching: classic radiologic sign caused by collateral flow through dilated, pulsatile posterior intercostal arteries (unusual 2.1 at end-systole on echo, and >2.3 at end-diastole on CMR) has poor specificity and cannot differentiate: “Pathological noncompaction” from DCM associated with “hypertrabeculation.”  • Proposed alternative criteria for diagnosing non compaction on cardiac MRI:    1. Noncompacted myocardial mass >25%/mass index >15 g/m2    2.  Trabeculation of basal segments  • Ratio of noncompacted to compacted myocardial thickness > 3.0 in 1 or more segments. Athlete’s heart  • Physiological LV dilation has been reported in up to 15% of highly trained athletes.  • However, they don’t have any symptoms/ signs of heart failure.  • Natriuretic peptide levels, ECG, LV systolic and diastolic function, LV peak systolic strain, and contractile reserve are normal.  • Cardiopulmonary exercise testing: High peak VO2.  • Reverse remodeling occurs with deconditioning. For detailed discussion on the above topics, the reader is referred to the respective sections

376

22.8.5 Role of Various Imaging Modalities for a Suspected Case of Nonischemic Cardiomyopathy (a) Plain Radiograph • Identifies chamber dilatation, signs of heart failure, easy tool for follow-up. (b) Echocardiography • Initial imaging modality performed bedside. • Monitor significant changes of wall motion and LV geometry (dilatation and shape e.g., spheroid shape). • Helps to rule out other causes of heart failure: valvular disease, congenital heart disease, or other cardiomyopathies. • Follow-up imaging. (c) CMR [2] • Morphologic and Functional Evaluation: –– Left Ventricle: CMR allows accurate and reproducible assessment of LV volumes, mass, and ejection fraction (considered gold standard noninvasive technique). –– Right ventricle: RV size and function is abnormal in 30–60% of cases of DCM. CMR provides the gold standard noninvasive assessment of both RV size and function [5]. –– Left atrium: Accurate quantification of LA volume using the biplane area-length method can be performed. LA size may be increased in cases of DCM secondary to pressure overload due to LV diastolic impairment, functional mitral regurgitation, and atrial fibrillation [6]. –– Functional mitral regurgitation: It is a common consequence of DCM secondary to mitral annular dilatation and leaflet tethering secondary to LV impairment. Degree of functional mitral regurgitation provides important prognostic information in DCM [7, 8]. –– Strain imaging: Longitudinal strain analysis can predict adverse outcomes, independently of other predictors such as LVEF and LGE, in patients with DCM. –– Due to the accuracy of CMR in functional assessment, it is proposed as the method of choice for the follow-up of patients with DCM after pharmacologic and surgical intervention. • Tissue characterization: –– Fibrosis detection and quantification • Detecting the Pre-DCM phenotype (latent disease) [2] –– LV enlargement without systolic dysfunction is a well-defined precursor of DCMP (inherited cases). –– Strain imaging can provide phenotypic markers of latent disease earlier than LV enlargement occurs.

R. Khurana

For example, in patients receiving cardiotoxic anticancer drugs, reductions in global longitudinal strain can predict development of overt LV dysfunction. –– T1 mapping technique can assess diffuse interstitial fibrosis even in the absence of LGE. –– T2 mapping technique can identify myocardial edema. –– T2* can detect cardiac iron overload. (d) CT: • Coronary evaluation to look for any significant coronary arterial disease and rule out ischemic dilated cardiomyopathy. • Before implantable device insertion. • CT may be done to look for lung parenchymal changes and mediastinal lymphadenopathy if suspected etiology is sarcoidosis. (e) PET: • Shows good agreement with CMR criteria of inflammation in cases of myocarditis • Clinical use remains uncommon (f) Catheter angiography: • Indicated in selected patients with clinical presentation indistinguishable from an acute coronary syndrome, lifestyle-limiting coronary disease despite medical therapy.

22.8.6 Management Strategies [8–11] The treatment strategy for dilated cardiomyopathy focuses on: 1. Medical therapy (beta blockers and ACE inhibiters) 2. Cardiac resynchronization therapy to treat and prevent heart failure 3. Implantable cardioverter defibrillators (ICDs): used to prevent sudden cardiac death

22.8.6.1 Evaluation for ICD 1. Current guidelines recommend ICD implantation in patients with DCM and a reasonable life expectancy and quality of life and either: (a) a history of ventricular arrhythmia with hemodynamic compromise (secondary prevention); or (b) NYHA functional class II/III symptoms and LVEF ≤35%, despite optimal medical therapy (primary prevention). How can Cardiac MRI help? LGE CMR can identifying patients who have high remodeling potential who may warrant lengthier periods of treatment before ICD decision making. Similar principle can be extrapolated to DCM patients who have a “reversible” etiology, for example, alcohol-­ related, peripartum, or acute inflammatory cardiomyopathy.

22  Cardiac MR Imaging in Non Ischemic Dilated Cardiomyopathy

22.8.6.2 Role of LGE in Decision Making for ICD Placement Studies have shown that myocardial fibrosis depicted by LGE-CMR is an independent predictor of SCD risk and all-­ cause mortality. It provides prognostic information (incremental to LVEF alone). On the contrary, absence of mid wall LGE is associated with a low risk of SCD, even when LVEF is ≤35%. 22.8.6.3 Screening of DCM [12] • Clinical screening with ECG and echocardiography should be offered to first-degree relatives of DCM patients who lack a clear underlying etiology. Multiple Choice Questions Question 1: NIDMP is characterized by which of the following? (a) Myocardial disease characterized by LV dilatation with systolic impairment in the absence of significant CAD (b) Myocardial disease characterized by LV dilatation with systolic impairment in the presence of significant CAD (c) Myocardial disease characterized by asymmetrical hypertrophy of septum associated with or without LVO obstruction (d) Myocardial disease characterized by impaired myocardial relaxation during diastole. Question 2: What is not the etiology behind cases of NIDCM? (a) Genetic mutations involving sarcomere proteins, e.g., Titin, myosin, desmin (b) Toxins (c) Anthracycline toxicity (d) Significant CAD Question 3: Which of the following can have a phenotypic overlap with DCMP? (a) Advanced HCM with wall thinning (b) ARVC (Left dominant form) (c) Athlete’s heart (d) All of the above Question 4: How much alcohol consumption can lead to alcohol-related DCMP? (a) 40 g/day for over 3 years (b) 80 g/day for over 5 years (c) 50 g/day for over 1 year (d) 80 g/day for over 2 years

377

Question 5: Which of the following is incorrect with respect to tachyarrhythmia related cardiomyopathy? (a) Presence of persistent tachyarrhythmia (>100 beats/ min) raises suspicion of tachycardia induced cardiomyopathy. (b) Marked LV recovery following effective rate or rhythm control, typically within 4  weeks, confirms the diagnosis. (c) Tests: 12-lead ECG Ambulatory and ECG monitoring (d) Imaging plays no role in the evaluation of this entity Question 6: Which is the following is a determinant of adverse outcome in DCMP? (a) Extent of LV dilatation (b) LVEF (c) LGE: presence and extent (d) All of the above Question 7: Right ventricular size and function can be deranged in what percentage of cases? (a) All cases (b) 80–90 percent of cases (c) 30–60 percent of cases (d) Never deranged Question 8: Increase in Left atrial size is associated with prediction of adverse outcomes. Which of the following LA volumes can predict this? (a) 100 mL/m2 (b) 50 mL/m2 (c) 20 mL/m2 (d) 30 mL/m2 Answers 1 a

2 d

3 d

4 b

5 d

6 d

7 c

8 a

References 1. Elliott P.  Cardiomyopathy. Diagnosis and management of dilated cardiomyopathy. Heart. 2000;84:106–12. 2. Japp AG, Gulati A, Cook SA, Cowie MR, Prasad SK. The diagnosis and evaluation of dilated cardiomyopathy. J Am Coll Cardiol. 2016;67(25):2996–3010. 3. Grothoff M, Pachowsky M, Hoffmann J, et al. Value of cardiovascular MR in diagnosing left ventricular non-compaction cardiomyopathy and in discriminating between other cardiomyopathies. Eur Radiol. 2012;22:2699–709. 4. Fauchier L, Babuty D, Poret P, et  al. Comparison of long-term outcome of alcoholic and idiopathic dilated cardiomyopathy. Eur Heart J. 2000;21:306–14.

378 5. Gulati A, Ismail TF, Jabbour A, et al. The prevalence and prognostic significance of right ventricular systolic dysfunction in nonischemic dilated cardiomyopathy. Circulation. 2013;128:1623–33. 6. Gulati A, Ismail TF, Jabbour A, et al. Clinical utility and prognostic value of left atrial volume assessment by cardiovascular magnetic resonance in non-ischaemic dilated cardiomyopathy. Eur J Heart Fail. 2013;15:660–70. 7. Rossi A, Dini FL, Faggiano P, et al. Independent prognostic value of functional mitral regurgitation in patients with heart failure. A quantitative analysis of 1256 patients with ischaemic and nonischaemic dilated cardiomyopathy. Heart. 2011;97:1675–80. 8. Stolfo D, Merlo M, Pinamonti B, et al. Early improvement of functional mitral regurgitation in patients with idiopathic dilated cardiomyopathy. Am J Cardiol. 2015;115:1137–43.

R. Khurana 9. Neilan TG, Coelho-Filho OR, Danik SB, et al. CMR quantification of myocardial scar provides additive prognostic information in nonischemic cardiomyopathy. J Am Coll Cardiol Img. 2013;6:944–54. 10. Mordi I, Jhund PS, Gardner RS, et al. LGE and NT-proBNP identify low risk of death or arrhythmic events in patients with primary prevention ICDs. J Am Coll Cardiol Img. 2014;7:561–9. 11. Kuruvilla S, Adenaw N, Katwal AB, et  al. Late gadolinium enhancement on CMR predicts adverse cardiovascular outcomes in nonischemic cardiomyopathy: a systematic review and meta-­ analysis. Circ Cardiovasc Imaging. 2014;7:250–8. 12. Charron P, Arad M, Arbustini E, et  al. Genetic counselling and testing in cardiomyopathies: a position statement of the European Society of Cardiology Working Group on myocardial and pericardial diseases. Eur Heart J. 2010;31:2715–26.

Cardiac MR Imaging in Left Ventricular Noncompaction

23

Vasundhara Arora

23.1 Case 23.1 23.1.1 Clinical History A 34-year-old woman was admitted with congestive heart failure. She complained of progressive dyspnea, orthopnea with bilateral pedal edema; no chest pain, palpitations, or syncope.

23.1.2 Investigations 23.1.2.1 Chest Radiograph See Fig. 23.1. Echocardiography revealed dilated Left ventricular cavity with hypertrabeculated myocardium and LV systolic dysfunction (LVEF = 20%)

23.1.3 The Differential Diagnosis Included 1. Dilated cardiomyopathy 2. Left ventricular noncompaction.

Fig. 23.1  Chest X-ray: Borderline cardiomegaly, LV configuration of the apex and mild PVH

V. Arora (*) Department of Cardio Vascular Radiology and Endovascular Interventions, AIIMS, Ansari Nagar, New Delhi, India © The Author(s), under exclusive license to Springer Nature Singapore Pte Ltd. 2023 S. Sharma (ed.), Case-based Atlas of Cardiac Imaging, https://doi.org/10.1007/978-981-99-5620-3_23

379

380

V. Arora

23.1.4 Next Step in Evaluation?

23.1.5 Case Summary

CMR needs to be done to establish a conclusive diagnosis. CMR will provide morphologic and functional evaluation, as well as assess for complications (Fig. 23.2).

Left ventricular noncompaction cardiomyopathy and biventricular dysfunction

a

b

e

f

i

j

m

c

d

g

h

l

k

n

Fig. 23.2 (a–d) SSFP cine four chamber views in End Diastole (a) and End systole (b), short axis midventricular views in End Diastole (c) and End systole (d): showing biventricular dilatation with hypertrabeculated myocardium (yellow star) along anterolateral and inferolateral segments from midventricular to apical LV myocardium. LVEF: 16%, RVEF: 27%. (e, f) T1 TSE SA and 4CH views: No Abnormal myocar-

o dial signal intensity. (g, h) T2 TSE SA and 4CH views: No Abnormal myocardial signal intensity. (i, j) T1 map and T2 map: Normal T1 values (1130 ms) and T2 values (44 ms). (k, l) Perfusion scan SA and 4CH view: No resting perfusion deficit noted. (m–o) LGE images SA views at basal, midventricular, and apical levels: shows patchy late gadolinium enhancement along the trabeculations at midventricular level

23  Cardiac MR Imaging in Left Ventricular Noncompaction

381

23.2 Case 23.2

23.2.1 CMR Diagnosis

A 21-year-old male boy, with complaints of progressive dyspnea, exertional intolerance, and history of resuscitation following sudden cardiac arrest. Echocardiography: concentric LVH, RVH with severe left ventricular dysfunction. CMR was done to provide a better morphologic and functional assessment with tissue characterization (Fig. 23.3).

Biventricular Noncompaction Cardiomyopathy resulting in DCMP

b

a

e

i

c

d

g

f

j

Fig. 23.3 (a, b) SSFP cine four chamber views in end-diastole (a) and in end-systole (b): showing dilated ventricles with prominent biventricular trabeculations (yellow star) s/o biventricular noncompaction. (c–h) SSFP cine SA views in end-diastole (c, e, g) and in end-systole (d, f, h) at basal, midventricular, and apical levels: showing similar

h

k findings of dilated ventricles with biventricular noncompaction and global hypokinesia. (i–k) T2 TSE SA basal, midventricular and apical level: abnormal T2 hyperintensity along subendocardium and along trabeculae (yellow arrow)

382

V. Arora

23.3 Case 23.3

23.3.1 CMR Diagnosis

An 11-year-old boy, product of consanguinity, with complaints of progressive dyspnea and generalized swelling all over the body. Physical examination revealed palmo plantar keratosis with presence of wooly hair. CXR: gross cardiomegaly. Echocardiography: dilatation with trabecular configuration of both ventricles and severe left ventricular dysfunction. CMR was done to provide a better morphologic and functional assessment with tissue characterization (Fig. 23.4).

Biventricular Noncompaction Dilated Cardiomyopathy

Fig. 23.4 (a, b) SSFP cine four chamber views in end-diastole (a) and SA view in end-diastole (b): showing dilated ventricles with prominent biventricular trabeculations. (c) T2 TSE SA midventricular level: abnormal T2 hyperintensity along subendocardium and along trabeculae. (d) LGE SA midventricular level: shows transmural septal enhancement with Patchy LGE along the interstices of trabeculae

23.3.2 Final Diagnosis Carvajal phenotype of Cardio cutaneous Syndrome, characteristic triad of wooly hair, palmoplantar keratosis, and cardiomyopathy

a

b

c

d

23  Cardiac MR Imaging in Left Ventricular Noncompaction

23.4 Case 23.4

mended to evaluate for structural etiology of patient’s chest pain and palpitations (Fig. 23.5).

A 25-year-old male, endurance runner with chronic mild chest pain and palpitations. ECG: left axis deviation with deep T wave inversion in precordial leads. ECHO: preserved ejection fraction, LVH, and prominent trabeculations. Further cardiac imaging including cardiac MRI was recom-

a

c

f

383

23.4.1 CMR Diagnosis Prominent trabeculations involving left ventricle

b

d

e

g

Fig. 23.5 (a, b) T1 TSE (a) and SSFP cine (b) 4CH views: No Abnormal myocardial signal intensity on T1 W images with concentric LVH with prominent trabeculations (Yellow arrow). (c–e) SSFP cine SA views in end-diastole at basal (c), midventricular (d) and apical (e)

h levels: concentric LVH with prominent trabeculations. (f–h) LGE images in SA at basal, midventricular, and apical levels: No abnormal LGE along LV myocardium

V. Arora

384

23.5 Left Ventricular Noncompaction Cardiomyopathy • Left ventricular noncompaction is characterized by hypertrabeculated myocardium with adjacent deep intertrabecular recesses that are in communication with the left ventricular cavity. • It is a relatively rare entity with prevalence less than 0.02%. • This anatomical architecture of prominent trabeculations with a thin compacted layer of myocardium beneath can also be present in the right ventricle, either in a biventricular pattern or as isolated unilateral right ventricular noncompaction.

23.5.1 Etiopathogenesis • The embryonic hypothesis puts forward LVNC as a genetic condition resulting from Intrauterine arrest of normal maturation and compaction of ventricular trabeculae. This hypertrabeculed myocardium with deep adjacent intertrabecular recesses unlike persisting sinusoids communicates with the cavity of left ventricle but not with the coronary circulation [1]. • Multiple gene mutations involving the mitochondrial, cytoskeleton, and sarcomeric proteins have been implicated in the genetic inheritance of LVNC [2, 3]. The Gene mutations identified in Human Left ventricular noncompaction include –– Alpha-dystrobrevin –– G4.5 –– LIM domain-binding protein –– Lamin A/C –– Sarcomeric proteins, that is, beta myosin heavy chain, alpha cardiac actin, cardiac troponin T • Acquired hypertrabeculation or noncompaction has been reported in association with neuromuscular disorders and myotonic dystrophies [3].

23.5.2 Pathophysiology and Clinical Features • LVNC cardiomyopathy manifests as heart failure with reduced ejection fraction and systolic contractiltity more commonly; however, heart failure with preserved ejection fraction and diastolic dysfunction may also be seen [4].

• Hypertrabeculation and LVNC cardiomyopathy are typically associated with Intraventricular conduction disturbances due to poor embryonic development of the His-purkinje network and associated subendocardial fibrosis serving as arrhythmogenic substrate. The ­depolarization disturbances also predispose to malignant ventricular tachy arrhythmias and sudden cardiac death [5]. • Dyssynchrony between noncompacted and compacted myocardium contributes to global LV dysfunction with resultant systemic thrombo embolic complications, which can eventually lead to morbidity and mortality [6].

23.5.3 Diagnostic Criteria • According to the 2006 AHA scientific statement, diagnosis of LVNC cardiomyopathy can be established with echocardiography (ECHO), magnetic resonance imaging (MRI), or LV angiography with ventriculography, but no specific guidelines or imaging criteria are formally recommended (Tables 23.1 and 23.2). Table 23.1  Echocardiography-based diagnostic criteria for ventricular noncompaction Echocardiography based diagnostic criteria for ventricular noncompaction Jenni et al. Stollberger et al. Gerhard et al.  • Prominent  • Noncompacted:  • Compacted ventricular Compacted myocardium trabeculations; myocardial thickness  2 in end in end systole along inferior and diastole in lateral wall in horizontal long apical and mid axis view ventricular LV cavity  • Noncompacted: Compacted myocardial ratio > 2 in end systole short axis view  • Deep intertrabecular recesses communicating with ventricular cavity  • No concurrent cardiac abnormality

23  Cardiac MR Imaging in Left Ventricular Noncompaction Table 23.2  CMR-based noncompaction

diagnostic

criteria

for

ventricular

CMR based diagnostic criteria for ventricular noncompaction Peterson et al. Jacquier et al. Grothoff et al.  • Prominent  • Noncompacted  • Noncompacted characteristic myocardial myocardial appearance of two mass >20% of mass >25% of myocardial layers: global left global left Compacted and ventricular ventricular noncompacted mass mass; myocardium noncompacted myocardium  • Noncompacted absolute mass myocardium >15 gm/m2 showing hypertrabeculations with deep intertrabecular recesses  • Noncompacted: Compacted myocardial ratio >2.3 in end-diastole

23.6 Imaging Modalities for Suspected Case of LVNC 1. Echocardiography: (a) Noninvasive imaging modality. (b) ECHO can be used in patients with a concerning clinical history, those undergoing screening echocardiography for a strongly positive family history, or for clarification of an incidental finding seen on previous ECHO for an unrelated cardiac evaluation. (c) However, operator dependence, poor visualization of the cardiac apex (the most commonly affected area), and the inability to accurately identify a double-­ ­ layered myocardium or trabeculations are some of its limitations. 2. CMR: (a) Cine cardiac imaging performed in different cardiac planes well depicts the double-layered myocardium with better evaluation of the anatomical extent of myocardial trabeculae at end-diastole. Any segmental noncompaction in any area of LV wall can also be well characterized. (b) Cardiac MRI yields a broader cardiac and thoracic and thus associated congenital heart defects; intramural, apical, and intertrabecular thrombi; and incidental extracardiac pathologies can be reliably identified.

385

(c) Cardiac MRI provides a better visualization of the right ventricle and therefore, concomitant right ventricular noncompaction may be better identified. (d) Late gadolinium enhancement imaging can detect myocardial fibrosis that are a harbinger of ectopic foci and potentially lethal arrhythmias in these patients apart from being associated with poor LV function. 3. CT Chest: (a) MDCT can be used for patients with intracardiac defibrillators or resynchronizing devices, where MRI is contraindicated. (b) It provides a noninvasive evaluation of the coronaries and can help exclude coronary artery disease as the underlying cause in patients with myocardial trabeculations and dilated cardiomyopathy.

23.6.1 Goals of CMR • Identification of myocardial trabeculations and noncompacted myocardium • Depiction of areas of ventricular involvement and segmental extent • Identify associated myocardial fibrosis • Evaluate for possible complication such apical or intertrabecular thrombi • Prognostication Cardiac MR modules Morphologic Ventricular (right and left) morphology features including the size Compacted and noncompacted myocardial thickness Anatomical extent of trabeculated myocardium Right ventricular involvement Ventricular thrombi Functional Measures end-diastolic and end-systolic information ventricular volumes Stroke volumes and ejection fractions for each ventricle Trabeculated myocardial mass Regional wall motion abnormalities/global abnormalities Tissue Detects myocardial fibrosis characterization Extracardiac Associated pericardial and pleural effusion findings

386

23.6.2 Complications • • • •

Ventricular tachyarrhythmias Thromboembolism Congestive heart failure Sudden cardiac death

23.7 How Imaging Impacts Management? 23.7.1 Genetic and Family Screening • Genetic evaluation is recommended for patients with LVNC, those with one of the genetic neuromuscular syndromes associated with LVNC, and for patients in whom LVNC is incidentally detected during evaluation for other cardiac ailments. • Family screening can identify familial inheritance as close relatives of patients with LVNC can have isolated LVNC, other forms of congenital heart defects, or different class of cardiomyopathies with or without LVNC [7, 8].

23.7.2 Anticoagulation • Prophylactic Oral anticoagulation for primary prevention of thromboembolism is recommended in patients of LVNC with reduced EF (LVEF  2.3 (c) End-systolic noncompacted: compacted myocardial ratio > 2.3 (d) Trabeculated myocardial mass 40% to ≤45% Low sensitivity of CMR for ARVC diagnosis is due to the fact that electrical abnormalities precede structural changes detected by CMR. So ARVC evaluation should not be solely based on one test particularly CMR

24.4.3.1 Goals of CMR • Diagnosis, which will guide clinical management. • Identify extent and areas of cardiac involvement. • Identify associated complications. • Prognostication and risk stratification. The modified task force criteria are tabulated in Table 24.1: CMR findings in arrhythmogenic ventricular cardiomyopathy

24.4.4 Morphology Increased indexed ventricular EDV Myocardial wall thinning and fatty infiltration (T1 WI) Focal bulges, microaneurysm, segmental dilatation Microaneurysms are not described in revised TFC for CMR.

24.4.5 Function Decreased ejection fraction RWMA: Ventricular akinesia/dyskinesia/dyssynchronous contraction Accordion sign: Focal crinkling of myocardium due to regional dyssynchronous contraction [2].

24.4.6 LGE RV LGE has been reported in up to 88% patients and LV LGE in 61% [2]. LGE in LV involvement: inferolateral walls Septal LGE in more than 50% cases with left dominant ARVC [3]

392

24.4.7 Changing Spectrum of ARVC 1. Displaced triangle of dysplasia: In various studies in last decade, it has been shown that there is preferential involvement of subtricuspid region and LV lateral wall. So it is suggested that there is displacement of RV apex from triangle of dysplasia [2]. 2. Left ventricular involvement: ARVC subjects can have early and predominant LV involvement.

24.4.8 Predictors of Poor Prognosis and Long-­ Term Mortality on CMR RVEF, LV involvement or biventricular involvement, LGE

24.4.9 Complications Arrythmias, blood stasis and formation of thrombi, heart failure, sudden cardiac death

24.4.10 Management The management goals include symptom reduction, delay of disease progression and prevention of SCD.  The various therapies include exercise restriction, beta blockers, antiarrhythmic drugs, ICD implantation and radiofrequency ablation of ventricular arrhythmias. ARVC patients with previous sustained ventricular arrhythmias undergo ICD placement.

24.4.11 Role of CMR 1. In patients without prior ventricular arrhythmia, CMR can help in risk stratification. In one of the study [4], it was shown that abnormal CMR is an independent predictor of arrhythmic events. 2. CMR with LGE is useful in characterizing the presence and distribution of ventricular scar and helps in planning endocardial ablation.

24.4.12 Controversies and Future Perspectives 24.4.12.1 Controversies Related to Revised Task Force Criteria The various cut off used are derived from MESA study that used FGRE sequence and the US ARVC study used cine SSFP sequence [2]. It was observed that RV volumes on SSFP sequence are at least 10% greater than FGRE. Also, in

M. Verma and V. Ojha

MESA study the average age of subjects was 60 years and subjects in Task force study were 20–30  years younger. It was suggested in various studies that RV EDV decrease 4% per decade [2]. So further developments are needed to improve the reproducibility of RV quantification by CMR. LGE is not a criterion in modified task force criteria as its detection is hampered by thin RV wall and also it is difficult to differentiate fat from fibrosis. There is need for improvement of methods to determine LGE in thin RV wall.

24.4.13 Strain Imaging It was shown in one of the study that in patients with ARVC, regional RV wall strain can predict arrhythmogenic ventricular tachycardia substrate and is better compared to LGE. It can increase the diagnostic accuracy of CMR and can help in planning of ablation procedures [5]. Multiple Choice Questions Question 1: The prevalence of ARVC is (a) 1 in 10 (b) 1 in 100 (c) 1 in 1000 (d) 1 in 10,000 Question 2: ARVC is seen in what percentage of patients with SCD less than 35 years (a) 20% (b) 30% (c) 40% (d) 50% Question 3: Fibrofatty infiltration starts from which cardiac layer (a) Subendocardium (b) myocardium (c) epicardial (d) apex Question 4: Triangle of dysplasia is formed by (a) Inflow, lateral wall, apex (b) Outflow, lateral wall, apex (c) Inflow, outflow and apex (d) Inflow, outflow, and lateral wall Questions 5: The most common site for LV involvement in arrythmogenic ventricular cardiomyopathy is (a) Inferior wall (b) Anterior wall (c) Apex (d) Lateral wall Questions 6: ARVC is mainly inherited as (a) AR (b) AD (c) Both AR and AD (d) Not inherited

24  Cardiac MR Imaging in Arrhythmogenic Ventricular Cardiomyopathy

Question 7: Mutations in ARVC occur against (a) Troponin (b) Myosin (c) Desmosome (d) Actin Question 8: Which of the following is not a part of major criteria according to modified Task Force criteria on CMR? (a) RV akinesia (b) RV hypokinesia (c) RV dyssynchrony (d) RV aneurysms Question 9: What is the cut-off for EF according to modified Task Force criteria? (a) 30% (b) 40% (c) 50% (d) 60% Question 10: LV LGE seen in what percentage of patients with ARVC? (a) 41% (b) 51% (c) 61% (d) 71% Answers 1 c

2 a

3 c

4 c

5 d

6 b

7 c

8 d

9 b

10 c

393

References 1. Basso C, Corrado D, Marcus FI, Nava A, Thiene G.  Arrhythmogenic right ventricular cardiomyopathy. Lancet. 2009;373(9671):1289–300. 2. te Riele AS, Tandri H, Bluemke DA. Arrhythmogenic right ventricular cardiomyopathy (ARVC): cardiovascular magnetic resonance update. J Cardiovasc Magn Reson. 2014;16(1):50. 3. Sen-Chowdhry S, Syrris P, Ward D, Asimaki A, Sevdalis E, McKenna WJ. Clinical and genetic characterization of families with arrhythmogenic right ventricular dysplasia/cardiomyopathy provides novel insights into patterns of disease expression. Circulation. 2007;115(13):1710–20. 4. Deac M, Alpendurada F, Fanaie F, Vimal R, Carpenter JP, Dawson A, Miller C, Roussin I, di Pietro E, Ismail TF, Roughton M, Wong J, Dawson D, Till JA, Sheppard MN, Mohiaddin RH, Kilner PJ, Pennell DJ, Prasad SK. Prognosticvalue of cardiovascular magnetic resonance in patients with suspected arrhythmogenic right ventricular cardiomyopathy. Int J Cardiol. 2013;168:3514–21. 5. Zghaib T, Ghasabeh MA, Assis FR, Chrispin J, Keramati A, Misra S, et al. Regional strain by CMR improves detection of right ventricular scar compared to late-gadolinium enhancement on a multi-modality scar evaluation in patients with arrhythmogenic right ventricular cardiomyopathy. Circ Cardiovasc Imaging. 2018;11(9):e007546.

Cardiac MR Imaging in Hypertrophic Cardiomyopathy

25

Rishabh Khurana

25.1 Case 25.1

25.1.5 What Investigation Will You Do Next?

25.1.1 Clinical Presentation

See Fig. 25.1. CMR will provide a comprehensive evaluation of the disease, including morphologic and functional evaluation. It will also assess for right ventricular involvement, mitral and papillary muscle abnormalities, flow and gradient assessment, and the presence of scar tissue (fibrosis).

A 35-year-old normotensive man presented with dyspnea on exertion (NYHA-II) for 1-year duration with recent worsening. Family history was negative for any cardiovascular disease. There was no history of syncope. Patient was hemodynamically stable. Chest X-ray, ECG, and cardiac biomarkers were within normal limits.

25.1.2 Echocardiography Basal septal hypertrophy with LVO obstruction, Pressure gradient ~24 mmHg (at rest)

25.1.3 Provisional Diagnosis Asymmetric Hypertrophic cardiomyopathy

25.1.4 Differential Diagnosis

25.1.6 Final Diagnosis Asymmetric HOCM SAM with dynamic LVO obstruction with LGE Normal biventricular function

25.1.7 Note 25.1.7.1 Isolated Basal Septal Hypertrophy Focal hypertrophy of the basal inter-ventricular septum can be seen in approximately 10% of cardiac patients without HCM. It is more common in the elderly and hypertensives. It occurs due to susceptibility of basal septum to hypertrophy due to pressure overload from hypertension. However, further studies are needed to understand its clinical relevance.

Isolated basal septal hypertrophy

R. Khurana (*) Department of Cardio Vascular Radiology and Endovascular Interventions, AIIMS, Ansari Nagar, New Delhi, India © The Author(s), under exclusive license to Springer Nature Singapore Pte Ltd. 2023 S. Sharma (ed.), Case-based Atlas of Cardiac Imaging, https://doi.org/10.1007/978-981-99-5620-3_25

395

396

R. Khurana

b

a

c

f

e

h

l

p

d

i

g

j

k

n

m

q

Fig. 25.1 (a–d) (Cine frames): There is asymmetric LVH. Maximum wall thickness is 44  mm. RVH present with maximum thickness of 8  mm. Biventricular ejection fraction [LVEF:75%; RVEF: 68%]. Myocardial mass was elevated 354 grams. (e) (T1-TSE): No abnormal signal intensity. (f, g) (T2-TSE SA and 4Ch): Few areas of hyperintensity in LV myocardium. (h) (3 Chamber LVO Cine): SAM present with e/o dynamic LVO obstruction. (i) (Flow analysis across LVO): Pressure

r

o

s

gradient was 20 mmHg. (j-k) (Mapping): Focal areas of increased T1 & T2 mapping values (upto 1150 ms and upto 70 ms, respectively. (l–o) (Dynamic post contrast): No resting perfusion deficit. (p–s) (LGE): Multiple patchy areas of LGE seen in LV myocardium. Scar quantification: 4% of total LV myocardium. Other findings: No pericardial effusion. Normal atria. No Thrombus. MR+. No TR. No pleural effusion. No mediastinal lymphadenopathy

25  Cardiac MR Imaging in Hypertrophic Cardiomyopathy

397

25.2 Case 25.2

25.2.2 Provisional Diagnosis

25.2.1 Clinical Presentation

Concentric HCM

A 35-year-old hypertensive man presented with dyspnea on exertion (NYHA-II) of 1-year duration with recent worsening. His blood pressure was 160/96 mmHg, not on treatment. Family history is positive for sudden cardiac death. Chest radiograph revealed mild cardiomegaly (LV configuration). ECG revealed features of LV hypertrophy. Cardiac biomarkers were normal. Echo revealed concentric LVH

25.2.3 Differential Diagnosis LVH secondary to hypertension

25.2.4 What Investigation Will You Do Next? CMR (Fig. 25.2).

a

b

c

e

f

h

l

d

g

i

m

j

n

Fig. 25.2 (a–d) (Cine frames): There is presence of concentric LVH.  Maximum wall thickness is 30  mm in the mid cavity-infero-­ septal region. RVH is also noted with maximum RV free wall thickness  ~  7  mm. Biventricular ejection fraction [LVEF: 55%; RVEF: 50%]. Myocardial mass was 542 grams. There was no RWMA/SAM/ LVOTO. (e, f) (T1-TSE): Normal signal intensity. (g) (T2-TSE SA):

k

o

p

Normal signal intensity. (h–k) (Dynamic post contrast): No resting perfusion deficit. (l–p) (LGE): Multiple punctate areas of LGE seen in LV myocardium. Other findings: No pericardial effusion. Normal atria. No Thrombus. No MR/TR.  No pleural effusion. No mediastinal lymphadenopathy

398

R. Khurana

25.2.5 Final Diagnosis

25.3.2 Provisional Diagnosis

Concentric Symmetric HCM with biventricular involvement No SAM/LVOT obstruction. LGE + Normal biventricular function

LVH secondary to hypertension

25.3 Case 25.3

Concentric HCM

25.3.1 Clinical Presentation A 35-year-old hypertensive man presented with dyspnea on exertion (NYHA-II) of 1-year duration with recent worsening. His blood pressure was 170/100  mmHg, not on treatment. Family history was negative for any cardiovascular disease. Clinical examination was unremarkable. Chest radiograph revealed mild cardiomegaly (LV configuration). ECG revealed features of LV hypertrophy. Cardiac biomarkers were normal. Echo revealed concentric LVH

25.3.3 Differential Diagnosis

25.3.4 What Investigation Will You Do Next? CMR (Fig. 25.3).

25.3.5 Final Diagnosis Concentric HCM/LVH secondary to hypertension How will you distinguish between LVH secondary to HTN and concentric HCM? (Table 25.1)

25  Cardiac MR Imaging in Hypertrophic Cardiomyopathy

a

b

e

d

g

h

j

k

n

c

f

i

399

l

o

m

p

Fig. 25.3 (a–d) (Cine frames): There is presence of concentric LVH.  Maximum wall thickness is 16  mm in the mid cavity-infero-­ septal region. Biventricular ejection fraction normal [LVEF: 55%; RVEF: 52%]. Myocardial mass was elevated 164 grams. (e, f) (T2-TSE SA and 4Ch): No abnormal signal intensity. (g, h) (T1-TSE 4Ch and

q

r

SA): No abnormal signal intensity. (i, j) (Mapping): Within normal limits. (k–m) (Dynamic post contrast): No resting perfusion deficit. (n–r) (LGE): Focal LGE seen in superior and inferior RV insertion sites. Other findings: No pericardial effusion. Normal atria. No Thrombus. MR+. No TR. No pleural effusion. No mediastinal lymphadenopathy

400

R. Khurana

Table 25.1  Distinguish between LVH secondary to HTN and concentric HCM Feature History of hypertension Hypertrophy pattern

LVEF LV cavity size LV wall stress Systolic strain

Systolic anterior motion (SAM) mitral valve LGE, elevated T1 mapping Aggressive anti-­ hypertensive therapy

HCM

Almost always asymmetric Concentric HCM: Dec size of LV cavity Supernormal Decrease Reduced LV wall stress Decreased longitudinal systolic strain One-third cases

Hypertensive heart disease + Concentric WT: Rarely exceeds 16 mm Normal/reduced EF Increase Increased LV wall stress Reduced anteroseptal systolic strain Rare

and easy fatiguability for the past 2 weeks. The symptoms had increased over the past 1 day. Family history is negative for any cardiovascular disease. Chest radiograph revealed mild cardiomegaly. Trop T was borderline elevated. Echo revealed pericardial effusion, thickened septum, and lateral LV wall in mid-cavity region with thinned apical LV. Suboptimal evaluation of apex. Mild LV systolic dysfunction.

25.4.2 Provisional Diagnosis Mid ventricular HCM/Burnt-out HCM

25.4.3 What Investigation Will You Do Next? Cardiac MRI (Fig. 25.4).

Favors HCM Persists

Regression

25.4 Case 25.4 25.4.1 Clinical Presentation A 45-year-old male presented with complains of intermittent chest pain, dyspnea on exertion, swelling of bilateral limbs,

25.4.4 Final Diagnosis Mid ventricular HCM with mid ventricular obstruction with global thinning of apical segments to apex with apical aneurysm and thrombus with LGE.  Moderate pericardial effusion. Mild biventricular systolic dysfunction

25  Cardiac MR Imaging in Hypertrophic Cardiomyopathy

a

b

c

f

e

g

k

j

h

p

s

r

v

i

m

o

q

d

l

n

u

401

w

Fig. 25.4 (a–d) (Cine frames): Asymmetric LVH with greater involvement of the mid cavitary region as compared to the basal segments. Maximum wall thickness is 3 cm in the mid cavity-infero-septal region. There is also mid cavitary to apical RVH. Maximum RV free wall thickness: 13 mm. Biventricular ejection fraction [LVEF:40%; RVEF: 42%]. Myocardial mass was elevated 164 grams. (e–i) (end diastolic frames from basal to apical LV): Mid ventricular LV thickening (f), with normal basal LV wall thickness (e), with concentric thinned apical wall (g–i) with LV thrombus. LV apex is aneurysmal. Apical segments and apex were also dyskinetic. (j–k) (T2-TSE SA and 4Ch): No abnormal

t

x

y

z

signal intensity. (l, m) (T1-TSE): No abnormal signal intensity. (n, o) (Mapping): Focal areas of increased T1 & T2 mapping values (upto 1130  ms and upto 65  ms, respectively in mid LV myocardium. (p) (Flow analysis across mid ventricle): Pressure gradient was 20 mmHg. (q–t) (Dynamic post contrast): Resting perfusion deficit along apical LV segments and apex. (u–z) (LGE): Heterogeneous LGE (Subendocardial to mid-myocardial) is noted in the entire hypertrophied myocardium. Transmural LGE is noted in apical segments to apex. Other findings: Moderate pericardial effusion. Normal atria. MR+. No TR. No pleural effusion. No mediastinal lymphadenopathy

402

R. Khurana

25.5 Case 25.5

25.5.2 Provisional Diagnosis

25.5.1 Clinical Presentation

Mid ventricular HCM/Burnt-out HCM

A 30-year-old male presented to emergency with chest pain and dyspnea on exertion for the past 2 h. Family history is negative for any cardiovascular disease, and the past medical history is unremarkable. Chest Radiograph and cardiac biomarkers were normal. ECG revealed giant T-wave inversion in the precordial lead. Echo revealed normal LV ejection fraction, Suboptimal evaluation of LV apex—appeared thickened

25.5.3 What Investigation Will You Do Next?

a

b

25.5.4 Final Diagnosis Apical HCM. No LGE. Normal biventricular function.

c

f

e

d

g

j

i

l

Cardiac MRI (Fig. 25.5).

m

Fig. 25.5 (a–d) (Cine frames): There is thickening of all the apical LV segments and the LV apex (maximum thickness-22 mm) causing spade-­ like configuration of apical region of LV cavity. RV wall thickness: normal. Biventricular ejection fraction [LVEF:63%; RVEF: 65%].

h

k

n

o

Myocardial mass was elevated 110 grams. (e, f) (T2-TSE): No abnormal signal intensity. (g, h) (T1-TSE SA and 4Ch): Few areas of hyperintensity in LV myocardium. (i–l) (Dynamic post contrast): No resting perfusion deficit. (m–o) (LGE): No LGE seen in LV myocardium

25  Cardiac MR Imaging in Hypertrophic Cardiomyopathy

403

25.6 Case 25.6

25.6.2 Final Diagnosis

25.6.1 Clinical Presentation

Apical HCM with fibrosis. No aneurysm. Normal biventricular function.

A 30-year-old asymptomatic man underwent echo for screening for hypertrophic cardiomyopathy as his father has been recently diagnosed with asymmetric HCM. Echo showed thickening of LV apex. He was referred for cardiac MRI for morphologic and functional evaluation (Fig. 25.6).

a

b

e

h

25.6.3 Screening of Relatives of Patients Diagnosed with HCM • It should be done for all first-degree relatives.

c

d

f

i

g

j

Fig. 25.6 (a–d) (Cine frames): Diffuse thickening of LV walls; asymmetrically greater at the apex, with maximum thickness of 22  mm. Myocardial hypertrophy extends to the mid-ventricular segments. RV wall thickness is normal. Biventricular ejection fraction [LVEF: 75%; RVEF: 68%]. Myocardial mass: 160 g. (e, f) (T1-TSE): No abnormal

k

l

signal intensity. (g) (T2-TSE SA): No abnormal signal intensity. (h–l) (LGE): Diffuse and patchy LGE involving apical LV and apex. Other findings: No pericardial effusion. Normal atria. No Thrombus. No MR/ TR. No pleural effusion. No mediastinal lymphadenopathy

404

R. Khurana

• Screening should begin at onset of adolescence (12– 18 years)—repeat imaging performed annually, and then every 5 years until the fourth decade of life. • Trans-thoracic echo is the preferred initial modality for screening • If detected: Close surveillance with serial CMR is suggested. • CMR: increasing role (as it is more precise), benchmark for future studies for potential progression, myocardial crypts, T1 mapping, LGE (fibrosis).

region. There is suspicion of noncompaction in along lateral LV apical wall.

25.7 Case 25.7

Cardiac MRI (Fig. 25.7).

25.7.1 Clinical Presentation A 40-year-old female presented with dyspnea on exertion (NYHA-II) of 1-year duration. Family history is negative for any cardiovascular disease. Clinical examination was unremarkable. Chest radiograph, ECG, cardiac biomarkers were normal. Echo revealed septal hypertrophy more in mid ventricular

25.7.2 Provisional Diagnosis Asymmetric Hypertrophic cardiomyopathy with? focal LVNC

25.7.3 What Investigation Will You Do Next?

25.7.4 Final Diagnosis Nonobstructive Asymmetric HCM (mid-cavity region) with component of noncompaction along lateral apical LV. LGE+. Normal biventricular function.

25  Cardiac MR Imaging in Hypertrophic Cardiomyopathy

a

b

e

f

g

o

c

h

k

405

l

p

Fig. 25.7 (a–d) (Cine frames): Asymmetric anteroseptal thickening (maximum 24 mm) in mid-cavity and apex region is seen. RV wall: No thickening. Biventricular ejection fraction [LVEF: 70%; RVEF: 65%]. Myocardial mass 140  g. (e, f) (Cine frames): Non compaction seen along lateral apical LV. The thickness of non-compacted to compacted myocardium is >2.3. (g, h) (T2-TSE): No abnormal signal intensity. (i,

i

d

j

m

q

n

r

j) (T1-TSE): No abnormal signal intensity. (k–n) (Dynamic post contrast): No resting perfusion deficit. (o–r) (LGE): Linear strands of LGE are seen in LV myocardium mid-cavity, apical region, and apex. Other findings: No pericardial effusion. Normal atria. No Thrombus. No MR/ TR. No pleural effusion. No mediastinal lymphadenopathy

R. Khurana

406 Table 25.2  Differences between hypertrophic cardiomyopathy versus athletic heart Hypertrophy pattern

LGE, elevated T1 mapping ECV Athletic deconditioning (3 months)

HCM Focal Concentric HCM: Dec size of LV cavity Favors HCM Increased ECV Hypertrophy persists

Athletic heart Concentric with chamber enlargement (LV cavity >55 mm)

Wall thickness regresses >2 mm

(a) Myofibrillary disarray. (b) Dysfunction of the coronary microvasculature, with increased wall thickening leading to luminal narrowing, silent myocardial ischemia, and myocardial injury and fibrosis. (c) Increase in interstitial connective tissue. (d) Mitral valve abnormalities, including aberrant papillary muscles, abnormal papillary muscle insertion into the anterior mitral leaflet, and enlargement and elongation of mitral leaflets, are extremely common in HCM and are responsible for the creation of a pathologic dynamic LVO gradient.

25.8 Case 25.8

25.9.4 Natural History [1]

25.8.1 Clinical Presentation

• Phenotypic manifestation of HCM is usually seen in adolescents (pubertal growth spurt). • HCM manifesting in infancy, early childhood, and midlife have also been recognized. • With the availability of various genotyping assays for detection of sarcomeric protein mutation, asymptomatic genotype-positive but phenotype-negative individuals have been recognized in the family members of index case. Clinical as well as imaging assessment plays a crucial role for long-term monitoring. • Most common arrhythmic complication in HCM is atrial fibrillation (affects approximately one-fourth HCM patients). • The most worrisome complication of HCM is sudden cardiac death. • Progressive cardiac remodeling with increasing fibrosis is observed in majority of the patients. However, the rate of progression varies.

A 25-year-old male was playing soccer at his annual college sports event. He suddenly felt dizzy and collapsed. He was rushed to the emergency. ECG showed ventricular tachycardia. Cardiopulmonary resuscitation was carried out. However, the resuscitation measures failed and patient was declared dead. Autopsy was carried out. There was presence of asymmetrical septal hypertrophy involving basal septum (Maximum thickness was 35 mm). Histopathology revealed myocardial disarray with presence of extensive fibrosis involving the hypertrophied septum. Diagnosis of Sudden Cardiac Death due to Hypertrophic Cardiomyopathy was established. His firstdegree relatives were referred to screening program for HCM. Hypertrophic Cardiomyopathy vs. Athletic Heart—Table 25.2

25.9 Discussion 25.9.1 Introduction

25.9.5 Morphological Pattern: HCM Phenotypes

Hypertrophic cardiomyopathy is defined as abnormal LV myocardial thickening without associated chamber dilata- The most common LV segment to be undergo hypertrophy is tion, and in the absence of any other systemic disease-­ the interventricular septum. The various morphological patcausing excess afterload [1]. terns of HCM phenotype have been tabulated in Table 25.3.

25.9.2 Etiopathogenesis

25.9.6 Diagnostic Criteria [1]

• Mutation in sarcomere proteins. • These have autosomal-dominant pattern inheritance with variable penetrance [2, 3].

• The normal end-diastolic LV wall thickness is 11 mm or less. • Asymmetric septal hypertrophy: When septal thickness  ≥  15  mm or when the ratio of septal thickness to thickness of inferior wall at the midventricular level > 1.5 • However, in children, z scores are calculated. Septal hypertrophy: wall thickness ≥ 2 S.D. above the mean for age, sex, or body size (i.e., z score ≥ 2).

25.9.3 Histopathology Structural abnormalities include

25  Cardiac MR Imaging in Hypertrophic Cardiomyopathy

407

Table 25.3  Different types of HCM phenotypes HCM phenotype Preclinical Asymmetric (focal or diffuse)

Description Crypts: Blind pit or V-shaped fissure in myocardium Most common HCM (60–70%)

Focal or diffuse septal involvement/other segment SAM, mitral regurgitation (eccentric and posteriorly directed) Dynamic LVO obstruction with increased gradient Concentric

Mid ventricle

Apical

Mass like Burned-out (late stage HCM)

Diastolic dysfunction in HCM [4] Apical aneurysm formation in HCM [5] RV involvement in HCM Mass vs. mass-like HCM [1]

Crypts [6]

Second most common HCM, diffuse symmetric LV thickening DD: HTN, athlete heart, infiltrative heart disease (amyloidosis, sarcoidosis) Rare, hourglass or dumbbell shape of LV, apical aneurysm formation, apical thrombus

Dynamic obstruction Not seen Across LVO It is seen in approximately 30% cases (at rest) Provocative maneuvers, e.g., Valsalva may demonstrate in approximately 30–40% cases Can be seen

Obstruction and increased gradient at midventricular level Not seen

LGE Absent Patchy and nodular at RV insertion points or mid myocardial

Patchy and nodular at RV insertion points or mid myocardial Seen in a thinned-out apex

Spade-like configuration of LV, burned-out apex with apical Seen in a thinned-out aneurysm formation apex with nonenhancing thrombus if One-third cases have MI/stroke/atrial fib. present DD: EMF May involve septum or lateral wall Can be seen Patchy and nodular mid myocardial Important to differentiate from mass Progressive heart failure Disappearance of LVOT Frequent with HCM gradient pattern, additionally due LV thinning and dilatation with hypokinesia and depressed to development of ejection fraction ( 30 mm) –– Gradient across LVOT >30 mmHg –– LGE extent >15 percent of total LV myocardial mass –– Burned-out phase (LVEF  0.86.

25.9.10 Management Strategies/How Imaging Impacts Management 25.9.10.1 Postprocedure Imaging • Cardiac MR imaging is the preferred modality to detect and quantify infarct size after septal ablation procedure. • Aim of posttreatment imaging: –– Identification septal infarct (seen as a focal area of LGE, within 3 months of ablation) –– Quantifying septal thickness –– Assess LVOT gradient –– Estimate ejection fraction. • Successful ablation procedure: symptom relief –– Decrease in septal thickness –– Decreased LVOT gradients –– Decreased left atrial dimensions –– Decreased mitral regurgitation. 25.9.10.2 Controversies • Estimation of gradient across dynamic obstruction using flow analysis on MRI 25.9.10.3 Future Perspective • MR spectroscopy: Phosphocreatine (PCr)/adenosine triphosphate (ATP) ratio significantly reduced in hypertrophied myocardium (HCM patients vs. controls) • Preclinical and overt HCM detection using abnormal strain parameters • 4D flow to assess gradient across dynamic obstruction • Diffusion Tensor Imaging to identify myocardial disarray. • Dispersion Mapping

25  Cardiac MR Imaging in Hypertrophic Cardiomyopathy

Multiple Choice Questions Question 1: Which of the most common pattern of inheritance of sarcomere mutations in HCM? (a) Autosomal dominant with variable penetrance (b) Autosomal recessive (c) X-linked recessive (d) X-linked dominant Question 2: What out of the following is the most commonly involved sarcomeric mutation in HCM? (a) MYBPC3 (myosin-binding protein), chromosome 11 (b) MYH7 (myosin heavy chain), chromosome 14 (c) TNNT2 (cardiac muscle troponin), chromosome 1 (d) TNNI3 (troponin I type 3), chromosome 19 Question 3: Which out of the following histopathological finding is not observed in HCM cases? (a) Myocardial disarray (b) Silent myocardial ischemic changes (c) Myocardial fibrosis (d) Viral inclusion bodies (e) Papillary muscle abnormalities Question 4: What is the most common age group for manifestation of HCM phenotype? (a) Infant (b) Early childhood (c) Elderly (d) Middle age (e) Adolescent (Pubertal growth spurt) Question 5: Which of the following are the class I indicators for predicting sudden cardiac death risk? (a) Prior cardiac arrest (b) Spontaneous sustained ventricular tachycardia (c) Both a and b (d) Dynamic LVO obstruction Question 6: Which out of the following is a marker of sudden cardiac death in a case of hypertrophic cardiomyopathy? (a) Maximum LV ventricular wall thickness 25 mm (b) Gradient across LVOT 15 mmHg (c) LGE extent 7% of total LV myocardial mass (d) LVEF 65% (e) Global dispersion score = 0.9 Question 7: What percent of cases of asymmetric septal HCM have an associated dynamic outflow obstruction? (a) Up to 70% (b) 5% (c) All cases (d) 10%

409

(e) 95% Question 8: What gradient will classify HCM into hypertrophic obstructive cardiomyopathy? (a) 10 mmHg (b) 20 mmHg (c) >30 mmHg (d) 25 mmHg Question 9: What percentage of cases of HCM may have a concomitant RV involvement? (a) 17–25% (b) 50–60% (c) 80–90% (d) All cases Question 10: Which of the following is true with respect to screening for HCM? (a) Done for all first-degree relatives. (b) Screening should begin at 12–18  years (onset of adolescence). (c) Repeat imaging is performed annually, and then every 5 years until the fourth decade of life. (d) Trans-thoracic echo is the preferred initial modality for screening. (e) All of the above. Answers 1 a

2 a

3 d

4 e

5 c

6 e

7 a

8 c

9 a

10 e

References 1. Baxi AJ, Restrepo CS, Vargas D, et al. Hypertrophic cardiomyopathy from A to Z: genetics, pathophysiology, imaging, and management. Radiographics. 2016;36(2):335–54. 2. Ho CY, Charron P, Richard P, Girolami F, Van Spaendonck-Zwarts KY, Pinto Y.  Genetic advances in sarcomeric cardiomyopathies: state of the art. Cardiovasc Res. 2015;105(4):397–408. 3. Ingles J, Sarina T, Yeates L, et  al. Clinical predictors of genetic testing outcomes in hypertrophic cardiomyopathy. Genet Med. 2013;15(12):972–7. 4. Pasipoularides A.  LV twisting and untwisting in HCM: ejection begets filling—diastolic functional aspects of HCM.  Am Heart J. 2011;162(5):798–810. 5. Maron MS, Finley JJ, Bos JM, et  al. Prevalence, clinical significance, and natural history of left ventricular apical aneurysms in hypertrophic cardiomyopathy. Circulation. 2008;118(15):1541–9. 6. Rowin EJ, Maron MS.  Myocardial crypts in hypertrophic cardiomyopathy: the new gang in town. Eur Heart J Cardiovasc Imaging. 2012;13(4):281–3.

Imaging in Cardiac Amyloidosis

26

Mansi Verma and Vineeta Ojha

26.1 Case 26.1

26.1.5 What Investigation Will You Do Next?

26.1.1 Clinical Presentation

CMR will help in diagnosis. It will provide morphologic information, functional information and will help in tissue characterization. CMR was done (Fig. 26.1).

Forty-two-year-old man; a known case of multiple myeloma, gradually progressive exercise intolerance (NYHA III) and occasional palpitations for 6 months

26.1.2 Electrocardiography Low voltage QRS complexes

26.1.6 CMR See Fig. 26.1.

26.1.7 Final Diagnosis

26.1.3 Echocardiography

Cardiac Amyloidosis (likely AL type)

Systolic dysfunction (LVEF = 35%) with thickening of LV wall, which showed speckled pattern

26.1.8 Differential Diagnosis

26.1.4 The Differential Diagnosis Will Include 1. Cardiac amyloidosis 2. Concentric hypertrophic cardiomyopathy

See Table 26.1. 1. Concentric hypertrophic cardiomyopathy 2. Hypertensive heart disease 3. Other infiltrative heart diseases as Anderson Fabry disease

M. Verma (*) · V. Ojha Department of Cardio Vascular Radiology and Endovascular Interventions, AIIMS, Ansari Nagar, New Delhi, India © The Author(s), under exclusive license to Springer Nature Singapore Pte Ltd. 2023 S. Sharma (ed.), Case-based Atlas of Cardiac Imaging, https://doi.org/10.1007/978-981-99-5620-3_26

411

M. Verma and V. Ojha

412

a

b

c

e

f

i

j

m

n

Fig. 26.1  T1 TSE (a): Thickened LV (18 mm) and RV (11 mm) myocardium with thickening of interatrial septum; T2-TSE (b): No hyperintense signal in the myocardium; 4 Ch (ES: c, ED: d) and SA (ES: e, ED: f) Cine frame: Reduced LVEF: 31% with biventricular hypokinesia. RVEF was 34%. Posterior wall of RA is thickened (8 mm); T1 (g) and T2 (h) Mapping: Elevated T1 (1189ms) values with normal T2 mapping

d

g

k

h

l

o

p

(49 ms) values; Perfusion (i): Diffuse subendocardial perfusion deficit; T1 scout at 172 ms (j), 267 ms (k) and 290 ms (l) Reversal of nulling with myocardial nulling earlier than blood pool nulling; Short axis (SA) base (m), SA mid cavity (n) and SA apex (o) LGE 4ch (p): Diffuse subendocardial LGE noted in both ventricles, septum and wall of RA

Table 26.1  Differential diagnosis for cardiac amyloidosis Concentric hypertrophic cardiomyopathy Hypertensive heart disease

Anderson Fabry disease

Imaging findings Ventricular wall hypertrophy with normal or increased ejection fraction and normal chamber volumes. LGE is seen in hypertrophied and non-hypertrophied segments in mid myocardium or RV insertion points. Concentric LVH with preserved ejection fraction, increased chamber volumes, and occasional fibrosis. LGE usually seen in mid myocardial distribution in basal septum or inferolateral wall. No nulling abnormality/ perfusion deficit. Usually concentric LV thickening with LGE (50%) in basal inferolateral wall in mid myocardial or subepicardial distribution. Native T1 values are low due to lipid deposition.

26  Imaging in Cardiac Amyloidosis

413

26.2 Case 26.2

led pattern of LV. MRI was done to rule out cardiac amyloidosis (Fig. 26.2).

26.2.1 Clinical Presentation

26.2.2 Final Diagnosis

Fifty-four-year-old man; gradually progressive shortness of breath NYHA grade II.  Holter revealed NSVT and ECHO suggested LVH with thickened interatrial septum and speck-

a

b

e

f

i

j

m

n

Fig. 26.2  T1 TSE (a): Thickened LV myocardium; T2-TSE (b): Normal signal intensity; 4 Ch (ES: c, ED: d) and SA (ES: e, ED: f) Cine frame: Reduced LVEF: 37% with no RWMA.  RVEF was 30%. Interatrial septum is thickened; T1 (g) and T2 (h) Mapping: Elevated T1 (1202  ms) and T2 mapping (53  ms) values. T1 scout at 77  ms (i),

Cardiac amyloidosis with diffuse involvement and pleural effusion

c

c

g

k

h

l

o

p

195  ms (j) and 290 (k) Reversal of nulling with myocardial nulling earlier than blood pool nulling; LGE 4ch (l), SA base (m), SA mid cavity (n) and SA apex (o): Diffuse subendocardial to transmural LGE; TRUFI (p): Bilateral pleural effusion with pericardial effusion

414

M. Verma and V. Ojha

26.3 Case 26.3

led pattern of LV. MRI was done to rule out cardiac amyloidosis (Fig. 26.3).

26.3.1 Clinical Presentation Fifty-four-year-old man; gradually progressive shortness of breath NYHA grade II.  Holter revealed NSVT and ECHO suggested LVH with thickened interatrial septum and speck-

a

e

b

m

j

n

Cardiac amyloidosis with atrial thrombi

c

f

i

26.3.2 Final Diagnosis

o

Fig. 26.3  4 Ch (a) and SA T2 TSE (b): Thickened LV myocardium. No hyperintense signal in the myocardium; 4 Ch (ES: c, ED: d) and SA (ES: e, ED: f) Cine frame: Reduced LVEF: 33% with global biventricular hypokinesia and mild biatrial dilatation. RVEF was 14%. Interatrial septum is normal; T1 (g) and T2 (h) Mapping: Elevated T1 (1204 ms) and T2 mapping (59 ms) values; T1 scout (i, j): Reversal of nulling with

d

g

h

k

l

p

myocardial nulling earlier than blood pool nulling; Magnitude images (k, l): Thrombi in RA and LA; LGE SA base (m), SA mid cavity (n) and SA apex (o): Diffuse subendocardial to transmural enhancement seen in LV and RV; TRUFISP (p): Bilateral pleural effusion with pericardial effusion

26  Imaging in Cardiac Amyloidosis

26.4 Discussion Cardiac amyloidosis is characterized by deposition of amyloid fibrils in the extracellular space of the heart. It is a primary interstitial disease independent of cardiomyocyte necrosis but still carries a poor prognosis. Its estimated prevalence is 0.001% [1]. More than 30 precursor proteins can form amyloid, and the difference among them is the basis for classification of this entity. Most of the cases occur either due to immunoglobulin light chain (AL type) or transthyretin (ATTR type). Clinically recognized cardiac involvement occurs in 60% patients in AL variety.

26.4.1 Etiology and Pathophysiology 26.4.1.1 Light Chain (AL) Amyloidosis It occurs due to abnormal clonal proliferation of plasma cells. Such plasma cells clone can be harmless and excrete in urine (monoclonal gammopathy of undetermined significance), can cause multiple myeloma or misfold to form beta pleated sheets that deposits in various organs leading to AL amyloidosis [2]. 26.4.1.2 Transthyretin (ATTR) Amyloidosis Transthyretin is a protein produced by liver and is a transporter of thyroxine and retinol. It usually circulates as tetramer, while the monomer form is prone to misfold and deposits as amyloid. There are two types of ATTR amyloidosis: [2] Wild-type ATTR: normal transthyretin protein. Mutant ATTR: transthyretin gene is mutated causing accelerated amyloid deposition. Amyloid subtypes and clinical characteristics are tabulated in Table 26.2.

415 Table 26.2  Amyloid subtypes and clinical characteristics Subtypes AL type (MC type) Wild type ATTR Mutant ATTR

Demographics M ~ F Age: 40–80 years M >>> F Age 65–95 M >> F Age 55–75

Organ involved Any (heart, kidney, GI, tongue, nerves, liver, soft tissue) Heart (and carpal tunnel syndrome) Heart and nerves (and carpal tunnel syndrome)

26.4.2 Imaging Modalities for Suspected Case of Cardiac Amyloidosis (a) Echocardiography: It is the first imaging test performed in suspected cardiac amyloidosis. The characteristic findings are usually observed in advanced stage and include increased ventricular thickness with frequent involvement of right ventricle, decreased ejection fraction and thickening involving the valves and interatrial septum. (b) Cardiac MRI: CMR is widely used in the assessment of cardiac amyloidosis and is better than echocardiography in detecting early changes.

26.4.2.1 Goals of CMR • Diagnosis of cardiac amyloidosis, which will guide clinical management. • Identify extent and areas of cardiac involvement. • Identify associated complications. • Prognostication and risk stratification. • Assessment of response to treatment and monitoring of disease. CMR provides comprehensive and multidimensional assessment of the cardiac amyloidosis, including morphologic, functional evaluation and tissue characterization in a reproducible and operator-independent manner.

26.4.2.2 CMR Findings in Cardiac Amyloidosis Are Tabulated in Table 26.3 The difference between AL and ATTR amyloid is shown in Fig. 26.4.

416

M. Verma and V. Ojha

26.4.2.3 Predictors of Poor Prognosis and Long-­Term Mortality on CMR LVEF Increased myocardial mass and wall thickness. RV and RA involvement.

ECV correlates with amyloid burden and is an independent prognostic marker in ATTR amyloid [7]. Transmural LGE is a prognostic marker for all-cause mortality [8].

Table 26.3  CMR findings in cardiac amyloidosis Morphology

Function Nulling pattern

LGE T1 map

T2 map ECV

Ventricular hypertrophy with decreased chamber size. Biatrial dilatation (restrictive pattern). Thickening of valves and inter atrial septum. A ratio of myocardial to skeletal muscle signal intensity of 40% is likely to be cardiac amyloidosis and it is an early disease marker.

Features

AL

ATTR

LV MASS

Mildly increased (100 g/m2) Assymetrical LV thickening (maximum in septum)

Thickness of IVS

More

Atrial mass

Larger

Cavity volume

Less

LV ejection fraction

Low

LGE

Less extensive Ofter global

Amyloid deposit

More extensive and diffuse, Transmural, RV Larger

Native T1

More (1104+/–54)

1066 +/ –42)

ECV

Not different, Less (concomitant edema)

>40 %

T2

More (63.2+/–4.7)

56.2+/–3.1

Fig. 26.4  The difference between AL and ATTR amyloid

26  Imaging in Cardiac Amyloidosis Table 26.4  Prognosis and treatment of different types of cardiac amyloidosis Type AL

Wild ATTR

Mutant ATTR

Prognosis Median survival ~6 months From onset of heart failure Median survival ~6 years From onset of heart Failure Dependent on mutation

Treatment Supportive for heart failure Chemotherapy to eliminate abnormal plasma cells Stem cell transplantation in selected patients Supportive for heart failure Pacemaker for advanced AV block Several agents undergoing trials Supportive for heart failure Pacemaker for advanced AV block Several agents undergoing trials

26.4.3 Complications 1. Biventricular dystolic dysfunction. 2. Involvement of atria leads to blood stasis and formation of thrombi. 3. Heart failure.

26.4.4 Management The treatment depends on the type of amyloid infiltration (Table 26.4).

26.4.5 Role of MRI in Determining Response to Treatment T1 mapping (native and ECV) can track changes over time. ECV is the earliest disease marker to track amyloid regression. With successful chemotherapy for AL where there is a complete response and switch-off of clonal light chains, T1, ECV, and LGE can reverse, and the time course for these processes can be tracked. Multiple Choice Questions Question 1: Deposition of amyloid fibrils occurs in which compartment of heart? (a) Intracellular (b) Extracellular (c) Both intracellular and extracellular (d) Intramural Question 2: The estimated prevalence of cardiac amyloidosis is (a) 1% (b) 0.1%

417

(c) 0.01% (d) 0.001% Question 3: Cardiac involvement occurs in what percentage of AL patients? (a) 40% (b) 50% (c) 60% (d) 70% Question 4: Carpal tunnel syndrome is associated with which type of amyloidosis? (a) AL (b) ATTR (both wild and mutant) (c) Only wild ATTR (d) Only mutant ATTR Questions 5: Increased native T2 value is seen in which of following type of amyloidosis? (a) Wild ATTR (b) Mutant ATTR (c) AL (d) Both ATTR and AL Questions 6: Which of the following nulling pattern has highest specificity for cardiac amyloidosis? (a) Type 1 (b) Type 2 (c) Type 3 (d) Type 4 Question 7: Which variety is associated with systolic dysfunction? (a) AL (b) ATTR (c) Both AL and ATTR (d) None Question 8: Diffuse extensive transmural LGE is seen mostly in which type of cardiac amyloidosis? (a) AL (b) ATTR (c) Both AL and ATTR (d) None Question 9: Earliest marker to track amyloid regression is (a) T1 mapping (b) T2 mapping (c) LVEF (d) ECV Question 10: Chemotherapy and stem cell transplantation forms part of treatment of which type of amyloidosis? (a) Mutant ATTR (b) Wild ATTR (c) Both wild and mutant ATTR (d) AL Answers 1 b

2 d

3 c

4 b

5 c

6 d

7 b

8 b

9 d

10 d

418

References 1. Fulton N, Rajiah P.  Utility of magnetic resonance imaging in the evaluation of left ventricular thickening. Insights Imaging. 2017;8(2):279–93. 2. Cardiac amyloidosis. American College of Cardiology. https:// www.acc.org/latest-­i n-­c ardiology/articles/2016/07/07/14/59/ cardiac-­amyloidosis 3. Fontana M, Ćorović A, Scully P, Moon JC. Myocardial amyloidosis: the exemplar interstitial disease. JACC Cardiovasc Imaging. 2019;12(11, Part 2):2345–56. 4. Pandey T, Jambhekar K, Shaikh R, Lensing S, Viswamitra S. Utility of the inversion scout sequence (TI scout) in diagnosing myocardial amyloid infiltration. Int J Cardiovasc Imaging. 2013;29(1):103–12.

M. Verma and V. Ojha 5. Ridouani F, Damy T, Tacher V, Derbel H, Legou F, Sifaoui I, et al. Myocardial native T2 measurement to differentiate light-chain and transthyretin cardiac amyloidosis and assess prognosis. J Cardiovasc Magn Reson. 2018;20(1):58. 6. Baggiano A, Boldrini M, Martinez-Naharro A, Kotecha T, Petrie A, Rezk T, et al. Noncontrast magnetic resonance for the diagnosis of cardiac amyloidosis. JACC Cardiovasc Imaging. 2020;13(1 Part 1):69–80. 7. Martinez-Naharro A, Treibel TA, Abdel-Gadir A, Bulluck H, Zumbo G, Knight DS, et  al. Magnetic resonance in transthyretin cardiac amyloidosis. J Am Coll Cardiol. 2017;70(4):466–77. 8. Fontana M, Pica S, Reant P, Abdel-Gadir A, Treibel TA, Banypersad SM, et  al. Prognostic value of late gadolinium enhancement cardiovascular magnetic resonance in cardiac amyloidosis. ­ Circulation. 2015;132(16):1570–9.

Cardiac MR Imaging in Restrictive Cardiomyopathy

27

Amit Ajit Deshpande

27.1 Case 27.1 27.1.1 Clinical Presentation Forty-nine-year-old woman; complaints with bilateral lower limb edema and gradually progressing dyspnea on exertion for last 5  years. No cyanosis or pallor. Lab investigation showed eosinophillia.

27.1.2 Chest Radiograph See Fig. 27.1. In an acyanotic adult, CXR with massive cardiomegaly, RA enlargement, and normal to reduced lung vascularity, the following differentials are to be considered (Table 27.1).

27.1.3 Echocardiography Dilated RA with very small RV. Severe LV systolic dysfunction with global hypokinesia. Mild TR. No significant apical displacement of tricuspid valve leaflets. Mild pericardial and right pleural effusion. No e/o pericardial thickening.

Fig. 27.1  Massive cardiomegaly with globular heart suggestive of RV enlargement. RA enlargement (increased height of right atrial convexity >50% of total mediastinal cardiovascular height) and RVOT enlargement. Narrow pedicle

27.1.4 Provisional Diagnosis

2. Isolated tricuspid regurgitation 3. Constrictive Pericarditis

Restrictive cardiomyopathy secondary to endomyocardial fibrosis.

27.1.5 What Investigation Will You Do Next?

27.1.4.1 Based on Echo Findings the Differentials to Be Included Are 1. Ebstein’s anomaly

• CMR for –– Etiologic evaluation: identification of underlying cause of restrictive cardiomyopathy.

A. A. Deshpande (*) Department of Cardio Vascular Radiology and Endovascular Interventions, AIIMS, Ansari Nagar, New Delhi, India © The Author(s), under exclusive license to Springer Nature Singapore Pte Ltd. 2023 S. Sharma (ed.), Case-based Atlas of Cardiac Imaging, https://doi.org/10.1007/978-981-99-5620-3_27

419

A. A. Deshpande

420 Table 27.1  Differential diagnosis Pressure overload Volume overload Congenital Cardiomyopathy

Rheumatic heart disease Pericardial effusion

Pulmonary stenosis in failure Pulmonary hypertension Pulmonary regurgitation Tricuspid regurgitation Ebstein’s anomaly Uhl’s anomaly Arrhythmogenic right ventricular dysplasia, DCMP Right ventricular endomyocardial fibrosis RV predominant myocarditis RV predominant sarcoidosis Mitral stenosis with tricuspid regurgitation Mitral stenosis with tricuspid stenosis

–– Functional evaluation: calculation of biventricular systolic and diastolic functions and volumetric evaluation. • Available imaging options include: –– Catheter angiography –– Cardiac CT –– Cardiac MR • Catheter angiography is not used currently, because it is –– Invasive. –– Only anatomic evaluation is possible, functional quantification and tissue characterization is not possible. –– Unable to provide 3D information.

–– Information provided is limited to the lumen and one cannot evaluate associated anomalies. • Cardiac CT is not the investigation of choice for cardiomyopathy, because –– Functional and volumetric evaluation is less reliable as compared to CMR. –– Tissue characterization is not possible. –– Radiation exposure. • Contrast MR is the investigation of choice (Fig.  27.2, Table 27.2).

27.1.6 Final Diagnosis Restrictive Cardiomyopathy secondary to isolated right ventricular endomyocardial fibrosis with obliterated right ventricle.

27.1.7 The Differential Diagnosis Will Include See Table 27.3. 1. Ebstein’s anomaly 2. Isolated Tricuspid regurgitation 3. Constrictive pericarditis

421

27  Cardiac MR Imaging in Restrictive Cardiomyopathy

a

b

c

d

e

f

g

h

i

j

k

Fig. 27.2  Four chamber diastole (a) and systole images (b) show dilated RA and small RV with normally positioned tricuspid valve leaflets (black arrow) with respect to mitral leaflets (red arrow) and tricuspid regurgitation (*) [Apical displacement is  R). Short axis (k) and four chamber (l) LGE images show LV apical thrombus (black arrow), sub-endocardial enhancement (white arrowheads) along right and left ventricular apices with three layer appearance (due to nonenhancing apical thrombus) (Right ventricular three layer appearance is not shown)

Table 27.5  Functional parameters of Case 29.3 Functions EF EDV

LV Absolute 51 (56–78%) 91 (77–195 mL)

Indexed – 63.4 (47–92 mL/m2)

RV Absolute 74 (47–74%) 81.3 (88–227 mL)

Indexed – 56.6 (55–105 mL/m2)

27.4 Case 27.4

27.4.2 Echocardiography

27.4.1 Clinical Presentation

Bi-atrial dilatation with diastolic dysfunction of left ventricle. Severe right ventricular systolic dysfunction with global hypokinesia. Mild left ventricular apical thickening. Tricuspid regurgitation.

Ten -year-old boy presented with progressive dyspnea and exercise intolerance on exertion for a year. No e/o edema, cyanosis, pallor, or icterus. Lab investigations showed eosinophilia.

A. A. Deshpande

426

27.4.3 Provisional Diagnosis

27.4.5 Final Diagnosis

Restrictive cardiomyopathy

Restrictive cardiomyopathy secondary to LV endomyocardial fibrosis with TR and severe RV systolic dysfunction

27.4.4 Cardiac MR See Fig. 27.5 and Table 27.6.

a

b

c

d

e

f

g

h

i

j

k

Fig. 27.5 Chest Radiograph (a) showed bi-atrial enlargement— increased right atrial convexity with widening of carina. No e/o pleural effusion. Normal lung vascularity. Four chamber systolic (b) and diastolic (c) images show impaired relaxation of LV with severe systolic dysfunction of right ventricle. Mild LV apical thickening. TR with RA enlargement. Short axial diastolic (d) and systolic (e) images show severe systolic dysfunction of the right ventricle and moderate systolic

dysfunction of left ventricle. No ventricular thickening noted. Four chamber T1 (f) and T2 (g) images do not show abnormal myocardial intensity. Four chamber (h, i) first-pass perfusion images show perfusion defect in the left ventricular apex (arrow). Two chamber (j) and four chamber (k) LGE images show sub-endocardial enhancement along LV apex (arrows)

Table 27.6  Functional parameters of Case 29.4 Functions EF EDV

LV Absolute 38.91 (56–78%) 52.23 (77–195 mL)

Indexed – 65.67 (47–92 mL/m2)

RV Absolute 16.47 (47–74%) 123.52 (88–227 mL)

Indexed – 155.3 (55–105 mL/m2)

27  Cardiac MR Imaging in Restrictive Cardiomyopathy

427

27.5 Case 27.5

27.5.3 Provisional Diagnosis

27.5.1 Clinical Presentation

RV endomyocardial fibrosis

A 41-year-old man presented with progressive dyspnea and exercise intolerance on exertion for last 5 years and generalized swelling over bilateral legs for last 1 year. No e/o cyanosis, pallor, or icterus. Normal CBC with Eosinophilia.

27.5.4 Differential to Be Considered Ebstein’s anomaly

27.5.2 Echocardiography

27.5.5 Cardiac MR

Massive RA dilatation with near obliteration of RV. Preserved LV systolic function without regional wall motion abnormality. No significant apical displacement of tricuspid leaflets. No e/o mitral or tricuspid regurgitation. No e/o pericardial/ pleural effusion. No evident pericardial thickening noted.

See Fig. 27.6 and Table 27.7.

27.5.6 Final Diagnosis Isolated right ventricular endomyocardial fibrosis with severe RV systolic dysfunction.

a

b

c

d

e

f

g

h

i

j

Fig. 27.6  Four chamber diastolic (a) and systolic (b) images show massively dilated RA with obliterated RV and apical thickening. Preserved LV systolic functions. Short axial diastolic (c) and systolic (d) images show severe systolic dysfunction of the right ventricle and preserved systolic function of LV. No ventricular thickening noted. Two

chamber T2 (e) image does not show abnormal myocardial intensity. Four chamber (f, g) first-pass perfusion images show perfusion defect in RV apex (arrow). Two chamber (h, i) and four chamber (j) LGE images show sub-endocardial enhancement along RV apex (arrows)

A. A. Deshpande

428 Table 27.7  Functional parameters of Case 29.5 Functions EF EDV

LV Absolute 50.86 (56–78%) 102.86 (77–195 mL)

Indexed – 69.28 (47–92 mL/m2)

RV Absolute 17.09 (47–74%) 253.53 (88–227 mL)

Indexed – 170.77 (55–105 mL/m2)

27.6 Case 27.6

27.6.3 Provisional Diagnosis

27.6.1 Clinical Presentation

Restrictive cardiomyopathy

A 22-year-old man presented with progressive dyspnea and exercise intolerance on exertion for last 5 years and generalized swelling over bilateral legs for last 1 year. Lab investigations showed eosinophilia.

27.6.4 Cardiac MR See Fig. 27.7 and Table 27.8.

27.6.2 Echocardiography

27.6.5 Final Diagnosis

RA dilatation with biventricular diastolic dysfunction. Biventricular systolic dysfunction with biventricular hypokinesia. Biventricular apical thickening.

Restrictive cardiomyopathy with biventricular endomyocardial fibrosis with apical thrombi giving trilaminar appearance

27  Cardiac MR Imaging in Restrictive Cardiomyopathy

429

b

a

c

d

e

f

g

h

i

j

k

l

m

n

o

Fig. 27.7  Chest Radiograph (a) did not show any significant abnormality seen. Four chamber systolic (b) and diastolic (c) images show dilated RA with mild obliteration of RV. Biventricular apical thickening. Mildly reduced biventricular systolic functions. Short axial systolic (d) and diastolic (e) images show midly reduced systolic biventricular functions. No ventricular thickening noted. Four chamber T1 (f) and short axis T2 (g) image does not show abnormal myocardial intensity.

Short axis T1 (h) and T2 (i) maps do not show abnormal myocardial native values. Four chamber first-pass perfusion images (j, k) show perfusion defect in biventricular apices (arrow). Short axis, two chamber and four chamber LGE images (l–o) show sub-endocardial enhancement along mid segments of RV and LV (arrows). Biventricular apical thrombi with sub-endocardial enhancement along the apices giving “Tri-laminar appearance”

Table 27.8  Functional parameters of Case 29.6 Functions EF EDV

LV Absolute 35.96 (56–78%) 93.35 (77–195 mL)

Indexed – 69.28 (47–92 mL/m2)

RV Absolute 29.6 (47–74%) 62.19 (88–227 mL)

Indexed – 38.07 (55–105 mL/m2)

430

A. A. Deshpande

27.7 Case 27.7

27.7.3 Provisional Diagnosis

27.7.1 Clinical Presentation

RCM

A 38-year-old man patient presented with rash and eczema over flexural areas for last 1 year. He complained about occasional abdominal pain and diarrhea over last 6–7 months. He also complained about reduced exercise intolerance and easy fatiguability for last 3 months No e/o edema, cyanosis, pallor, or icterus. Blood investigations revealed eosinophilia.

27.7.2 Echocardiography RA enlargement. Biventricular diastolic dysfunction of both ventricles (LV  >  RV) with biventricular mild systolic dysfunction.

27.7.4 Cardiac MR See Fig. 27.8 and Table 27.9.

27.7.5 Final Diagnosis Restrictive cardiomyopathy with LV diastolic dysfunction with obliteration of biventicular apices and sub-endocardial enhancement. Blood eosinophilia. Findings are suggestive of RCM secondary to Loeffler’s Endocarditis.

a

b

c

d

e

f

g

h

i

j

Fig. 27.8  Four chamber diastolic (a) and systolic (b) images show small RV and LV with impaired relaxation of both the ventricles. Bi-atrial dilatation is evident (giving the appearance of tubular heart). Thickened LV apex is evident. Short axial diastolic (c) and systolic (d) images show biventricular systolic dysfunction. Vertical long axis two chamber images (e, f) show thickened LV apex. Four chamber T1 (g)

k and T2 (h) images show do not show abnormal myocardial intensity. Axial TRUFI image (i) shows bi-atrial dilatation with thickened left ventricular apex. Short axis (j) and four chamber (k) LGE images show sub-endocardial enhancement along right and left ventricular apices (arrows)

27  Cardiac MR Imaging in Restrictive Cardiomyopathy

431

Table 27.9  Functional parameters of Case 29.7 Functions EF EDV

LV Absolute 49 (56–78%) 48.80 (77–195 mL)

Indexed – 25.9 (47–92 mL/m2)

RV Absolute 32 (47–74%) 132.5 (88–227 mL)

Indexed – 70.4 (55–105 mL/m2)

27.8 Case 27.8

27.8.3 Provisional Diagnosis

27.8.1 Clinical Presentation

RCM with systolic dysfunction of unknown etiology.

Nine-year-old girl presented with gradually progressing exertional dyspnea and fatigue for last 2 years. No h/o palpitations. No personal/family history.

27.8.4 Cardiac MR See Fig. 27.9 and Table 27.10.

27.8.2 Echocardiography Biatrial dilatation. Biventricular diastolic dysfunction with systolic dysfunction. Left ventricle showed asymmetric septal thickening. Moderate pericardial effusion. Restrictive pattern on Doppler of mitral valve inflow.

27.8.5 Final Diagnosis Bi-atrial dilatation with predominant left ventricular thickening with dysfunction suggestive of Restrictive Cardiomyopathy? Metabolic cause.

A. A. Deshpande

432

a

b

c

d

e

f

g

h

i

j

k

l

Fig. 27.9  Four chamber systolic (a) and diastolic (b) images show impaired relaxation of both the ventricles with dilated RA and LA (giving the appearance of tubular heart). Moderate pericardial effusion noted. Short axial diastolic (c) and systolic (d) images show asymmetric septal hypertrophy of the left ventricle and preserved systolic func-

tion of the ventricles. Short axis T1 (e) and T2 (f) maps do not show altered native myocardial values. Four chamber first-pass perfusion images (g, h) do not show perfusion defects. Short axis (i, j) and four chamber (k) LGE images do not show abnormal enhancement. Coronal survey image (l) showing pericardial effusion and gross ascites

Table 27.10  Functional parameters of Case 29.8 Functions EF EDV

LV Absolute 41.57 (56–78%) 38 (52–141 mL)

Indexed – 62.39 (41–81 mL/m2)

RV Absolute 31 (47–74%) 41.3 (58–154 mL)

27.9 Case 27.9

27.9.3 Provisional Diagnosis

27.9.1 Clinical Presentation

RCM

A 43-year-old man presented with gradually progressing exertional dyspnea and fatigue for last 3 years. Patient also complained about palpitations.

27.9.4 Cardiac MR

Indexed – 67.8 (48–87 mL/m2)

See Fig. 27.10 and Table 27.11.

27.9.2 Echocardiography Bi-atrial dilatation. Diastolic dysfunction of both ventricles with mildly reduced systolic functions. No e/o mitral/tricuspid regurgitation. No e/o pleural/pericardial effusion. No evident pericardial thickening noted.

27.9.5 Final Diagnosis Restrictive cardiomyopathy with biventricular systolic and diastolic dysfunction with mediastinal lymphadenopathy and mid-myocardial LGE. Findings are suggestive of RCM secondary to granulomatous etiology—tuberculosis/sarcoidosis.

27  Cardiac MR Imaging in Restrictive Cardiomyopathy

433

a

b

c

d

e

f

g

h

i

j

k

l

m

n

o

p

q

r

s

t

Fig. 27.10  Chest Radiograph (a) showed mild cardiomegaly with left ventricular enlargement. No e/o pleural effusion. Normal lung vascularity. Four chamber systolic (b) and diastolic (c) images show impaired relaxation of both the ventricles with dilated RA and LA (giving the appearance of tubular heart). Trufi axial image (d) show pre and pra-­ tracheal lymph nodes. Short axial diastolic (e) and systolic (f) images show preserved systolic function of the ventricles. No ventricular thickening noted. Short axis T1 (g) and T2 (h) images do not show abnormal myocardial intensity. Short axis first-pass perfusion images (i, j) do not

show perfusion defects. Short axis TI scout images (k–m) show normal nulling pattern of the myocardium (blood f/b myocardium). Short axis (n, o) and four chamber (p) LGE images show irregular mid-­myocardial enhancement in mid and apical LV segments (arrows). CT axial mediastinal sections (q–s) show small noncaseating pre- and paratracheal lymph nodes (arrow). Few of them showed calcifications. CT coronal section (t) in lung window shows mosaic attenuation in bilateral lung fields. However, no septal thickening or peri-bronchovascular nodules noted

A. A. Deshpande

434 Table 27.11  Functional parameters of Case 29.9 Functions EF EDV

LV Absolute 30 (56–78%) 107 (77–195 mL)

Indexed – 61.7 (47–92 mL/m2)

RV Absolute 16.9 (47–74%) 66.2 (88–227 mL)

Indexed – 37.9 (55–105 mL/m2)

27.10 Case 27.10

27.10.3 Provisional Diagnosis

27.10.1 Clinical Presentation

RCM

Sixty-two-year old man presented with gradually progressing exertional dyspnea and fatigue for last 5 years. No h/o palpitations. No other personal or family history.

27.10.4 Cardiac MR See Fig. 27.11 and Table 27.12.

27.10.2 Echocardiography Bi-atrial dilatation. Diastolic dysfunction of both ventricles with reduced systolic functions. No e/o mitral/tricuspid regurgitation. No e/o pleural/pericardial effusion. No evident pericardial thickening noted.

27.10.5 Final Diagnosis Restrictive cardiomyopathy with biventricular systolic and diastolic dysfunction with reversed nulling myocardial-­ blood pattern with diffuse sub-endocardial LGE.  Findings are suggestive of RCM secondary to Amyloidosis.

27  Cardiac MR Imaging in Restrictive Cardiomyopathy

435

a

b

c

d

e

f

g

h

i

j

k

l

m

n

o

p

Fig. 27.11  Four chamber diastolic (a) and systolic (b) images show impaired relaxation of both the ventricles with dilated RA and LA (giving the appearance of tubular heart). Mild concentric LV hypertrophy. Short axial diastolic (c) and systolic (d) images show preserved systolic function of the ventricles. Mild concentric LV hypertrophy. Four chamber first-pass perfusion images (e, f) do not show perfusion defects.

Short axis T1 (g) and T2 (h) images do not show abnormal myocardial intensity. Short axis TI scout images (i–k) show reversed nulling pattern of the myocardium (myocardium f/b blood). Trufi axial image (l) shows bilateral pleural effusion and pericardial effusion. Short axis (m, n) and four chamber (o, p) LGE images show diffuse sub-endocardial enhancement of the left ventricle

Table 27.12  Functional parameters of Case 29.10 Functions EF EDV

LV Absolute 37 (56–78%) 57.82 (77–195 mL)

Indexed – 31.95 (47–92 mL/m2)

RV Absolute 32 (47–74%) 64.77 (88–227 mL)

Indexed – 35.79 (55–105 mL/m2)

436

A. A. Deshpande

27.11 Case 27.11

27.11.3 Provisional Diagnosis

27.11.1 Clinical Presentation

RCM possibly secondary to hemochromatosis

A 20-year-old woman, k/c/o thalassemia major presented with mild dyspnea on exertion for last 3 months. She is on routine blood transfusions. No h/o palpitations. No other personal or family history.

27.11.4 Cardiac MR See Fig. 27.12 and Table 27.13.

27.11.2 Echocardiography

27.11.5 Final Diagnosis

Bi-atrial dilatation. Diastolic dysfunction of both ventricles with preserved systolic functions. No e/o mitral/tricuspid regurgitation. No e/o pleural/pericardial effusion. No evident pericardial thickening noted.

Bi-atrial dilatation, biventricular diastolic dysfunction and reduced cardiac and liver T2* and T1 values suggestive of restrictive cardiomyopathy secondary to hemochromatosis.

a

b

e

c

f

d

g

Fig. 27.12  Four chamber diastolic (a) and systolic (b) images show impaired relaxation of both the ventricles with dilated RA and LA (giving the appearance of tubular heart). Short axial diastolic (c) and systolic (d) images show preserved systolic function of the ventricles. No

e/o ventricular wall thickening noted. Short axis T2* map (e) shows reduced T2* values of heart and liver. Short axis (f) and axial (g) T1 map shows diffusely reduced native T1 values of the myocardium and liver

Table 27.13  Functional parameters of Case 29.11 Functions EF EDV

LV Absolute 63.87 (56–78%) 76.64 (52–141 mL)

Indexed – 67.16 (41–81 mL/m2)

RV Absolute 46.69 (47–74%) 37.36 (58–154 mL)

Indexed – 31.9 (48–87 mL/m2)

27  Cardiac MR Imaging in Restrictive Cardiomyopathy

437

27.12 Discussion

27.12.4 Imaging

27.12.1 Introduction

1. 2.

The European Society of Cardiology has defined cardiomyopathy as structural or functional abnormality of the myocytes in the absence of underlying coronary artery disease, valvular disease, or congenital heart disease [1]. The restrictive cardiomyopathy (RCM) is least common type of cardiomyopathy in which there is restrictive filling of the ventricle (diastolic dysfunction) with preserved systolic function until the later stage of the disease.

27.12.2 Etiology and Pathogenesis (Table 27.14) There are many causes of RCM, which can be classified as given in Table 27.1. The ventricular wall becomes stiff due to infiltration between myocytes, storage within myocytes, or fibrosis secondary to systemic disease, drugs, or radiation. It is unable to relax optimally resulting in diastolic dysfunction [2, 3].

27.12.3 Clinical Presentation 1. Patients generally present with dyspnea on exertion, fatigue, exercise intolerance, and rarely angina. 2. Some can also present with thromboembolic complications or with congestive heart failure. 3. RCM secondary to hemochromatosis can have bronze skin, liver cirrhosis, arthralgia, and diabetes.

Goals of Imaging: (a) Diagnosis (b) Prognostication (c) Management planning (d) Follow-up and identification of complications Imaging Modalities: (a) Chest Radiograph (Table 27.15): • CXR may show cardiomegaly with left atrial or biatrial enlargement (Table 27.2). • Patients with dilated left atrium may show signs of pulmonary venous hypertension. • It may be important to distinguish calcified constrictive pericarditis and restrictive cardiomyopathy. Lateral CXR is more sensitive to detect the pericardial calcification than PA view. (b) Echo: • Initial investigation of choice in evaluation of RCM. • Presence of diastolic dysfunction with normal or near-normal systolic function of the ventricle should suggest the diagnosis of restrictive cardiomyopathy. • Following information can be obtained with echo –– Specific cardiac chamber enlargement –– Functional and volumetric evaluation of the ventricles with detection of wall motion abnormalities –– Presence or absence of mitral/tricuspid regurgitation –– Flow Doppler study of mitral inflow

Table 27.14  Etiology of restrictive cardiomyopathy Infiltrative causes Amyloidosis Sarcoidosis Hemochromatosis Primary hyperoxaluria

Storage disorders Anderson-Fabry’s disease Gaucher’s disease Glycogen storage disorder Mucopolysaccharidosis Neimann-pick Dannon disease Friedrich’s Ataxia

Idiopathic Radiation therapy

Drug induced Anthracyclines Busulfan Ergotamines Methysergide Mercurial agents Serotonin containing agents

Systemic Diseases Diabetes Scleroderma Myofibrillar pathologies Psueodoxanthoma elasticum Werner syndrome Carcinoid Cardiomyopathy Hypereosinophilic cardiomyopathy (Loeffler’s syndrome) Endomyocardial fibroelastosis Metastatic malignancies

A. A. Deshpande

438 Table 27.15  CXR features of right and left atrial enlargement Right atrial enlargement Increased right atrial convexity—>50% of total cardiovascular height Maximum convexity of right cardiac border—>3 cm from right lateral vertebral border

Left atrial enlargement Superior displacement/lifting up of left main bronchus with widening of carinal angle (normal—51–71 degree). Double density sign—projection of left atrium lateral to right lower heart border. Lateral CXR with barium can identify the indentation over esophagus in the early stages.

Table 27.16  Information obtained with CMR Morphologic features

Functional information

Tissue characterization Extracardiac findings Flow quantification

Cardiac MR modules Ventricular (right and left) morphology including the size Ventricular and septal wall thickness (thickening/thinning) Ventricular aneurysms Measures end-diastolic and end-systolic ventricular volumes Stroke volumes and ejection fractions for each ventricle Regional wall motion abnormalities/global abnormalities Detects active disease (edema) and scarring Lymphadenopathy, lung parenchymal changes Trans-mitral flow to prognosticate the disease

(c) CMR: • CMR is the investigation of choice for volumetric and functional evaluation of the ventricles. • Following information can be obtained with the use of CMR (Table 27.16).

27.12.5 Goals of CMR

27.13 Endomyocardial Fibrosis 27.13.1 Introduction Endomyocardial fibrosis (EMF) is a rare disease. It was first discovered in Uganda in 1940s and is mainly seen in tropical countries. It involves fibrosis of apical segments of either left or right or both the ventricles, resulting in restrictive cardiomyopathy. It is primarily disease of the young population affecting in late 20s and 30s [4].

27.13.2 Etiology and Pathogenesis • The exact cause of EMF is unknown (Table 27.17). • EMF shows involvement of both ventricles, left ventricle, or right ventricle in 50%, 40%, and 10% of cases, respectively. • The fibrosis starts in the apices of the ventricle and can progress to involve entire ventricle. However, the left ventricular outflow tract is usually spared. • The advanced cases show the development of apical thrombus, which gives the characteristic “Triple layer appearance” on late gadolinium-enhanced images. • Also, in advanced stages, there can be tethering of AV valves or involvement of papillary muscles, resulting in AV valve regurgitation.

Table 27.17  Etiology of EMF Eosinophilia

Infections

1. To confirm the diagnosis of restrictive cardiomyopathy. It can rule out constrictive pericarditis in the absence of pericardial thickening. 2. To identify the etiology of RCM based on the myocardial Environmental intensity, parametric mapping, and late gadolinium-­ and dietary enhanced images. 3. To identify the ventricular involvement and activity of the disease. Immunologic 4. To assess the response to treatment. CMR provides comprehensive and multidimensional assessment, including morphologic, functional evaluation and tissue characterization in a reproducible and operatorindependent manner.

Genetic

The pathogenesis of EMF resembles that of fibrotic stage of Loeffler’s Endocarditis. However, eosinophilia may not be seen in every case of EMF. Several infectious agents are considered in the pathogenesis including toxoplasma, plasmodium, Schistosoma, Helminths, and Arbovirus. However, no definite association has been identified. Cerium and thorium have been found to be in high quantities in endemic areas. Excessive consumption of Cassava (Tapioca) has also been hypothesized to be a causative factor for development of EMF [5]. Antimyosin antibodies are found in some patients of EMF. However, it is nonspecific as it can also be seen in other conditions like Dressler syndrome, postcardiac transplant rejection, and rheumatic heart disease. Few HLA alleles have found to be associated with EMF; however, the exact role of genetics in development of EMF is unclear [6].

27  Cardiac MR Imaging in Restrictive Cardiomyopathy

439

27.13.3 Clinical Presentation • The clinical presentation depends on the cardiac chamber involved. • The early phase of EMF consists of acute febrile illness, which may progress to subacute to chronic stage of cardiac failure. • Atrial fibrillation is also common in patients with end-­ stage EMF and is associated with poor prognosis. • Patients with predominant right ventricular involvement present with hepatomegaly, ascites, lower limb edema, and elevated jugular venous pressure. • Patients with predominant left ventricular involvement present with dyspnea, fatigue, or orthopnea.

27.13.4 Imaging 1. Echo: The findings are similar to that seen in RCM. The specific findings in EMF include • Apical fibrosis of RV, LV, or both • Apical thrombus • Tethering of papillary muscles, leading to AV regurgitation 2. CMR: All the findings of RCM are seen in EMF as described previously. Characteristic findings seen in EMF include • Obliteration of the LV, RV, or biventricular apices • Sub-endocardial late gadolinium enhancement in the ventricular apex/apices • Advanced stages show characteristic “Triple layer appearances” (hyperintense—hypointense—hyperintense) on late gadolinium-enhanced images (hyperintense blood, hypointense clot, and hyperintense subendocardial enhancement of the apex)

27.13.5 How Imaging Impacts the Management? The management of RCM mainly depends upon the underlying etiology (refer to section of amyloidosis, sarcoidosis, and hemochromatosis for specific treatments). The management of idiopathic RCM is mainly supportive with the goal of reduction of systemic and pulmonary congestion. The goal of management includes • Lowering the venous pressure using diuretics. • Maintaining heart rate using calcium channel blockers and beta-blockers. • Increasing filling time using ACE inhibitors. • Maintenance of atrial contractions using a dual chamber cardiac pacemaker in case of AV block. Patients with

• • • •

atrial fibrillation can also be started with anticoagulation to prevent the risk of thromboembolism. Correction of atrioventricular conduction disturbances. Avoidance of anemia, nutritional deficiency, calcium overload, and electrolyte imbalance. Cardiac Transplant can be done in eligible patients in the advanced stage [7]. The medical therapy is similar to the heart failure management as in RCM.  The surgical treatment includes endomyocardial resection with valve replacement or repair. Most of the patients of EMF present at an end-­ stage and the mortality rate can be as high as 25% despite the best medical therapy.

27.13.6 Current Controversies and Future Aspects • Endomyocardial Biopsy: –– It may be performed in RCM if the underlying diagnosis is uncertain. It may reveal the underlying cause such as amyloidosis, sarcoidosis, or hemochromatosis. –– CMR may help to reduce the false-negative biopsy by guiding the site of biopsy. • Feature tracking–Cardiac MR (FT–CMR): –– Myocardial strain analysis is routinely used using speckle tracking echo (STE) for diagnosis, disease monitoring, and prognosis of the RCM. –– It can also be done with CMR using feature tracking technique and is shown to be as effective as STE in previous studies [8].

Multiple Choice Questions Question 1: All of the following can cause restrictive cardiomyopathy except (a) Radiation (b) Hypereosinophilic syndrome (c) Anthracyclin toxicity (d) None of the above Question 2: Which of the following can be an underlying etiology of restrictive cardiomyopathy? (a) Hypereosinophilc syndrome (b) Cardiac hemochromatosis (c) Anthracyclines (d) All of the above Question 3: All of the following are true regarding endomyocardial fibrosis EXCEPT (a) It is seen in tropical countries more commonly. (b) It can be an underlying cause of restrictive cardiomyopathy.

A. A. Deshpande

440

(c) It can be seen in end stage of hypereosinophilic syndrome. (d) It is generally seen in elderly population. Question 4: All of the following can be seen with restrictive cardiomyopathy EXCEPT (a) Normal ventricular systolic function (b) Restrictive pattern of filling on trans-mitral flow study (c) Increased ventricular end-diastolic volumes (d) Bi-atrial dilatation Question 5: All of the following can be seen in endomyocardial fibrosis EXCEPT (a) Dilated cardiomyopathy (b) Obliteration of the ventricular apex (c) Ventricular apical thrombus (d) Mitral regurgitation Question 6: Which of the following is/are proposed underlying cause/s of endomyocardial fibrosis? (a) Malarial infection (b) Malnutrition (protein poor diet) (c) Eosinophilia (d) All of the above Question 7: Which of the following surgical procedure can be performed for endomyocardial fibrosis? (a) Pericardectomy (b) Endomyocardial resection (c) Pulmonary artery banding (d) Norwood procedure Question 8: Which of the following is true regarding management of endomyocardial fibrosis? (a) It has excellent long-term prognosis in most of the cases. (b) Diuretics are indicated to reduce the venous congestion. (c) CCBs or B-blockers are indicated to control the heart rate. (d) Cardiac transplant can be performed in selective cases. Question 9: Which of the following CMR sequence may not be useful in restrictive cardiomyopathy? (a) Myocardial tagging sequence

(b) Parametric mapping sequences (c) Feature tracking analysis (d) None of the above Question 10: All of the following are infiltrative causes of restrictive cardiomyopathy EXCEPT: (a) Endomyocardial fibrosis (b) Amyloidosis (c) Primary oxaluria (d) Hemochromatosis Answers 1 d

2 d

3 d

4 c

5 a

6 d

7 b

8 a

9 d

10 a

References 1. Elliott P, Andersson B, Arbustini E, et al. Classification of the cardiomyopathies: a position statement from the European Society of Cardiology working group on myocardial and pericardial diseases. Eur Heart J. 2008;29(2):270–6. https://doi.org/10.1093/eurheartj/ ehm342. 2. Brown KN, Pendela VS, Diaz RR. Restrictive (infiltrative) cardiomyopathy. In: StatPearls. Treasure Island, FL: StatPearls Publishing; 2020. https://www.ncbi.nlm.nih.gov/books/NBK537234/. 3. Rammos A, Meladinis V, Vovas G, Patsouras D. Restrictive cardiomyopathies: the importance of noninvasive cardiac imaging modalities in diagnosis and treatment-a systematic review. Radiol Res Pract. 2017;2017:2874902. https://doi.org/10.1155/2017/2874902. 4. Bhatti K, Bandlamudi M, Lopez-Mattei J.  Endomyocardial fibrosis. In: StatPearls. Treasure Island, FL: StatPearls Publishing; 2020. https://www.ncbi.nlm.nih.gov/books/NBK513293/. 5. Anandan PK, Shukkarbhai PJ, George J, Bhatt P, Manjunath CN.  Tapioca cardiomyopathy: curse of cassava endomyocardial fibrosis. Cardiol Res. 2015;6(2):260–2. https://doi.org/10.14740/ cr394w. 6. Beaton AMD, Sable CMD, Brown JPD, et  al. Genetic susceptibility to endomyocardial fibrosis. Glob Cardiol Sci Pract. 2014;2014(4):473–81. https://doi.org/10.5339/gcsp.2014.60. 7. Moraes F, Lapa C, Hazin S, et al. Surgery for endomyocardial fibrosis revisited. Eur J Cardiothorac Surg. 1999;15:309. 8. Amaki M, Savino J, Ain DL, et al. Diagnostic concordance of echocardiography and cardiac magnetic resonance-based tissue tracking for differentiating constrictive pericarditis from restrictive cardiomyopathy. Circ Cardiovasc Imaging. 2014;7(5):819–27. https://doi. org/10.1161/CIRCIMAGING.114.002103.

Imaging in Cardiac Tuberculosis

28

Sravan Nagulakonda

28.1 Case 28.1 28.1.1 Clinical Presentation A 62-year-old woman with chest pain and breathlessness from 2 weeks. ECG showed nonspecific changes. Troponin levels are elevated. Catheter angiography showed no signifi-

cant coronary artery disease. On echocardiography, there is diffuse hypokinesia of left ventricle.

28.1.2 CMR See Fig. 28.1.

S. Nagulakonda (*) Department of Cardio Vascular Radiology and Endovascular Interventions, AIIMS, Ansari Nagar, New Delhi, India © The Author(s), under exclusive license to Springer Nature Singapore Pte Ltd. 2023 S. Sharma (ed.), Case-based Atlas of Cardiac Imaging, https://doi.org/10.1007/978-981-99-5620-3_28

441

442

S. Nagulakonda

a

b

f

j

c

g

k

Fig. 28.1 (a–c) T1  W and T2  W short axis and T2 four chamber images showing T2 hyperintensities in interventricular septum (d, e) cine systolic and diastolic images showing diffuse hypokinesia. (f, g) T1 mapping images showing elevated T1 values in anterior and inferior septal segments at basal and mid ventricular regions. (h, i) T2 mapping

d

e

h

i

l

images showing elevated T2 values in anterior and inferior septal and inferior segments at basal and mid ventricular regions. (j–l) Subepicardial, mid wall and subendocardial LGE in anterior and inferior septal segments at basal and mid ventricular levels

28  Imaging in Cardiac Tuberculosis

28.2 Case 28.2

443

28.2.2 Final Diagnosis Chronic constrictive pericarditis

See Fig. 28.2.

28.2.1 Clinical Presentation A 26-year-old woman with history of progressive breathlessness from 1-year, pedal edema, and past history of tuberculosis. Echo features are consistent with CCP.

a

c

b

d

Fig. 28.2  CXR: (a) Pericardial calcifications, pulmonary venous hypertension, and pleural effusion. CT (b, c) pericardial calcifications. (d) Reduced volume with tubular configuration of both ventricles and dilated RA and LA

444

S. Nagulakonda

28.3 Case 28.3 See Fig. 28.3.

pedal edema from 6 months. Not a known hypertensive or diabetic. Past history of pulmonary tuberculosis. On examination, there is tender hepatomegaly.

28.3.1 Clinical Presentation

28.3.2 Final Diagnosis

A 47-year-old man presented with chest pain, breathlessness, progressive fatigability, abdominal distention, and

Chronic constrictive pericarditis secondary to tubercular infection; Myocardial fibrosis and diastolic dysfunction.

a

f

b

g

c

e

d

h

Fig. 28.3  CXR: (a, b) Cardiomegaly with biatrial enlargement, dilated SVC, and pulmonary venous hypertension. Curvilinear and nodular radio opacities along right heart border (frontal radiograph), left heart border (on lateral radiograph) likely pericardial calcifications. CT (c–f)

i

j

Pericardial calcifications extending into RV and LV myocardium. (g) Mediastinal lymphadenopathy (h, i) myocardial T1 and T2 hypointense lesion involving inferolateral wall with LGE (j)

28  Imaging in Cardiac Tuberculosis

445

28.4 Case 28.4

there is pericardial thickening and effusion with restrictive ventricular filling.

See Fig. 28.4.

28.4.2 Final Diagnosis

28.4.1 Clinical Presentation

Effusive constrictive pericarditis A 13-year-old girl present with progressive breathlessness, pedal edema, and abdominal pain from 2 months. On Echo,

a

c

b

d

e

Fig. 28.4  CXR reveals (a) bilateral pleural effusion. CT (b) pericardial thickening and pleural effusion (c) mediastinal lymphadenopathy (d, e) enhancing pericardial layers

446

S. Nagulakonda

28.5 Case 28.5

28.5.2 Final Diagnosis

See Fig. 28.5.

Inflammatory mediastinal mass (tubercular) infiltrating into RA and LA

28.5.1 Clinical Presentation A 30-year-old male presented with progressive breathlessness, facial and arm swelling. On echo, vascular mass is seen infiltrating both atria.

a

d

b

e

Fig. 28.5 (a, b) Soft tissue density mass lesion in posterior mediastinum infiltrating into RA and LA. (c) Tumor vascularity and (d) nodules in right upper lobe (e, f) T1 and T2 W MRI images showing heteroge-

c

f

g

neous mass in mediastinum infiltrating into RA and LA (g) LGE image showing heterogeneous enhancement

28  Imaging in Cardiac Tuberculosis

447

28.6 Case 28.6

28.6.2 Final Diagnosis

See Fig. 28.6.

Right intracardiac mass–inflammatory–possible tubercular

28.6.1 Clinical Presentation A 30-year-old woman presented with progressive breathlessness, facial and arm swelling. On echo, vascular mass in right atrium extending into right ventricle was seen.

a

c

b

e

g

h

Fig. 28.6  CXR: (a, b) Pericardial calcifications along aortic recess and along diaphragmatic surfaces. (c–e) Soft tissue density mass in the RA extending into RV and RA/SVC junction occluding SVC. (f) Normal

d

f

i

j

lungs. (g, h) lesion is T1 hypointense and T2 hyperintense and (i, j) increased perfusion and heterogeneous enhancement on LGE

448

S. Nagulakonda

28.7 Case 28.7

28.7.2 Final Diagnosis

See Fig. 28.7.

Pericardial mass infiltrating into right atrium

28.7.1 Clinical Presentation A 23-year-old woman presented with progressive breathlessness and pedal edema. On echo, a mass seen along free wall of right atrium.

a

b

d

e

Fig. 28.7  CT reveals (a) Necrotic mediastinal lymph nodes (b) Soft tissue density lesion in pericardium extending into right atrium. (c) (Trufi), (d) (T1W) and (e) (T2W) showing lesion with epicenter in the

c

f

pericardium extending into right atrium. (f) (LGE) showing heterogeneous enhancement of the soft tissue with adherent thrombus

28  Imaging in Cardiac Tuberculosis

449

28.8 Case 28.8

28.8.2 Final Diagnosis

See Fig. 28.8.

Tubercular myopericarditis with pseudoaneurysm

28.8.1 Clinical Presentation A 30-year-old woman presented with progressive breathlessness, facial and arm swelling. On echo, vascular mass in right atrium extending into right ventricle was seen.

a

b

f

g

c

h

Fig. 28.8  CXR: (a) shows bilateral pleural effusion. (b–e) Bilateral pleural effusion with air pockets within the pleural effusion with pericardial calcifications and calcification along the pseudoaneurysm. (f) T2 FS images showing rent in anterior wall of LV with pseudoaneurysm

d

e

i

j

(g, h) Cine systolic and diastolic images showing diffuse hypokinesia. (i) Cine image showing LV pseudoaneurysm (j) LGE image shows pericardial thickening and enhancement along RV and along the wall of the pseudoaneurysm

450

28.9 Dviscussion 28.9.1 Introduction Tuberculosis is still rampant across the world with an estimated incidence of ~10 million cases each year. Cardiac involvement in tuberculosis was uncommon, with only 2 affected out of 100 patients with the disease on autopsy studies. However, extrapulmonary involvement has been reported more commonly in the recent decades because of HIV/AIDS coinfection. Autopsy studies of patients who had tuberculosis and were coinfected with HIV/AIDS showed involvement of two or more systems in up to 80% patients [1–3]. Cardiac involvement in TB is usually severe and is a leading cause of fatality, following central nervous system disease.

28.9.2 Pathophysiology and Clinical Features Cardiac tuberculosis may affect either the pericardium or the myocardium. Disease limited to the pericardium is usually seen in individuals with a competent immune system. In these individuals, an exudative pericardial effusion is seen as a result of type I hypersensitivity reaction to the tubercle protein. Bacteremia is usually not significant. Cytokines are released by the helper T cells which may lead to hemodynamic sequelae. Immunosuppressed individuals fail to control the bacterial replication, with resultant high bacterial loads. Here, the pathogen directly affects the pericardium and in majority of cases myocardial involvement is also seen. Presentation is variable in cardiac tuberculosis and can be grouped into the following four categories [4].

S. Nagulakonda

This constellation of symptoms has been named effusive constrictive pericarditis. • Disease course and chronology in this scenario is difficult to predict in an individual patient. • Patients may be complicated by tamponade in the acute stage, while fusion between the layers of the pericardium in the chronic stages may result in constrictive pericarditis.

28.9.2.3 Constrictive Pericarditis • Detailed examination for subtle clinical signs combined with the information obtained from imaging evaluation including echocardiography, computed tomography or MRI is required to make a diagnosis in the backdrop of high clinical suspicion of constrictive pericarditis. Rarely, invasive catheterization may be required for accurate evaluation. 28.9.2.4 Myopericarditis • Myocardium is involved along with pericardial involvement. • Myocardial involvement, like any other myocarditis, will show elevated enzymes (troponin, lactate dehydrogenase, etc.), typical ECG changes, and mild reductions in ejection fraction. • Postcontrast enhancement of myocardium and pericardium may be seen on cardiac MRI. • In patients with a competent immune system, this presentation is extremely rare. In immunosuppressed individuals, especially those with CD4 counts of less than 100 cells/mL, this is much more common.

28.9.2.5 Myocarditis • Tuberculous myocarditis is very uncommon. In a study by Rose and colleagues, it was found only in 0.14% of ~13,000 postmortem studies performed over 27 years. 28.9.2.1 Acute Pericarditis • Mode of involvement of the myocardium is like those of • Acute pericarditis is characterized by fluid accumulation the pericardium and aorta. There is an inexplicable in the pericardial cavity and presentation depends on the ­predilection toward the right-sided cardiac chambers, parrate at which it accumulates, along with the severity of ticularly the right atrium. inflammation and reduction in pericardial compliance. • On gross specimens, these may manifest as tuberculous • This form of pericarditis is rare in this patient cohort, connodules with central caseation, miliary nodules, or a difstituting 2–8% of cardiac tuberculosis. fuse infiltrative pattern with pericarditis. • Large quantity of effusion or rapid accumulation of fluid • It is difficult to diagnose antemortem and has a high fatalmay overwhelm the compensatory mechanism, resulting ity rate due to this reason. in significant reduction in the stroke volume and end-­ • Patients may present with tachyarrhythmias of the atria or diastolic volume. ventricles, conduction defects, and abnormal dilatation of the • Conversely, when the compensatory mechanisms cope cardiac chambers including aneurysms or ­pseudoaneurysm. with the fluid accumulation, patients present with sympThey may also present with dilated cardiomyopathy associtoms of heart failure without hypotension, and may have ated with heart failure or a sudden cardiac death. signs of pericardial effusion. • A high degree of clinical suspicion is essential for diagnosis. Imaging investigations including echocardiography, 28.9.2.2 Effusive Pericarditis computed tomography, or MRI can show findings that are • Compliance of the visceral pericardium may be affected suggestive of the condition. Definitive diagnosis requires by inflammation and sequelae of infection. In this situademonstration of caseous necrosis alone or in combination, patients present with hemodynamic features of tamtion with Mycobacterium tuberculosis bacilli on endoponade and a physiology akin to constrictive pericarditis. myocardial biopsy.

28  Imaging in Cardiac Tuberculosis

Multiple Choice Questions Question 1: Cardiac tuberculosis most commonly manifests as (a) Pericarditis (b) Myocarditis (c) Myopericarditis (d) Endocarditis Question 2: Imaging modality of choice to detect pericardial calcification (a) Echo (b) CT (c) CMRI (d) Chest X ray Question 3: Imaging modality of choice in a case of suspected chronic constrictive pericarditis (a) Echo (b) CT (c) CMRI (d) Cath Angio Question 4: Chronic constrictive pericarditis can be seen with (a) Radiation (b) Postcardiac Surgery (c) Tubercular Infection (d) All of the above Question 5: Regarding myocardial tuberculoma, identify the correct statement (a) CT is more sensitive than MRI in identifying myocardial tuberculoma. (b) Central T2 hyperintensity is seen on MRI. (c) It is the most common manifestation of cardiac tuberculosis. (d) None of the above. Question 6: Types of myocardial involvement in TB are all except (a) Military (b) Diffuse myocarditis (c) Caseous tuberculoma (d) None

451

Question 7: Pericardial thickening is best detected by (a) X-ray (b) CT (c) CMRI (d) Echo Question 8: Tagging sequences are used in cardiac TB to detect (a) Early pericarditis (b) Early myocarditis (c) Pericardial adhesions to myocardium (d) Pericardial abscess Question 9: Pericardial calcifications are most commonly seen at (a) Along diaphragmatic surface (b) Along AV groove and right heart margin (c) Along AV groove and left heart margin (d) Along anterior wall Question 10: Imaging findings in constrictive pericarditis can be all except (a) Dilated RA and RV (b) Dilated RV and LV (c) Pericardial thickening with calcification (d) Pericardial thickening without calcification Answers 1 a

2 b

3 c

4 d

5 d

6 d

7 c

8 c

9 b

10 b

References 1. Fowler NO. Tuberculous pericarditis. JAMA. 1991;266(1):99–103. 2. Rana FS, Hawken MP, Mwachari C, et al. Autopsy study of HIV-­ 1-­positive and HIV-1-negative adult medical patients in Nairobi, Kenya. J Acquir Immune Defic Syndr. 2000;24(1):23–9. 3. Shafer RW, Kim DS, Weiss JP, et al. Extrapulmonary tuberculosis in patients with human immunodeficiency virus infection. Medicine (Baltimore). 1991;70(6):384–97. 4. Mutyaba AK, Ntsekhe M. Tuberculosis and the heart. Cardiol Clin. 2017;35(1):135–44. https://doi.org/10.1016/j.ccl.2016.08.007.

Imaging in Iron Overload Cardiomyopathy

29

Surya Pratap Singh

29.1 Case 29.1

29.1.3 ECG

29.1.1 Clinical History

Low voltage QRS complex, nonspecific ST/T wave changes

Twenty-year-old woman, a k/c/o thalassemia major presented with mild dyspnea on exertion for last 3 months. She is on routine blood transfusions. No h/o palpitations. No other personal or family history.

29.1.2 Echo Diastolic dysfunction of both ventricles with preserved systolic functions. No e/o mitral/tricuspid regurgitation.

29.1.4 Provisional Diagnosis Restrictive Cardiomyopathy possibly secondary to hemochromatosis. CMR was done to assess the iron overload in heart in liver specifically as the diagnosis of cardiac hemochromatosis was evident on the basis of history and echo findings (Fig. 29.1, Table 29.1).

S. P. Singh (*) Department of Cardio Vascular Radiology and Endovascular Interventions, AIIMS, Ansari Nagar, New Delhi, India © The Author(s), under exclusive license to Springer Nature Singapore Pte Ltd. 2023 S. Sharma (ed.), Case-based Atlas of Cardiac Imaging, https://doi.org/10.1007/978-981-99-5620-3_29

453

454

S. P. Singh

a

b

e

c

f

Fig. 29.1 (a, b) Four chamber systolic and diastolic images show impaired relaxation of both the ventricles with dilated RA and LA (giving the appearance of tubular heart). (c, d) Short axis diastolic and systolic images show preserved systolic function of the ventricles. No e/o

d

g ventricular wall thickening noted. (e) Short axis T2* map shows reduced T2* values of heart and liver. (f, g) Short axis and axial T1 map shows diffusely reduced native T1 values of the myocardium and liver

Table 29.1  T2* values of myocardium and liver Myocardium Liver

T2 * value 6.1 1.8

Interpretation Normal (T2* > 20 ms); mild (T2* = 15–20 ms); moderate (T2* = 10–15 ms); severe (T2*  11.4; R2*  20 ms); mild (T2* = 15–20 ms); moderate (T2* = 10–15 ms); severe (T2*  20 ms); mild (T2* = 15–20 ms); moderate (T2* = 10–15 ms); severe (T2*  20 ms); mild (T2* = 15–20 ms); moderate (T2* = 10–15 ms); severe (T2*  20 ms); mild (T2* = 15–20 ms); moderate (T2* = 10–15 ms); severe (T2* 200 mL. • Orthogonal imaging like CT or MR can be utilized to evaluate the underlying cause of the effusion, rather than to confirm the diagnosis. 2. Aneurysm or pseudo-aneurysm: LV aneurysm after myocardial infarction is seen in nearly 12% of patients. It can either be a true or pseudo-aneurysm. Difference between the two entities has already been discussed in Table 31.1. 3. Thrombus: Patients with MI are five times more probable to have thrombus when compared to patients without an infarct having similar levels of systolic dysfunction. Therefore, scarring is an important risk factor for thrombus formation. Thrombus is seen in 20% of all infarcts, 60% of apical infarcts, and 40% of anterior infarcts. ­During the acute phase, secondary to endocardial inflammation, it occurs on the endocardial surfaces overlying the infarct. • Echocardiography is good screening modality in identifying an intracardiac thrombus. • MRI offers higher contrast resolution, allowing detection of even small thrombi. • The delayed-enhancement technique has a sensitivity of 88% and specificity of 99% for detection of a thrombus, making this method superior to both SSFP and echocardiography. 4. Mitral regurgitation: Mitral regurgitation is an independent adverse prognostic factor, which results in increased chances of heart failure and mortality. Development of mitral regurgitation is associated with global and regional remodeling; papillary muscle rupture, infarction, or dysfunction; and acute systolic mitral annular dilatation. It is found in 11–59% of patients with MI. Also, in the setting

A. Taxak

478

5.

6.

7.

8.

of ischemic mitral regurgitation because of associated LV remodeling and systolic dysfunction, the prognosis is worse. Postinfarction ventricular septal defect: it is a rare complication in patients after MI, seen in only 2% of patients, if present, is often lethal and seen 2–8 days after an episode. • Cardiac CT can be used to identify and ascertain the exact anatomy, thus also helps in planning a suitable closure/surgical strategy for management. • Apart from anatomical depiction using cine MR imaging, 2-D and 4-D phase-contrast MR imaging can additionally demonstrate the shunt as well as allow quantification of the shunt across the defect, either directly or indirectly. Particularly in cases that present with RV dilatation and/or dysfunction, can be thought to be due to pulmonary emboli in cases of circulatory compromise. Free wall rupture: The most common site of involvement is the LV free wall rather than septum. Found in 10–20% of patients with MI. Myocardial rupture is more common in the first 1–4 days after an MI, when the wall is weakest because of coagulation necrosis and neutrophil infiltration. Delayed-enhancement MR imaging can be helpful in demonstrating the loss of continuity of the myocardial wall, with or without hematoma. Patients at increased risk include • Female • Those experiencing their first MI • ≥60 years of age • Transmural infarct involving 20% of the wall • Multiple vessel involvement • Poor collateral supply, or absence of ventricular hypertrophy • Those who experienced delayed initiation of thrombolytics Ventricular arrhythmias: Two principal mechanisms appear to contribute Myocardial ischemia and increased myocardial stretch can lead to enhanced automaticity or triggered activity. The myocardium located at the border zone is heterogeneous, consisting of a mixture of fibrotic tissue, inflammatory cells, and damaged and disorganized muscle fibers. A reentrant tachycardia may develop when two or more electrically heterogeneous pathways having different conduction velocities and refractoriness are connected proximally and distally. Intramyocardial dissecting hematoma (IDH): It is the infiltration of blood through the myocardial wall, with intact endocardium and epicardium. It is initially contained within the myocardium; however, the hematoma can enlarge and rupture into adjacent chambers/pericardium. It is rare complication of myocardial infarction and

Table 31.9  Management of complications of myocardial ischemia LV aneurysm

LV pseudo-­ aneurysm

Pericardial effusion

• Surgery is the preferred therapeutic option. • The rate of mortality in patients with left ventricular aneurysms is up to 6× higher than in patients without aneurysms. • Approximately 25% (range 19–32%) of patients are at risk of fatal ventricular rupture, and about 30–45% of left ventricular pseudoaneurysms will eventually rupture. • Most ruptures will occur in the early postoperative period. • If small, asymptomatic and clinically not suspect, then conservative management. • If large, symptomatic or there is a clinical concern of the underlying cause (e.g., infection, malignancy), then pericardiocentesis. • If recurrent and symptomatic (e.g., malignancy), then pericardial fenestration.

can develop in the LV free wall, the IV septum, or the right ventricle. In certain cases, it can spontaneously resolve as well. • The diagnosis is usually made after surgery, postmortem examination, or by echocardiography. • Differential diagnosis includes pseudoaneurysm, intracavitary thrombosis, or prominent ventricular trabeculations.

31.6.6 Management Surgical correction versus medical management is decided on case-to-case basis. Preoperative cross-sectional imaging guides toward the surgical approach. Death is often sudden and may be related to the high incidence of associated ventricular tachyarrhythmias (Table 31.9). Multiple Choice Questions Question 1: All of the following are features of a true aneurysm except (a) Myocardial thinning (b) Has a narrow neck (c) Dyskinetic during systole (d) Usually arise from anterior or apical wall Question 2: All of the following are complications following myocardial ischemia except (a) LV pseudoaneurysm (b) Intramyocardial dissecting hematoma (c) LV diverticulum (d) Mitral regurgitation Question 3: All of the following are acute complications of myocardial ischemia except (a) Pericarditis (b) Myocardial rupture (c) LV aneurysm (d) Cardiogenic shock

31  Imaging in Complications of Myocardial Ischemia

479

Question 4: Most common site of true aneurysm (a) Anterior/ apical (b) Posterior (c) Inferior (d) Perivalvular Question 5: All are true about a LV pseudo-aneurysm except (a) Most common site is anterior/ apical wall (b) Akinetic with the surrounding myocardium (c) Neck mouth ratio of 0.25–0.5 (d) Myocardium is not a part of the sac Answers 1 c

2 b

3 c

4 a

5 a

References 1. Rajiah P, Desai MY, Kwon D, Flamm SD.  MR imaging of myocardial infarction. Radiographics, Cardiac Imaging. 2013;33:1383– 412. https://doi.org/10.1148/rg.335125722. 2. Ko SM, Kim TH, Chun EJ, Kim JY, Hwang SH. Assessment of left ventricular myocardial diseases with cardiac computed tomography. Korean J Radiol. 2019;20(3):333–51. https://doi.org/10.3348/ kjr.2018.0280. 3. Kamanahalli R, Ngo A, O’Regan D, et al. Myocardial infarction and its complications—CT and MR appearances. ECR. 2014; https:// doi.org/10.1594/ecr2014/C-­1984.

Imaging in Takotsubo Cardiomyopathy

32

Rishabh Khurana

32.1 Case 32.1

32.1.6 Provisional Clinical Diagnosis

32.1.1 Clinical Presentation

Postmenopausal elderly woman with acute chest pain, ST elevation, raised cardiac markers, normal coronaries, and LV dysfunction—provisional diagnosis of MINOCA was made.

A 78-year-old diabetic and hypertensive woman presented to emergency with left sided chest pain, shortness of breath, and sweating for past 10 h. There was a history of accidental death of family member on the previous day. Blood Pressure was elevated (180/90 mmHg).

32.1.2 Chest Radiograph

32.1.7 Differential Diagnosis 1. Takotsubo cardiomyopathy 2. Acute myocardial infarction with recanalized coronary artery 3. Myocarditis

Within normal limits

32.1.3 ECG ST segment elevation in leads II, II, and aVF

32.1.4 Echo

32.1.8 Next Step in Evaluation? CMR needs to be done to establish a conclusive diagnosis. CMR will provide morphologic and functional evaluation, as well as assess for complications.

32.1.9 Cardiac MRI

Apical hypokinesia, LV dysfunction (LVEF: 40%) See Fig. 32.1c–v

32.1.5 What Investigation Will You Do Next? Catheter coronary angiography: Coronaries were normal (Fig. 32.1a, b).

32.1.10 Final Diagnosis Takotsubo Cardiomyopathy

32.1.11 Follow-Up Echo (After 3 Weeks) Normal LVEF with no RWMA R. Khurana (*) Department of Cardio Vascular Radiology and Endovascular Interventions, AIIMS, Ansari Nagar, New Delhi, India © The Author(s), under exclusive license to Springer Nature Singapore Pte Ltd. 2023 S. Sharma (ed.), Case-based Atlas of Cardiac Imaging, https://doi.org/10.1007/978-981-99-5620-3_32

481

482

R. Khurana

a

c

Fig. 32.1 (a, b) Catheter angiogram showing normal coronaries. (c–j) [Cine frames four chamber: ED (c) and ES (d); Cine frames Short axis: basal (e, f), mid (g, h) and apical LV (i, j)]: Basal LV shows hypercontractility. There is ballooning of mid-apical LV with wall thinning with associated hypokinesia. LVEF: 33.6%. RV EF: 55%. Mild pericardial effusion seen. (k–m) (T2 TSE; T1-TSE): No abnormal signal intensity. (n, o) (Mapping): The T1 and T2 mapping values were elevated, 1200– 1300 ms and 65–85 ms, respectively. (p–r) (Dynamic post contrast): No

b

d

resting perfusion deficit. (s–v) (LGE): Diffuse patchy LGE is seen involving mid to apical LV segments. Fibrosis/Scar quantification: Using mean  +  3SD method, the quantified scar was 3.3%. Using mean + 5SD method, no scar was visible. Other findings: Mild pericardial effusion. Normal atria. No Thrombus. No MR/TR. No pleural effusion. No mediastinal lymphadenopathy. Permission was obtained from The British Journal of Radiology [1]

32  Imaging in Takotsubo Cardiomyopathy

483

e

f

g

h

Fig. 32.1 (continued)

484

R. Khurana

i

k

n

Fig. 32.1 (continued)

j

l

m

o

32  Imaging in Takotsubo Cardiomyopathy

485

p

q

r

s

t

u

v

Fig. 32.1 (continued)

32.1.12 Differential Diagnosis 1. Acute Myocardial Infarction with recanalized coronary artery (Refer to Chap. 30) The myocardial edema involving Takotsubo Cardiomyopathy matches with cine abnormalities (nonvascular territory) and resolves within weeks, whereas in myocardial infarction, it involves vascular territory and resolves within 2–3 months. 2. Myocarditis (Refer to Chap. 20) In myocarditis, the edema and LGE is seen involving sub-epicardial/mid-myocardial region

32.2 Discussion 32.2.1 Introduction Takotsubo cardiomyopathy is characterized by an acute transient cardiomyopathy. Although the etiology of this syndrome is unknown, however, stressful situations (physical or emotional) are postulated to be the most common trigger. It is most commonly seen in postmenopausal women. It was described first by Sato et al. in 1990 [1].

32.2.2 Pathophysiology Stressful situation leads to activation of the sympathetic nervous system causing excess catecholamine release and stimulation of cardiac adrenergic system, which causes contractile

dysfunction and neurocardiogenic stunning of the electrophysiological system of the heart [2, 3].

32.2.3 Natural History It is generally regarded as a relatively benign disease. Following the acute presentation, the prognosis is generally very good, with complete resolution occurring in a number of weeks to months [4, 5].

32.2.4 Diagnostic Criteria Mayo Clinic Diagnostic Criteria (All four points required) [6, 7] (i) Transient LV systolic dysfunction: RWMA → extends beyond a single epicardial coronary distribution. (ii) Absence of obstructive CAD. (iii) Elevation in cardiac troponin or new ECG abnormalities (either ST-segment elevation and/or T wave inversion) or (iv) Absence of pheochromocytoma or myocarditis.

32.2.5 Role of MRI in a Suspected Case of Takotsubo Cardiomyopathy [1] (i) Diagnosis and evaluation of stress cardiomyopathy (when echo is technically suboptimal). (ii) Delineate extent of ventricular abnormalities.

R. Khurana

486

(iii) Identify complications. (iv) Prognostication.

32.2.6 Types of Takotsubo Cardiomyopathy (i) Apical (Typical) (ii) Mid Ventricular ballooning (iii) Basal ballooning (iv) Biventricular pattern (v) Focal ballooning pattern

32.2.7 Role of Strain Imaging Strain values are deranged in the involved segments. It improves over time. In a study by Stiermaier et al., a GLS poorer than −14.75% was shown to be a poor prognostic marker [8].

32.2.8 LGE/Fibrosis in Case of Takotsubo Cardiomyopathy Conventional literature on imaging in TC suggests that there is no LGE (using a 5SD threshold) in the involved myocardium in contrast to acute myocarditis or Acute MI. Presence of LGE in patients with TC is an adverse prognostic marker [9].

32.2.9 Estimation of Scar Tissue Scar tissue is estimated using mean + 5SD method (Circle Cvi 42 software)

32.2.10 Complications See Table 32.1

32.2.11 Independent Predictors of Long-Term Mortality Presence of LGE, heart failure (class 3/4 on admission), cardiogenic shock life threatening arrythmia, systemic embolism

Table 32.1 Various complications in Takotsubo cardiomyopathy along with their frequency of occurrence [9] Complication (frequency) Pericardial effusion (~57%) Functional MR (~25%) Dynamic LVOT obstruction (~57%) Heart failure, class III/IV (~13%) RV involvement (~15–22%) Cardiogenic shock (~10–15%) Life-threatening arrhythmias (~13.5%) Thrombus formation (~2.5%) Systemic embolism (56 ms had a negative predictive value of 97% to detect acute cellular rejection [4].    – Patchy LGE- marker of poor prognosis [5].  • Gold standard for monitoring rejection.  • Complications high and invasive. No significant role. No significant role.

Endomyocardial biopsy SPECT/PET IVUS/OCT

Cardiac allograft vasculopathy  • Recommended for surveillance by ISHLT.  • Detects luminal stenosis.  • No information on arterial wall thickness.  • Detects luminal stenosis + wall thickness.  • Detects early disease.    – Meta-analysis by Pinzon et al- CT had >90% sensitivity, specificity, and negative predictive value for detecting significant stenosis (>50%) of CAV [3].  • Pharmacological/dobutamine stress.  • Echocardiography to detect ischemia.  • Ischemia—perfusion imaging.  • Infarct-subendocardial LGE.

No significant role for detecting vasculopathy in epicardial coronary arteries.  • Perfusion abnormalities in significant vasculopathy.  • Detect intimal thickening earlier than other modalities.    – IVUS Wall thickening >0.5mm on IVUS is predictor of poor outcome at 5 years [6].

V. Ojha

494

33.4.4 Predictors of Poor Prognosis and Long-­ Term Mortality on CMR Reduced left ventricular ejection fraction Late gadolinium enhancement Deranged left ventricular global longitudinal strain Raised T1, T2 and ECV mapping values [7]

33.4.5 Problems with Imaging of Heart Transplant Recipients with CT High heart rate, nonresponsive to beta-blockers due to vagal denervation [1]—Dual source CTA can overcome this problem because of its high temporal resolution. Risk of contrast-induced nephropathy due to deranged renal functions because of nephrotoxic medication Radiation exposure—Overcome by advanced radiation dose reduction techniques

Questions 5: Gold standard for ACR (a) Biopsy (b) MRI (c) Both (d) None Questions 6: Recommended imaging modality for CAV is (a) Invasive coronary angiography (b) CT (c) Both (d) None Question 7: Sign of ACR on MRI? (a) Increased T2 map value (b) Decreased T2 map value (c) Both (d) None Answer 1 a

2 b

3 c

4 a

5 a

6 a

7 a

33.4.6 How Imaging Impacts Management? A diagnosis of acute cellular rejection warrants treatment References with steroids and sometimes, higher doses of immuno-­ 1. Olymbios M, Kwiecinski J, Berman DS, Kobashigawa suppressants or sometimes, intravenous immunoglobulins. JA.  Imaging in heart transplant patients. J Am Coll Cardiol Img. A diagnosis of CAV warrants changes in immunosuppres2018;11(10):1514–30. https://doi.org/10.1016/j.jcmg.2018.06.019. sion and palliative PCI (if limited to one artery). Some 2. Mehra MR, Crespo-Leiro MG, Dipchand A, et  al. International Society for Heart and Lung Transplantation working formulation patients may require re-transplantation. Multiple Choice Questions Question 1: CAV is (a) Diffuse (b) Focal (c) None (d) Both Question 2: CAV is (a) Acute (b) Chronic (c) Mostly acute (d) After 50 years of transplant Question 3: MRI detects (a) Acute rejection (b) CAV (c) Both (d) None Question 4: CT detects (a) CAV (b) ACR (c) Both (d) None

3.

4.

5.

6.

7.

of a standardized nomenclature for cardiac allograft vasculopathy2010. J Heart Lung Transplant. 2010;29(7):717–27. https://doi. ­ org/10.1016/j.healun.2010.05.017. Wever-Pinzon O, Romero J, Kelesidis I, et  al. Coronary computed tomography angiography for the detection of cardiac allograft vasculopathy: a meta-analysis of prospective trials. J Am Coll Cardiol. 2014;63(19):1992–2004. https://doi.org/10.1016/j. jacc.2014.01.071. Marie PY, Angioï M, Carteaux JP, et  al. Detection and prediction of acute heart transplant rejection with the myocardial T2 determination provided by a black-blood magnetic resonance imaging sequence. J Am Coll Cardiol. 2001;37(3):825–31. https://doi. org/10.1016/s0735-­1097(00)01196-­7. Pedrotti P, Vittori C, Facchetti R, et  al. Prognostic impact of late gadolinium enhancement cardiac magnetic resonance in the risk stratification of heart transplant patients. J Cardiovasc Magn Reson. 2016;18(1):O63. https://doi.org/10.1186/1532-­429X-­18-­S1-­O63. Kobashigawa JA, Tobis JM, Starling RC, et al. Multicenter intravascular ultrasound validation study among heart transplant recipients: outcomes after five years. J Am Coll Cardiol. 2005;45(9):1532–7. https://doi.org/10.1016/j.jacc.2005.02.035. Vermes E, Pantaléon C, Auvet A, et al. Cardiovascular magnetic resonance in heart transplant patients: diagnostic value of quantitative tissue markers: T2 mapping and extracellular volume fraction, for acute rejection diagnosis. J Cardiovasc Magn Reson. 2018;20(1):59. https://doi.org/10.1186/s12968-­018-­0480-­9.

Part III Cardiac Masses

Imaging Approach to Cardiac Masses

34

Mumun Sinha and Mansi Verma

34.1 Case 34.1 34.1.1 Clinical History A fifty-two-year-old female, with no prior comorbidities, presented with gradual onset, progressive dyspnea on exertion (DOE) of 3 months duration, low-grade fever, and generalized fatigue. The DOE increased on lying down. Clinical examination: Diastolic rumble at the apex in the left lateral position. No opening snap. General physical examination unremarkable.

34.1.2 Investigations 34.1.2.1 Electrocardiogram (ECG) Normal 34.1.2.2 Chest Radiography (CXR) Grade II pulmonary venous hypertension. Lung parenchyma—normal. No dilatation of the pulmonary arteries (PA), left atrial appendage (LAA), or ascending aorta. 34.1.2.3 Inflammatory Markers ESR mildly elevated, CRP and WBC count normal In this background, differential diagnosis to be considered are Rheumatic heart disease Left-sided cardiac mass, possibly left atrial Diastolic dysfunction Idiopathic restrictive cardiomyopathy, Pericardial thickening Congenital obstructive mitral valvular and subvalvular lesions

M. Sinha (*) · M. Verma Department of Cardio Vascular Radiology and Endovascular Interventions, AIIMS, Ansari Nagar, New Delhi, India

Pulmonary vein stenosis, Pulmonary veno-occlusive disease

34.1.2.4 Echocardiogram A large, globular mass in the left atrium (LA) attached to the interatrial septum, protruding into the left ventricle during diastole. Continuous wave Doppler showed left ventricle inflow obstruction at the level of the mitral valve with mild tricuspid regurgitation and moderate pulmonary arterial hypertension.

34.1.3 Provisional Diagnosis LA myxoma (to exclude thrombus)

34.1.4 What Investigation Will You Do Next? Cardiac MR or CT or Both? Localization Assessment of the extent of involvement Evaluation of the hemodynamic consequence / significance of of the lesion

Tissue characterization including morphology with thromboembolic consequences

MRI or CT MRI = CT MRI > CT for confirmation of mural, pericardial, and mediastinal invasion MRI > CT for demonstration of tumor movement, quantification of mitral valvular obstruction/ regurgitation, regional wall motion abnormality, and cardiac function MRI > CT because of better soft tissue resolution, superior assessment of fat, hemorrhage, hypercellularity (elevated perfusion and diffusion restriction) CT better for assessment of calcification

With this knowledge of the role of both modalities, we use Contrast-Enhanced MR (Fig. 34.1) to derive the relevant information. The patient also underwent Cardiac CTA (Fig. 34.2) for assessment of coronaries.

© The Author(s), under exclusive license to Springer Nature Singapore Pte Ltd. 2023 S. Sharma (ed.), Case-based Atlas of Cardiac Imaging, https://doi.org/10.1007/978-981-99-5620-3_34

497

498

M. Sinha and M. Verma

a

b

c

d

e

f

g

h

i

j

k

l

Fig. 34.1 (a, b) T1-weighted non-fat-saturated images in the short axis and four chamber views show a left atrial mass that is isointense to the myocardium. (c) The mass appears hyperintense on T2-weighted fat-saturated image in the four chamber view. (d) Steady-State Free Precession (SSFP) image in the short axis view shows that the mass is isointense to the blood pool. (e, f) Steady-State Free Precession (SSFP)

image in the two chamber view shows prolapse of the mass into the left ventricular inlet. (g–i) First-pass perfusion images in the short axis views show no perfusion. (j–l) Phase-Sensitive Inversion Recovery (PSIR) images (at 15 min post Gadolinium injection) in the short axis, four chamber, and two chamber views, respectively, show heterogeneous enhancement in the mass

34  Imaging Approach to Cardiac Masses

a

499

b

Fig. 34.2 (a, b) Cardiac computed tomography (CTA) images in the four-chamber view show a mobile left atrial mass attached to the interatrial septum and prolapsing across the mitral valve orifice during diastole

34.1.5 Is There a Role of Cardiac CT?

34.2 Case 34.2: Left Atrial Thrombus

Yes,

See Fig. 34.3.

1. To rule out the presence of significant coronary artery disease in low- to intermediate-risk patients prior to the surgery 2. To look for the presence of calcification in the mass 3. To look for lung parenchymal abnormalities

• Smaller than myxomas, broad based, less mobile • More commonly homogenously hypointense on T1 and T2 • Located along the posterior or lateral wall of left atrium or atrial appendage in cases of rheumatic heart disease (RHD), in the dilated right atrium (RA), in the ventricle with infarct, aneurysm/pseudoaneurysm or endomyocardial fibrosis (EMF) • May demonstrate calcification • Enhancement uncommon unless organized • “Long inversion time” (550–650 ms) PSIR sequence produces an image on which the thrombus appears black and surrounding myocardium bright

This comes at the cost of exposure to ionizing radiation and lower soft-tissue contrast resolution compared with MR imaging.

34.1.6 Differential Diagnosis of Left Atrial Myxoma 34.1.6.1 Left Atrial Thrombus Thrombus, Paraganglioma, Hemangioma, Lipoma, Inflammatory/Infective masses, Secondaries Malignant primary cardiac tumors are usually located in the right atrium and are most commonly angiosarcomas.

500

M. Sinha and M. Verma

a

b

c

d

e

f

Fig. 34.3 (a, b) Cardiac coronary computed tomography images show a filling defect in the left atrial appendage. (c) Cardiac coronary computed tomography images in the four-chamber view show a hypodense mass with broad base of attachment to the posterior LA wall. (d, e) Steady-State Free Precession (SSFP) image in the four-chamber view

34.3 Case 34.3: Primary Malignant Atrial Sarcoma See Fig. 34.4. • Typically seen in the right atrium, may extend into the LA, pericardium, or are primary pericardial. • Can have hemorrhagic pericardial effusion.

shows a nonmobile mass that is hypointense to the blood pool. (f) Long inversion time Phase-Sensitive Inversion Recovery (PSIR) sequence produces an image in which the thrombus appears black and surrounding myocardium bright

• Sarcomas other than angiosarcoma are more commonly seen in the LA. • Leiomyosarcomas can cause obstruction of pulmonary venous inflow. • Cardiac MR or CT can be useful to demonstrate sarcoma-­ associated vascularity. When the tumor outgrows its vascular supply, there can be multiple areas of necrosis within the tumor.

34  Imaging Approach to Cardiac Masses

501

a

b

c

d

Fig. 34.4 (a) T2-weighted fat-saturated image in the four-chamber view shows a left atrial mass that is hyperintense to the myocardium. (b, c) SSFP image in the two and four chamber views shows that the mass invades into the pulmonary vein orifice and is hypointense to the blood

pool. Associated pleural and pericardial effusion present. (d) PSIR image in the four chamber view shows heterogeneous enhancement in the mass

502

M. Sinha and M. Verma

34.4 Case 34.4: Paraganglioma

• On cardiac MR, they appear intensely hyperintense on T2-weighted images, with areas of T1 hyperintensity suggestive of hemorrhage.

See Fig. 34.5. • Make up 2% of all paragangliomas. • At echocardiography, paragangliomas appear large and echogenic with broad base of attachment. a

b

c

A

P

*

d

e

A

P

Fig. 34.5 (a) SSFP image in the axial plane shows a hypointense mass in the left atrium. (b) The mass is intensely bright on T2 weighted image. (c) Perfusion images show avid first-pass enhancement (*) almost equal to the blood pool in the ascending aorta (A) and main

pulmonary artery (P). (d, e) CTA images show intense arterial enhancement within the mass almost equal to the blood pool in the ascending aorta (A) and main pulmonary artery (P)

34  Imaging Approach to Cardiac Masses

503

34.5 Case 34.5: Lipoma

• On CT, seen as low signal attenuation (i.e., HU density ≤ −30) and appears dark. • On MR, fat appears bright on T1-weighted imaging, hypointense on fat-suppressed sequence.

See Fig. 34.6. • Lipomatous hypertrophy of IAS extends to the adjacent LA or RA wall with sparing of the fossa ovalis.

a

b

c

Fig. 34.6 (a–c) Contrast CT and CTA images respectively show thickening of the interatrial septum with attenuation similar to fat

504

M. Sinha and M. Verma

34.6 Case 34.6: Papillary Fibroelastoma

• Seen as mobile, pedunculated masses with papillary projections.

See Fig. 34.7. • Commonest tumors of cardiac valves. • Aortic and mitral valve are commonly involved. a

b

c

d

e

f

g

Fig. 34.7 (a, b) CTA images in the two and four chamber views show a hypodense papillary lesion attached to the mitral leaflet. (c, d) SSFP image in the four-chamber view shows a hypointense papillary lesion attached to the mitral leaflet and projecting into the left atrium. (e, f)

T2-weighted and T1-weighted images in the four-chamber view show intermediate signal intensity polypoidal lesion involving the mitral valve. (g) PSIR images in the four-chamber view show enhancement of the mass

34  Imaging Approach to Cardiac Masses

34.7 Imaging Features of Cardiac Myxoma 34.7.1 Radiograph • Findings vary with tumor location. There can be left atrial enlargement, pulmonary edema. • Cardiomegaly and pleural effusions may occur with either right-sided or left-sided myxomas. • Calcification has been reported in 50% of right atrial myxomas, rarely seen in left atrial myxomas. • One-third chest radiographs normal.

34.7.2 Echocardiography • Homogeneous or heterogeneous appearance, with echogenic foci due to calcification and hypoechoic areas due to hemorrhage or necrosis. • Attached to the interatrial septum, mobile with prolapse during diastole.

34.7.3 Computed Tomography: Done Primarily for Coronary Evaluation in Low to Intermediate CAD Risk • Located at or near fossa ovalis. • Heterogeneous appearance due to hemorrhage, necrosis, calcification.

505

• Cine sequences demonstrate prolapse into the ventricular cavity depending on length of stalk. • Secondary complications—pulmonary, coronary, or visceral emboli.

34.7.4 Magnetic Resonance Imaging See Fig. 34.8. • T1-weighted sequence—isointense • T2-weighted sequence—hyperintense • May appear heterogeneous due to hemorrhage, calcification, necrosis, myxoid, or fibrous tissue. • GRE sequence—loss of signal intensity due to high iron content or calcifications. • SSFP sequence—hypointense to isointense to the blood pool and hyperintense to the myocardium. • Perfusion—may or may not show elevated first-pass perfusion Post contrast sequence—heterogenous enhancement. • Cine Sequence—prolapse of the tumor across cardiac valve.

34.7.5 Management • Surgical resection is the treatment of choice. • Damaged valves may require repair or prosthetic replacement.

506

M. Sinha and M. Verma

a

b

c

d

e

f

Fig. 34.8 (a) T1-weighted non-fat-saturated image in the short axis plane shows a well-defined mass in the right ventricle that is isointense to the myocardium with narrow base attachment to the IVS. (b) T2-weighted fat-saturated image in the short axis view shows that the mass is hyperintense to the myocardium with hypointense areas, which

could represent hemorrhage or calcification. (c, d) The mass is hyperintense to the myocardium on SSFP sequence images. (e, f) PSIR image in the four-chamber and short axis views show heterogeneous enhancement of the mass. Nonenhancing areas could represent thrombi, hemorrhage, or calcification

34  Imaging Approach to Cardiac Masses

507

34.8 Case 34.8

34.8.3 Provisional Diagnosis

34.8.1 Clinical History

Malignant right atrial mass We use Contrast-Enhanced MR (Fig. 34.9) to derive the relevant information after pericardial tapping. Is there a role of cardiac CT (Fig. 34.10)? Yes,

A 30-year-old female, with no prior comorbidities, presented with recent onset (1 month duration) easy fatigability, right upper quadrant pain, significant weight loss, and bilateral pedal edema. Examination revealed raised JVP and tender hepatomegaly.

34.8.2 Investigations 34.8.2.1 Electrocardiogram (ECG) Normal 34.8.2.2 Chest X-Ray Double density with normal carinal angle. In this background, differential diagnosis to be considered are: 1. Mediastinal mass with large right atrium and possibly right heart failure 2. Primary infiltrating right atrial mass with extramural component 3. RV endomyocardial fibrosis with thrombus in right atrium

34.8.2.3 Echocardiogram A large infiltrating and obliterating right atrial lesion with color flow within. Moderate pericardial effusion present.

1. To look for coronary artery infiltration 2. To rule out the presence of significant coronary artery disease in low- to intermediate-risk patients prior to the surgery 3. To look for the presence of calcification in the mass 4. To look for lung parenchymal metastasis

34.8.4 Diagnosis Right atrial angiosarcoma

34.8.5 Differential Diagnosis Right atrial myxoma Tuberculoma Lymphoma RA metastasis Mediastinal mass infiltrating the right atrium

508

M. Sinha and M. Verma

a

b

c

d

e

f

Fig. 34.9 (a) T1-weighted non-fat-saturated images in the four-­ chamber view show a heterogeneous expansile intermediate signal intensity mass in the right atrium. (b) The mass appears hyperintense on T2- weighted fat-saturated images. (c) PSIR images in the four-­chamber

view show heterogeneous enhancement with nonenhancing areas. (d) Upper abdominal sections show metastatic liver lesions. (e, f) Diffusionweighted and apparent diffusion coefficient images show true restriction within the mass

34  Imaging Approach to Cardiac Masses

509

a

b

c

d

Fig. 34.10  Contrast-enhanced CT (a–c) images show a heterogeneous expansile mass in the right atrium. (d) CTA images reveal that the mass is separate from the right coronary artery

510

M. Sinha and M. Verma

34.9 Case 34.9: Right Atrial Myxoma See Fig. 34.11.

a

b

Fig. 34.11 (a, b) CTA images show a lobulated mobile mass in the right atrium prolapsing across the tricuspid valve

34  Imaging Approach to Cardiac Masses

511

34.10 Case 34.10: Tuberculoma See Fig. 34.12.

a

Fig. 34.12 (a) PSIR four chamber image shows a heterogeneously enhancing mass in the right atrium with broad base of attachment to the right atrial wall with associated pericardial thickening and enhance-

b

ment. (b) CT chest shows the enlarged mediastinal lymph nodes subsequently proven to be tubercular

512

M. Sinha and M. Verma

34.11 Case 34.11: Cardiac Lymphoma See Fig. 34.13. Cardiac lymphomas are seen as infiltrative masses with associated pericardial effusion.

a

b

c

d

Cross-sectional imaging is necessary for further characterization of the tumor, extent of involvement, and extracardiac spread.

e

Fig. 34.13 (a, b) CT chest shows multiple enlarged mediastinal lymph nodes. (c, d) Images at the level of heart show infiltrative soft tissue along the interatrial septum, left atrium, pulmonary veins, and posterior mediastinum. (e) Findings are confirmed on MR images

34  Imaging Approach to Cardiac Masses

34.12 Case 34.12: Right Atrial Thrombus See Fig. 34.14. MR Appearance - Homogenously hypointense on T1 and T2,

a

513

“Long inversion time” (550–650 ms) PSIR sequence produces an image on which the thrombus appears black and surrounding myocardium bright. Risk factors for RA thrombus are endomyocardial fibrosis, mechanical valves, pacemaker leads, ventricular or atrial septal closure devices, and indwelling central venous lines.

b

c

Fig. 34.14 (a) SSFP four chamber image shows a free thrombus in the right atrium. (b) SSFP four chamber image shows adherent thrombus in the right atrium. (c) SSFP four chamber image shows adherent throm-

bus in the giant right atrium. In addition, there are features of endomyocardial fibrosis in the form of obliteration of the RV apex, pericardial and right pleural effusion

514

M. Sinha and M. Verma

34.13 Case 34.13: Cardiac Angiosarcoma

34.13.3 Computed Tomography

34.13.1 Radiography

Done primarily for coronary evaluation and infiltration

Cardiomegaly with right-sided heart enlargement, Widening of mediastinum

34.13.4 Magnetic Resonance Imaging

34.13.2 Echocardiography Infiltrative, nonmobile mass with associated pericardial effusion

a

b

d

e

Fig. 34.15 (a, b) SSFP axial and sagittal image shows an expansile mass in the right ventricle with pericardial and pleural effusion. (c) PSIR axial images show heterogeneous enhancement of the mass with another small nonenhancing thrombus in the right atrium. (d) CTA

See Fig. 34.15. MR Appearance - Large infiltrative mass with heterogeneous signal intensity with areas of high T1 signal suggestive of hemorrhage and signal void suggestive of vascular channels. Associated hemorrhagic pericardial effusion. First-pass and LGE show avid enhancement.

c

images show dilated right atrium and expansile right ventricular mass. (e) Delayed images show enhancement of the half of the lesion. Rest of the right ventricular lesion and lesion in the right atrium are nonenhancing, likely thrombi

34  Imaging Approach to Cardiac Masses

515

34.14 Miscellaneous Tumors

MR shows homogeneous and well-circumscribed masses isointense to myocardium on T1-weighted images and slightly hyperintense on T2-weighted images. Perfusion is not elevated. No differential enhancement on LGE images. Most of the tumors should be followed, usually with echo or MR as there are high chances of spontaneous regression. The differential diagnosis of a neonatal cardiac mass include rhabdomyoma, fibroma, teratoma, and rhabdomyosarcoma. Rhabdomyomas are typically multifocal and lack the calcification and T2 hypointensity of fibromas, or the heterogeneity of teratomas and rhabdomyosarcomas.

34.14.1 Case 34.14: Rhabdomyoma See Fig. 34.16. Rhabdomyoma is the most common primary benign cardiac tumor in pediatric patients, highly associated with tuberous sclerosis Almost always arise in ventricular myocardium Tumor is seen as echogenic mass on echocardiography. CT shows mass of similar attenuation to myocardium on noncontrast and postcontrast images.

a

b

c

Fig. 34.16 (a, b) T1-weighted image in the four chamber view and SSFP image in the short axis view show a homogeneous mass in the interventricular septum that is isointense to myocardium. (c) PSIR image shows no differential enhancement

516

34.14.2 Case 34.15: Fibroma See Fig. 34.17. Cardiac fibroma is a congenital neoplasm or hamartoma of fibrous tissue. They are the second most common tumor in childhood after rhabdomyoma. Gorlin syndrome is an association. Symptoms include heart failure, arrhythmia, chest pain, syncope, and sudden death or incidentally diagnosed. Echocardiography shows a discrete mass or a focal area of echogenic wall thickening.

a

M. Sinha and M. Verma

On CT, the mass shows soft tissue attenuation and calcification with variable enhancement. On MRI, a cardiac fibroma appears as an intramural mass or focal myocardial thickening on T1-weighted sequences, isointense or hypointense to myocardium. It is hypointense on T2-weighted and SSFP sequences because of fibrous tissue composition and shows no enhancement on perfusion images. There is marked delayed enhancement of the mass.

b

Fig. 34.17 (a) T1-weighted image in the four chamber view shows focal myocardial thickening that is hypointense to myocardium. (b) PSIR image show marked delayed enhancement

34  Imaging Approach to Cardiac Masses

517

34.15 Case 34.16: Calcified Amorphous Tumor See Fig. 34.18. Calcified amorphous tumor is a rare, nonneoplastic, diffusely calcified mass of uncertain etiology. Its prevalence is extremely low.

a

b

Most cases of calcified amorphous tumor are pedunculated intracavitary masses. It can also involve the myocardium, papillary muscles, or chordae. The clinical presentation includes heart failure, syncope, and evidence of embolization. Chest radiograph may show dense calcification within the cardiac silhouette.

c

Fig. 34.18 (a, b) T1- and T2-weighted images in the short axis view show a hypointense myocardial mass. (c) CT images demonstrate calcification within the mass

M. Sinha and M. Verma

518

Multiple Choice Questions Question 1: What is the most common type of mass seen in the heart? (a) Myxoma (b) Angiosarcoma (c) Thrombus (d) Rhabdomyoma (e) Melanoma Answer 1 c

Appendix 1 CT Angiography Imaging Protocol Retrospective ECG-gated CT angiography examination is performed after injection of nonionic iodinated contrast 1.0  mL/kg body weight via a peripheral intravenous line using a dual head power injector at a flow rate of 4.5 mL/s followed by a 50-mL saline chase injected at the same flow rate. A ‘bolus tracking’ method is used whereby CT acquisition is automatically triggered when contrast opacification threshold of 100 Hounsfield units (HU) is achieved in the ascending aorta on the monitoring sequence. A delayed scan is acquired to confirm the presence of lesion and look for contrast characteristics.

Cardiac MRI Protocol Scout Turbo Spin Echo—4 chamber and Short Axis—T1 non-­ fat-­saturated and T2 fat-saturated Cine SSFP—Short Axis, 4-Ch, 2 Ch (Additional RVOT and LVOT view according to tumor location) Diffusion-weighted image and Apparent diffusion coefficient map of the tumor/mass Dynamic perfusion Delayed Gadolinium Enhancement—5 and 15  min— Short Axis, 4-Ch, 2 Ch (Additional RVOT and LVOT view according to tumor location). High TI sequence to look for/ rule out thrombus

Appendix 2 Reporting Points Location Extension (intracardiac and extracardiac) Precontrast scan: Signal Intensity on T1/T2/SSSP— hyper-/iso-/hypointense to the myocardium heterogenous/ homogenous; Attenuation on precontrast CT scan—presence of calcification. Cine: prolapse of the lesion across the atrioventricular valve and semilunar valves, Vascular stenosis and regurgitation, ventricular ejection fraction, and regional wall motion abnormalities. Dynamic perfusion on CEMR; first-pass CT perfusion—increased perfusion Delayed Gadolinium Enhancement—homogenous/heterogeneous/no enhancement. High TI sequences—thrombus Diffusion restriction for malignant lesions

Part IV Coronary Artery Anomalies

Imaging in Anomalies of Coronary Artery Origin

35

Sreenivasa Narayana Raju

35.1 Case 35.1 35.1.1 Clinical History Two-month old infant presented with dyspnea and feeding difficulty since 2 weeks. On examination, there was continuous to and fro murmur over left upper sternal border

35.1.2 Chest X-Ray Showed cardiomegaly with inferior and outward apex s/o left ventricular enlargement. Splaying of the carina with double density sign s/o left atrial enlargement. Cephalization of the pulmonary vascular marking s/o pulmonary venous hypertension (Fig. 35.1). In an acyanotic child, CXR with cardiomegaly, LV & LA enlargement, and features of pulmonary venous hypertension—following differentials need to be considered: • Anomalous origin of left coronary artery from the pulmonary artery (ALCAPA) • Interrupted aortic arch with left ventricular failure • Congenital renal artery stenosis with left ventricular failure • Congenital mitral regurgitation • Primary myocardial disease (myocarditis, dilated cardiomyopathy)

35.1.3 ECG Pathological “Q” and ST/T changes suggestive of myocardial ischemia.

Fig. 35.1  Chest X-ray: Cardiomegaly with inferior and outward apex s/o left ventricular enlargement. Splaying of the carina with double density sign s/o left atrial enlargement. Cephalization of the pulmonary vascular marking s/o pulmonary venous hypertension

35.1.4 Echocardiography Global left ventricular hypokinesia and decreased LV function with left atrial dilation.

35.1.5 Provisional Diagnosis ALCAPA

S. N. Raju (*) Department of Cardio Vascular Radiology and Endovascular Interventions, AIIMS, Ansari Nagar, New Delhi, India © The Author(s), under exclusive license to Springer Nature Singapore Pte Ltd. 2023 S. Sharma (ed.), Case-based Atlas of Cardiac Imaging, https://doi.org/10.1007/978-981-99-5620-3_35

521

S. N. Raju

522

35.1.6 What Investigation Will You Do Next? 35.1.6.1 Goals of Imaging in Coronary Anomalies Catheter angiography can provide coronary anatomic information to a large extent; however, it is invasive. Although both Cardiac CT and MRI have the ability of multiplanar reconstruction with ECG-gated acquisition, cardiac CT has emerged as a better noninvasive imaging modality for evaluation of anomalous coronaries as it allows for rapid scan acquisition with superior temporal and spatial resolution in addition to providing following advantages:

a

• Accurate evaluation of origin, course, and termination of the anomalous coronary arteries, including the inter-­ coronary collaterals in Bland White Garland Syndrome. • Precise depiction of the spatial relation of coronary artery in relation to the adjacent structures in volume rendered images pertinent for surgical approach. • Cardiac CT is excellent modality for assessment of the coronary wall. This patient underwent ECG-gated CT angiography (CTA) (Fig. 35.2).

b

MPA

MPA

Le Ventricle

c

d

Le Ventricle

Le Ventricle

Fig. 35.2  Cardiac CT findings: (a, b) Anomalous origin of the left main coronary artery from the right lateral wall of the main pulmonary artery (arrow). (c) Normal right coronary (arrow head) (d) Dilated left ventricle with diffuse subendocardial hypodensity (arrow head)

35  Imaging in Anomalies of Coronary Artery Origin

523

35.1.7 Final Diagnosis Infant ALCAPA.

35.1.8 Differential Diagnosis Includes the Following 1. Interrupted aortic arch with left ventricular failure (Case 35.2) (Fig. 35.3) There is a separation between the ascending and descending aorta with a VSD and or PDA, which are frequently present. Early LV failure can be present due to a

b

AA

c

left ventricle outflow tract obstruction and increase in afterload. 2. Congenital renal artery stenosis with left ventricular failure (Case 35.3) (Fig. 35.4) LV failure is seen due to increase in the afterload secondary to activation of the renin angiotensin pathway 3. Coarctation with LV dilatation (Case 35.4) (Fig. 35.5) Significant stenosis or diffuse hypoplasia of the aorta, distal to the origin of brachiocephalic artery upto the ductus arteriosus in infantile form or a short segment stenosis of involving postductal aorta in adult form. LV hypertrophy and failure can be present due to increase in afterload.

DTA

d

Fig. 35.3  Cardiac CT findings: (a) Type A interruption (arrow), (b) Normal origin of the left main coronary artery (arrow head), (c, d) LV hypertrophy and dilatation in the short axis and four chamber view

respectively (arrow). Final Diagnosis: Type A Interrupted aortic arch with left ventricular failure

S. N. Raju

524

a

b

Aorta

Le Ventricle

c

d

Fig. 35.4  Cardiac CT findings: (a) significant right renal artery stenosis (arrow). (b) Dilatation and hypertrophy of left ventricle. (c, d) Normal origin of the left and right coronary arteries from the left and right coronary sinus (arrow)

35  Imaging in Anomalies of Coronary Artery Origin

a

525

b

d

c

Left Ventricle

Left Ventricle

Fig. 35.5  Cardiac CT findings: (a) Coarctation of the aorta (aorta). (b) Normal origin of the right (arrow) and left coronary artery (arrow head), (c, d) dilatation of the Left ventricle in the short axis and four chamber views respectively

35.2 Case 35.5 35.2.1 Clinical History Thirty-year-old man presented with atypical chest pain and dyspnea on exertion since 3  months. Echocardiography

revealed mild global LV hypokinesia and LV dysfunction. In addition, echo revealed anomalous vessel inserting into main pulmonary artery (PA) with multiple large collaterals within the interventricular septum (IVS) on color Doppler. This patient underwent an ECG-gated CT angiography (CTA). Pertinent images are shown in Fig. 35.6.

S. N. Raju

526

a

b

RCS NCS

MPA

LCS

c

d

Le Ventricle

e

f

MPA Le Ventricle

Fig. 35.6  Cardiac CT findings: (a) Normal right coronary aortic sinus (arrow), (b) Anomalous origin of the left main coronary artery from the right lateral wall of the main pulmonary artery (arrow head), (c) Left main coronary along the anterior wall of left ventricle (arrow head), (d)

Hypertrophied septal and (e) epicardial inter coronary collaterals with dilated left ventricle (arrow), (f) VRT images showing anomalous origin of the left coronary artery from pulmonary artery

35  Imaging in Anomalies of Coronary Artery Origin

35.2.2 Final Diagnosis: Adult ALCAPA Catheter angiograms are not routinely preformed in morphologic evaluation of ALCAPA in infants; however, it is has critical role in the preoperative evaluation of the coronary artery disease in patients with indeterminate lesions on coronary CT angiography. Pertinent catheter angiography images depicting the anatomic abnormalities in adult ALCAPA are described in Fig. 35.7.

35.2.3 Differential Diagnosis of Diffuse Coronary Artery Dilatation Includes 1. Coronary artery dilatation related to atherosclerosis: Diffuse coronary artery dilatation with atherosclerotic plaque in affected coronary arteries

527

2. Kawasaki Disease: History of viral infection in young adults with diffuse or focal multiple coronary artery aneurysms 3. Coronary artery-coronary sinus fistula: Dilatation and tortuosity of coronary artery feeding the arteriovenous communication, associated with dilatation of the draining epicardial veins and coronary sinus 4. Takayasu arteritis: Stenosis and aneurysm of the coronary arteries in the background of aortic and arch vessel involvement

S. N. Raju

528

a

b

c

Fig. 35.7  Catheter angiogram: (a) Aortic root injection shows single coronary (RCA) and absent left main coronary. (b) Selective RCA injection showing dilated ectatic RCA, with retrograde filling of the left

main coronary via the multiple collaterals. (c) On delayed angiographic frames, opacification of the main pulmonary artery via the left main coronary is seen confirming

529

35  Imaging in Anomalies of Coronary Artery Origin

35.3 Anomalous Origin of the Left Coronary Artery from Pulmonary Artery

35.3.2 Embryology Two theories [1, 2]:

35.3.1 Introduction

1. Abrikossoff theory: The bulbus cordis undergoes abnormal septation into the aorta and pulmonary trunk. Anomalous origin of the left coronary artery from the pul- 2. Hackensellner’s theory: Persistence of the pulmonary monary artery (ALCAPA) syndrome, aka Bland-White-­ buds with concomitant involution of the aortic buds that Garland syndrome, is a rare congenital anomaly seen in 1 in are precursors of the coronary arteries. 300,000 live births and constitutes for 0.25–0.5% of all congenital cardiac defects. It typically presents as an isolated defect, but in 5% of cases, it may also be linked to other 35.3.3 Pathophysiology of ALCAPA cardiac defects including an aortic coarctation, atrial septal defect, or ventricular septal defect [1]. Neonatal and adult ALCAPA is described in Fig. 35.8. Fig. 35.8 Pathophysiology of neonatal and adult ALCAPA

ALCAPA Neonatal period Pressure in PA=Ao Antegrade flow in the LCA

No Symptoms Develops in two types based on the ability to develop collaterals between the RCA and LCA

Infant Type

Adult type

Pressure in PA1 mm (b) Any bridging >5 mm (c) Bridging causing systolic widening of coronary lumen (d) Bridging causing diastolic compression of coronary lumen

S. N. Raju

566

Question 9: All of the surgeries are preferred in the surgical correction of the malignant interatrial course except (a) Coronary unroofing (b) Reimplantation (c) CABG (d) Takeuchi repair Questions 10: Which of the following surgery is preferred in the correction of high take-off RCA? (a) Single coronary system repair (b) Benign course, no intervention is required (c) Coronary button transfer (d) Takeuchi repair Answers 1 d

2 c

3 d

4 b

5 d

6 d

7 d

8 a

9 d

10 b

References 1. Hauser M.  Congenital anomalies of the coronary arteries. Heart. 2005;91(9):1240–5. 2. Paolo A.  Coronary artery anomalies. Circulation. 2007;115(10):1296–305. 3. Kim SY, Seo JB, Do K-H, Heo J-N, Lee JS, Song J-W, et al. Coronary artery anomalies: classification and ECG-gated multi–detector row CT findings with angiographic correlation. Radiographics. 2006;26(2):317–33. 4. Vohra A, Narula H. Dual left anterior descending artery with anomalous origin of long LAD from pulmonary artery—rare coronary anomaly detected on computed tomography coronary angiography. Indian J Radiol Imaging. 2016;26(2):201. 5. Rahalkar AM, Rahalkar MD. Pictorial essay: coronary artery variants and anomalies. Indian J Radiol Imaging. 2009;19(1):49.

Imaging in Anomalies of Coronary Artery Termination

37

Sravan Nagulakonda

37.1 Case 37.1 37.1.1 Clinical Presentation A 3-year-old acyanotic child with failure to thrive and intermittent episodes of tachypnoea, excessive diaphoresis during feeds from 1 month. On cardiovascular examination, there is diffuse apex and a continuous murmur.

37.1.2 Chest Radiograph See Fig. 37.1.

37.1.3 Differential Diagnosis In an acyanotic child with extracardiac left to shunt above, differential diagnosis includes • • • •

Ruptured sinus of Valsalva aneurysm Coronary artery fistula Aortopulmonary window PDA

37.1.4 Echocardiography

Fig. 37.1  Chest radiograph reveals enlarged thymus, cardiomegaly with dilated right ventricle and left ventricle, enlarged pulmonary arteries, possible enlarged aortic pedicle, and pulmonary plethora suggesting extracardiac left to right

37.1.5 What Investigation Will You Do Next? CT angiography or catheter angiography for coronary artery anatomy (Table 37.1). In view of the advantages, ECG-gated MDCT is an optimal next investigation.

Enlarged LV with global LV dysfunction, dilated right. Dilated LAD with fistulous communication with RV

S. Nagulakonda (*) Department of Cardio Vascular Radiology and Endovascular Interventions, AIIMS, Ansari Nagar, New Delhi, India © The Author(s), under exclusive license to Springer Nature Singapore Pte Ltd. 2023 S. Sharma (ed.), Case-based Atlas of Cardiac Imaging, https://doi.org/10.1007/978-981-99-5620-3_37

567

S. Nagulakonda

568 Table 37.1  Gated CTA versus catheter angiography in evaluation of coronary arteries Advantages

Limitations

37.1.6 CT Angiography

Gated CTA Orthogonal reconstruction of entire course of the fistula from origin to termination Identification of additional fistula Ct measurement of neck of fistula as well as the anatomy and termination 3D measurement of neck Accurate measurement of diameters in multiple planes—for treatment planning The above are necessary for planning treatment Inability to quantify the shunt

Catheter angiography Shunt quantification Ability to perform diagnosis and treatment in the same setting Extending role—diagnosis to therapeutic

Lack of multidimensional assessment of the neck and termination site Optimal measurement of diameters for treatment planning Thrombosed components may be missed Difficult to assess proximal and distal neck in case of tortuosity

37.1.8 Differential Diagnosis of Coronary Cameral Fistula

See Fig. 37.2.

37.1.7 Final Diagnosis LAD: RV coronary artery fistula

1. Ruptured sinus of Valsalva 2. Aortopulmonary window

37  Imaging in Anomalies of Coronary Artery Termination

a

569

b

c

Fig. 37.2  Cardiac CT angiography reveals (a) and (b) dilated and tortuous course of LAD (arrow in a and b); (c) termination of the fistula into right Ventricle (arrow in c)

37.2 Case 37.2 37.2.1 Clinical Presentation Forty-year-old male with gradually progressive breathlessness. Echo showed dilated RA, RV, and pulmonary arteries with jet of flow from the right coronary sinus into RA. Further evaluated by cardiac CT. What are the imaging findings? (Fig. 37.3)

37.2.2 Final Diagnosis Ruptured sinus of Valsalva

37.2.3 Differentiating Features from Coronary Cameral Fistula Chest x-ray shows features of cardiomegaly with variable right and left heart chamber enlargement depending on the chamber into which sinus has ruptured. Features of pulmonary plethora and pulmonary venous hypertension may be seen. Simultaneous presence of pulmonary plethora and PVH is characteristic of an RSOV. Jet of flow may be seen from coronary sinus into any of the cardiac chambers most commonly from RCS into RV— MR more sensitive CT.

570

S. Nagulakonda

a

b

c Fig. 37.3 (a) CXR reveals cardiomegaly with biventricular enlargement and right atrial enlargement, dilated SVC, cephalization of pulmonary, and Kerley’s B lines. Cardiac CT angiography reveals (b) contrast

jet is from right coronary sinus into right ventricle across right atrium and tricuspid valve during diastole, (c) contrast jet into right atrium systole

37  Imaging in Anomalies of Coronary Artery Termination

571

37.3 Case 37.3

37.3.2 Final Diagnosis

37.3.1 Clinical Presentation

Aortopulmonary window

Thirty-five-year-old male with gradually progressive breathlessness. On Echo, aortopulmonary window was noted. What are the imaging findings? (Fig. 37.4)

37.3.3 Differentiating Features from Coronary Cameral Fistula Chest x-ray shows cardiomegaly with LV configuration. There is disproportionate dilatation of aortic root, prominent conus with pulmonary plethora and PAH.

a

b

c

d

Fig. 37.4 (a) CXR reveals cardiomegaly (LV configuration). Cardiac CT angiography reveals. (b) Pulmonary valve (PV) and aortic valve (AV) are seen separately with normal relation of PV and AV (PV is

anterior and to the left of aortic valve). (c, e) Pulmonary artery from RV and aorta from LV. (d–g) Aortopulmonary window (star in d–g)

572

S. Nagulakonda

e

f

g Fig. 37.4 (continued)

37.4 Spectrum of Imaging Abnormalities in Coronary Cameral Fistula 37.4.1 Case 37.4

37.4.2.1 Final Diagnosis LM – RA coronary fistula

37.4.3 Case 37.6

See Fig. 37.5

37.4.1.1 Final Diagnosis LAD – RV coronary fistula

37.4.2 Case 37.5 See Fig. 37.6

See Fig. 37.7

37.4.3.1 Final Diagnosis RCA – RV fistula with RCA aneurysm

37  Imaging in Anomalies of Coronary Artery Termination

573

a

b

c

d

Fig. 37.5  CXR reveals (a) Enlarged third mogul. Cardiac CT angiography reveals (b) dilated and tortuous LAD, (c, d) termination of fistula into right ventricle

574

S. Nagulakonda

a

b

Fig. 37.6  CXR (a) reveals no significant abnormality. Cardiac CT angiography reveals (b) LM to RA fistula (arrows)

37  Imaging in Anomalies of Coronary Artery Termination

575

a

b

c

e

d

f

Fig. 37.7  CXR reveals (a) cardiomegaly with pan chamber enlargement and pulmonary plethora. Cardiac CT angiography reveals (b) dilated RCA with a partially thrombosed saccular showing peripheral calcification (c) fistula termination into right ventricle (d) dilated pulmonary arteries e) VRT showing the course of the dilated and tortuous

course of RCA. Postoperative images—Topogram (f) and CT images (g, h) reveal closure device at the site of termination of fistula with contrast opacification of RA through the fistula suggesting residual patency of the fistula

576

g

S. Nagulakonda

h

Fig. 37.7 (continued)

37.5 Discussion

37.5.3 Classification

37.5.1 Introduction Anomalies of the coronary artery may be in number, origin, course, termination, or structure. Coronary artery fistulas are abnormalities in termination. It was initially described in 1865 by Krause [1]. They may be present at birth, which is the more common presentation, or may be acquired. It is an uncommon condition and constitutes only 0.2–0.4% of congenital cardiac anomalies [2]. In the general population also, this condition is extremely rare, with 2 out of 10,000 i­ndividuals having this anomaly. In patients who undergo angiography, it is more commonly seen [3–6].

When the communication is between the coronary artery and cardiac chamber, it is termed as coronary-cameral fistula and when it is with the systemic or pulmonary circulation, it is called coronary arteriovenous fistula [8]. CAF may be from the right coronary artery (RCA) in approximately 50% of patients, the left coronary artery (LCA) in approximately 42% of patients, and both the RCA and LCA in approximately 5% of patients [9]. In symptomatic patients, RCA is the culprit artery and LCA is predominantly involved in asymptomatic patients. Low-pressure structures are the major sites of drainage with drainage into the right ventricle seen in 41%, right atrium in 26%, and pulmonary artery in 17% [10, 11].

37.5.2 Embryology and Pathoanatomy

37.5.4 Imaging in Coronary Artery Fistulas

Coronary arteries terminate in multiple arborizing branches that invade the myocardium and form capillaries. In individuals with CAF, the coronary arteries directly communicate with the lumen of a cardiac chamber, the coronary sinus, the superior vena cava, pulmonary artery, or vein before capillary level [7]. When present at birth, abnormal connection between coronary artery and a cardiac chamber may represent intratrabecular spaces or sinusoids that are present in the embryologic life.

37.5.4.1 Goals of Imaging • Confirmation of diagnosis • Quantification of shunt and hemodynamic severity • Detection of complications • For planning treatment 37.5.4.2 Imaging Modalities Recommended Imaging Strategy for Isolated Coronary Artery Fistula

37  Imaging in Anomalies of Coronary Artery Termination

• For measuring the size of the fistula, associated physiological parameters and for establishing baseline, TTE should be the initial choice of investigation. • If the CAF is found to be significant and treatment is being planned, invasive catheterization, MRI or cardiac CT is needed for accurate anatomic localization of the origin and termination. • Cardiac catheterization or MRI may also play a complementary role to TTE in equivocal cases for ascertaining hemodynamic severity. • Imaging modalities can also be used for monitoring of ischemia post treatment.

37.5.4.3 Chest Radiographs In majority of cases, shunts are small and chest X-ray is unremarkable. With larger sizes and atypical locations, chest X-ray may reveal • Enlargement of the cardiac silhouette • Significant dilatation of the aortic root, out of proportion to rest of the cardiac structures • Increased pulmonary vascularity Diff diagnosis of a third mogul (Abnormal shadow in the area of left atrial appendage in the CAC triangle area): Left atrial appendage enlargement, asymmetric septal hypertrophy, dilated RVOT, and EMF

37.5.4.4 Computed Tomography Coronary Arteries CT evaluation in cases of CAF includes identification of the origin, complexity, and terminal drainage site of the fistula 1. Origin Clinical significance: Decision making related to treatment choices. Injury to adjacent arteries and structures can be avoided. While endovascular approach is used for more proximal shunts, surgical approach is preferred in distal shunts to avoid inadvertent embolization of branch vessels. Common imaging appearances: RCA is the artery that is most frequently involved. LAD is also commonly involved. Involvement of LCx is rare. When the fistula is present in proximal part of the artery, arteries are more tortuous, dilated and prone to atherosclerosis – steal phenomenon is also common. While in cases of distal fistula, artery is small in caliber, but tortuous. 2. Complexities Clinical significance: With increasing steal, there is increasing dilatation of the coronary arteries. Potential

577

rupture of the fistula is also a factor with increasing dilatation. Common imaging appearances: While single fistulas are more common, multiple sites of communication may also occur making the fistulous sites more prone aneurysm formation. Treatment options: Transcatheter closure: For simple fistulas with single communication Surgical management: Multiple complex communications; tortuous arteries and prominent aneurysm 3. Drainage Clinical significance: Communication with the pulmonary system is usually asymptomatic till adulthood. Common imaging appearances: Contrast shunt sign may be a reliable indicator of fistula patency and drainage site Dilation and tortuosity: More severe degrees of left to right shunting results in more tortuosity and dilatation of low-pressure drainage sites. Wider drainage sites have higher coronary steal. Treatment options: Transcatheter closure: Single drainage site Surgical management: Multiple complex communications; Wide drainage site; drainage to coronary sinus 4. Complications related to coronary artery fistulas: Aneurysm formation Noncoronary Cardiac Findings in Congenital Coronary Artery Fistulas 1. Chambers of the heart, aorta, and pulmonary arteries may have associated anomalies. 2. When in failure, chamber enlargement of pulmonary artery dilatation may be seen. 3. Reduction in ejection fraction and RWMA

37.5.4.5 Catheter Angiography • Origin course and termination of the fistula • Shunt quantification • Severity and reversibility of PAH 37.5.4.6 Cardiac MRI • Noninvasive assessment of hemodynamics related to CAF and features of myocardial ischemia Hemodynamics • Quantify the severity of shunt, ventricular wall motion abnormalities, and ejection fractions Myocardial Ischemia • Perfusion deficits and LGE

S. Nagulakonda

578

37.5.5 How Imaging Impacts Management? Imaging points relevant to management are mentioned under the heading COMPUTED TOPOGRAPHY in imaging modalities. Multiple Choice Questions Question 1: Coronary cameral fistula is (a) Coronary artery to coronary venous fistula (b) Coronary artery to any cardiac chamber (c) Coronary artery to pulmonary artery fistula (d) Coronary to SVC fistula Question 2: Coronary cameral fistula is seen most commonly from (a) Left Main (b) Right Coronary Artery (c) Left Circumflex Artery (d) Left Anterior Descending Artery Question 3: Most common drainage site of coronary cameral fistula is (a) Right atrium (b) Right Ventricle (c) Left atrium (d) Left ventricle Question 4: Screening modality of choice in a case of suspected coronary artery fistula is (a) Echocardiography (b) Computed tomography (c) MRI (d) Catheter angiography Question 5: In a case of coronary cameral fistula, computed tomography is useful in all of the following except (a) Origin and termination of fistula (b) Associated cardiac anomalies (c) Aneurysm formation if present (d) None Question 6: Catheter angiography has added advantage compared to computed tomography in (a) Shunt quantification and reversibility of pulmonary hypertension (b) RWMA and myocardial fibrosis (c) Associated cardiac anomalies (d) All the above Question 7: MRI has following advantages except (a) Quantify the severity of shunt (b) Identification of ventricular wall motion abnormalities and ejection fraction (c) Perfusion deficits and LGE (d) Easy to acquire than computed tomography

Question 8: Regarding management of coronary cameral fistula (a) To plan treatment for a significant fistula delineation of origin, course and exit should be confirmed with cardiac catheterization, CCT, or CMR. (b) Supplementary hemodynamic information when needed may be obtained by cardiac catheterization or CMR. (c) Coronary ectasia following CAF closure can persist and should be monitored for development of thrombus/ischemia. (d) All of the following Question 9: Imaging findings favoring transcatheter closure of coronary artery fistula include (a) Proximally located fistula (b) No complicated fistula (c) Single drainage site (d) All of the above Question 10: Imaging findings favoring surgical management over transcatheter closure (a) Distally located fistula (b) Tortuous artery (c) Wide drainage site (d) All of the above Answers 1 b

2 b

3 b

4 a

5 d

6 a

7 d

8 d

9 d

10 d

References 1. Krause W. Ueber den Ursprung einer akzessorischen A. coronaria aus der A. pulmonalis. Z Ratl Med. 1865;24:225–9. 2. Chen CC, Hwang B, Hsiung MC, et  al. Recognition of coronary arterial fistula by Doppler 2-dimensiona echocardiography. Am J Cardiol. 1984;53(2):392–4. 3. Barbosa MM, Katina T, Oliveira HG, et  al. Doppler echocardiographic features of coronary artery fistula: report of 8 cases. J Am Soc Echocardiogr. 1999;12(2):149–54. 4. Balanescu S, Sangiorgi G, Castelvecchio S, et al. Coronary artery fistulas: clinical consequences and methods of closure. Ital Heart J. 2001;2(9):669–76. 5. Kamineni R, Butman SM, Rockow JP, et  al. An unusual case of an accessory coronary artery to pulmonary artery fistula: successful closure with transcatheter coil embolization. J Interv Cardiol. 2004;17(1):59–63. 6. Athanasias DA, Van Ommen V, Bar F.  Coronary artery pulmonary artery fistula originating from the left anterior descending artery: a case report and literature review. Hell J Cardiol. 2002;43:78–81.

37  Imaging in Anomalies of Coronary Artery Termination 7. Schumacher G, Roithmaier A, Lorenz HP, et  al. Congenital coronary artery fistula in infancy and childhood: diagnostic and therapeutic aspects. Thorac Cardiovasc Surg. 1997;45(6):287–94. 8. Padfield GJ. A case of coronary cameral fistula. Eur J Echocardiogr. 2009;10:718–20. 9. Nakamura M, Matsuoka H, Kawakami H, et  al. Giant congenital coronary artery fistula to left brachial vein clearly detected by multi-detector computed tomography. Circ J. 2006;70(6):796–9.

579 10. Fujimoto N, Onishi K, Tanabe M, et al. Two cases of giant aneurysm in coronary-pulmonary artery fistula associated with atherosclerotic change. Int J Cardiol. 2004;97(3):577–8. 11. Lin FC, Chang HJ, Wen MS, Yeh SJ, Wu D. Multiplane transesophageal echocardiography in the diagnosis of congenital coronary artery fistula. Am Heart J. 1995;130(6):1236–42.

Imaging in Coronary Artery Aneurysms

38

Mansi Verma

38.1 Case 38.1

38.1.4 Final Diagnosis

A 57-year-old man, known hypertensive presented with episodic atypical chest pain of 6 months duration.

Multiple fusiform coronary artery aneurysms

38.1.1 Electrocardiography Normal

38.1.2 Echocardiography

38.1.5 Etiology Atherosclerosis (multisegment involvement and associated calcified and noncalcified plaques with relative sparing of LM)

38.1.6 Differential Diagnosis

Normal

38.1.3 What Investigation Will You Do Next?

Vasculitis (chronic stage): multiple aneurysms; usually located at coronary ostia with associated aortic wall thickening

CT coronary angiography was done further to rule out coronary artery disease. CT angiography (Fig. 38.1)

M. Verma (*) Department of Cardio Vascular Radiology and Endovascular Interventions, AIIMS, Ansari Nagar, New Delhi, India © The Author(s), under exclusive license to Springer Nature Singapore Pte Ltd. 2023 S. Sharma (ed.), Case-based Atlas of Cardiac Imaging, https://doi.org/10.1007/978-981-99-5620-3_38

581

582

M. Verma

a

d

b

RCA

e

c

LAD

f

LCX

g

Fig. 38.1  CT Angiography axial image (a) showing left main coronary artery with normal origin and course. Eccentric calcific and noncalcific plaques seen in distal LM and ostioproximal LAD with fusiform aneurysmal dilatation of proximal and mid LAD. No calcification/thrombus seen in aneurysmal part. (b) Fusiform aneurysm of mid RCA. Sagittal

oblique image (b, c) depicting fusiform aneurysms of LAD and LCX. Curved planar images (d–f) showing multiple fusiform coronary artery aneurysms with no significant coronary artery disease. Volume rendered image (g) depicting multiple fusiform aneurysms

38.2 Case 38.2

38.2.1 Final Diagnosis

Five years-old-child presented with fever and generalized body rash. Physical examination revealed bleeding lips and red eyes. ECHO revealed a large aneurysm of LMCA, and CT coronary angiography was done further to delineate the exact extent of the aneurysm (Fig. 38.2).

Fusiform aneurysm of LM and LAD

38.2.2 Etiology Kawasaki disease

38  Imaging in Coronary Artery Aneurysms

583

a

b

c

d

Fig. 38.2  CT angiography coronal oblique (a), sagittal oblique (b) and axial (c) image showing fusiform aneurysmal dilatation of proximal LM and proximal and mid LAD with surrounding thrombus and periph-

eral calcification. Distal LAD was normal. LCX was not visualized in AV groove. Multiplanar reconstruction (d) showing RCA was dilated in proximal and mid part with e/o of thrombus

584

M. Verma

38.3 Case 38.3

38.3.1 Final Diagnosis

A 23  year-old-woman with suspected vasculitis and raised ESR and CRP underwent CT angiography to look for aortic arch and branch vessel involvement (Fig. 38.3).

Ectasia of left circumflex artery

38.3.2 Etiology Takayasu arteritis (chronic stage)

a

c

b

d

Fig. 38.3  Axial (a), sagittal oblique (b) and coronal oblique (c) image depicting stenosis of LM with ectasia of proximal left circumflex artery. Axial (d) image revealing chronic thrombotic occlusion of distal infrarenal aorta with diffuse mural calcification

38  Imaging in Coronary Artery Aneurysms

585

38.4 Case 38.4

38.4.1 Final Diagnosis

A 67-year-old man with history of stent implantation in RCA 3 years earlier, presented with intermittent fever for 2 months. ECHO revealed a mass lesion involving the RCA. CT angiography was done further (Fig. 38.4).

Pseudoaneurysm of RCA after stent placement

38.4.2 Etiology Iatrogenic

a

b

T

c

d

T

Fig. 38.4  Coronal oblique (a). Sagittal oblique (b) and axial image (c) depicting a giant (11.6 × 4.6 × 4.5 cm) peripherally thrombosed pseudoaneurysm arising from the proximal-most RCA via a narrow neck. Coronal oblique image (d) depicting that the stents in RCA were occluded

M. Verma

586

38.5 Discussion 38.5.1 Introduction The frequency of coronary artery aneurysms varies from 0.3% to 5% in literature [1]. In the largest postmortem study, the prevalence was 1.4% and in the largest antemortem angiographic series, it was 4.9%; however, the rest of angiographic studies report prevalence of 0.37–2.53% [1].

38.6 Nomenclature 38.6.1 Coronary Artery Aneurysms Diameter of aneurysmal segment exceeds the diameter of normal adjacent coronary segments or the diameter of the patient’s largest coronary vessel by 1.5 times and involves less than 50% of the total length of the vessel [1].

38.6.2 Ectasia Diffuse dilatation (>1.5 the normal diameter) of the coronary arteries and involves 50% or more of the length of the artery [1].

38.7 Classification of Coronary Artery Dilatation (1) 38.7.1 Vessel Wall Composition True aneurysm: Vessel wall composed of three layers False aneurysm: Vessel wall composed of one or two layers. Usually, history of chest trauma or catheter based interventions is present.

38.7.2 Shape

Giant aneurysm: Maximal diameter exceeds 20 mm in adults or 8 mm in children [1]. Etiology and pathophysiology (Table 38.1): In western population, atherosclerosis aneurysms are most common (50%), followed by congenital (17%) and infectious causes (10%) [1, 2].

38.7.4 Clinical Characteristics and Complications Most of the patients remain asymptomatic with incidental detection. When symptomatic, the clinical manifestations depend on the underlying cause.

38.7.5 Imaging Assessment of Coronary Artery Aneurysm Anatomic location, maximal diameter, length, shape saccular or fusiform, number, morphologic characteristics, wall calcification, associated stenosis, relationship to surrounding structures, compression of surrounding structure, vessel wall (thickening or enhancement), thrombi, associated plaque formation and potential complications, including fistula formation, extrinsic mass compression, or evidence of active rupture, including hemopericardium. The imaging features of some common disease entities with coronary aneurysms are tabulated in Table 38.2.

38.7.6 Management of Atherosclerotic Coronary Aneurysms There is lack of randomized trials or large-scale data and most of the current recommendations are based on small case series, or anecdotal evidence. The optimal management in incidentally found CAA or coronary ectasia in the absence of coronary stenosis or occlusion is uncertain. Treatment strategies should be individualized on case to case basis [4].

Saccular aneurysm: Transverse> longitudinal diameter Usually seen distal to stenosis, often multisegmented, and more prone to thrombosis and rupture Fusiform aneurysm: Longitudinal> transverse diameter

38.7.6.1 Medical Management • Risk factor modification. • Antiplatelets or therapeutic anticoagulation: No high-­ quality evidence to support or contradict the use of an antiplatelet or antithrombotic regimen.

38.7.3 Diffuse Dilatation: ECTASIA

38.7.6.2 Indications of Intervention • Obstructive coronary artery disease • Evidence of embolization leading to myocardial ischemia • Evidence of enlargement of saccular coronary artery aneurysms • Increased risk of rupture

Type I: diffuse ectasia in two or three vessels Type II: diffuse ectasia in one vessel and localized disease (aneurysm) in another Type III: diffuse ectasia in only one vessel Type IV: coronary aneurysm in one vessel

587

38  Imaging in Coronary Artery Aneurysms Table 38.1  Etiology and pathophysiology of coronary artery aneurysms Age group Adults (>50 years) Childhood

Aneurysm or ectasia A/E

Pathogenesis Mechanical stress

A

Autoimmune, vasculitis

Young adults

E

Inflammatory mediators

Adults

E

Inflammatory mediators

Any age Childhood form, adult form

E E

High flow state Myocardial ischemia

Most are congenital In infants, death occurs early due to myocardial infarction In adult form, collateral vessels develop

Young adults

E

IL-6, CRP

Any age

A

Trauma/iatrogenic

Adults

A

Cocaine abuse

Adults

A

Microemboli to vasa vasorum, direct invasion of arterial wall Trauma because of oversized balloon, high inflation pressures Severe hypertension → endothelial damage

Ehlers Danlos syndrome, Marfan syndrome Infection with most common Staphylococcus aureus Clinical history is important

Cause Atherosclerosis Kawasaki disease Inflammatory disorders Compensatory dilatation Fistula Coronary anomalies (ALCAPA)

Miscellaneous Connective tissue disorders Mycotic

Table 38.2  Imaging features of some common disease entities with coronary aneurysms Cause Atherosclerosis

Kawasaki disease

Inflammatory causes Mycotic

Iatrogenic

Imaging features Usually multiple, fusiform, and involve more than one coronary artery. The RCA is the most frequently involved vessel (40–61%) followed by the LAD and LCX. Main trunk involvement is rare (0.1–3.5%) [2] Coronary artery dilatation and aneurysm, premature atherosclerosis and stenosis, thrombosis, or occlusion. The left main coronary artery is involved in 12% of the cases, the RCA in 3%, and both arteries in 8% [2] Early phases: Consistent with arterial wall enhancement and thickening Later phases: Stenosis and aneurysms or ectasia The right coronary artery is MC involved (40%). Have lobulated contour or saccular shape (54.2%) with thick wall or mural thrombus. (87.5%).Associated abnormal appearance of the pericardium with either pericardial fluid, thickening or loculation is common (79.2%) [3] Single, saccular, usually at site of intervention

Comments Most common cause of CAE/ CAA Most common cause of CAA in childhood and in Japan Takayasu arteritis, SLE, Wegener granulomatosis RA, Behcet, PAN

Clinical history

38.7.6.3 Intervention Outcome data on PCIs in patients with CAA or CAE are sparse [4]. • PCI of an aneurysmal/ectatic vessel in background of acute MI is associated with lower procedural success and a higher incidence of no-reflow and distal embolization. • Saccular aneurysms and small pseudoaneurysms not involving a major side branch: covered stent exclusion • Saccular or fusiform aneurysms that involve a major side branch: balloon or stent-assisted coil embolization, or with surgical exclusion • For CAA involving the left main coronary artery, multiple CAAs, and for SVGAs: surgical resection is considered the first-line therapy • Large or rapidly expanding SVGAs or in those causing symptomatic external compression: percutaneous closure with Amplatzer occluders or coil embolization with or without PCI of the native grafted vessel is a feasible alternative to surgery [4].

M. Verma

588

Multiple Choice Questions Question 1: The most common cause of coronary artery aneurysm is (a) Kawasaki disease (b) Inflammatory (c) Atherosclerosis (d) Iatrogenic Question 2: The most common cause of coronary artery aneurysm in children is (a) Kawasaki disease (b) Inflammatory (c) Atherosclerosis (d) Iatrogenic Question 3: The reported frequency of coronary artery aneurysm is (a) 5–10% (b) 10–15% (c) 0.3–5% (d) 15–20% Question 4: Coronary aneurysm in one vessel is what type of coronary dilatation: (a) Type I (b) Type II (c) Type III (d) Type IV Questions 5: By definition, ectasia is (a) Diameter > 2 times; Length  1.5 times; Length > 50% (c) Diameter > 3 times; Length > 50% (d) Diameter > 2 times; Length > 50% Questions 6: Atherosclerotic aneurysms are usually of which shape (a) Saccular (b) Fusiform (c) Both (d) None of the above Question 7: The vessel relatively spared in atherosclerotic coronary ectasia/aneurysm: (a) RCA (b) LAD

(c) LM (d) LCX Question 8: The vessel most commonly involved by mycotic aneurysm is (a) RCA (b) LAD (c) LM (d) LCX Question 9: First-line therapy for aneurysm of left main coronary artery is (a) Covered stent exclusion (b) Balloon assisted coiling (c) Stent assisted coiling (d) Surgical Question 10: The preferred intervention in case of saccular aneurysm not involving side branch is (a) Covered stent exclusion (b) Balloon assisted coiling (c) Stent assisted coiling (d) Surgical Answers 1 c

2 a

3 c

4 d

5 b

6 b

7 c

8 a

9 d

10 a

References 1. Díaz-Zamudio M, Bacilio-Pérez U, Herrera-Zarza MC, Meave-­ González A, Alexanderson-Rosas E, Zambrana-Balta GF, et  al. Coronary artery aneurysms and ectasia: role of coronary CT angiography. Radiographics. 2009;29(7):1939–54. 2. Kawsara A, Gil IJN, Alqahtani F, Moreland J, Rihal CS, Alkhouli M. Management of coronary artery aneurysms. J Am Coll Cardiol Intv. 2018;11(13):1211–23. 3. Restrepo CS, Gonzalez TV, Baxi A, Rojas CA. Infected (“Mycotic”) coronary artery aneurysm: systematic review. J Cardiovasc Comput Tomogr. 2020;14(6):e99–e104. 4. Jeudy J, White CS, Kligerman SJ, Killam JL, Burke AP, Sechrist JW, Shah AB, Hossain R, Frazier AA. Spectrum of coronary artery aneurysms: from the radiologic pathology archives. Radiographics. 2018;38(1):11–36.

Part V Miscellaneous

Advances in Cardiovascular MRI in Heart Failure

39

Amit Ajit Deshpande and Manish Shaw

39.1 Introduction [1, 2]

39.2 Advances in CMRI

Heart failure (HF) is a complex clinical syndrome caused by either structural or functional cardiac abnormalities, leading to decrease in Cardiac output or elevated intracardial pressure at rest or stress condition. Heart failure can be divided into three categories depending upon Left Ventricular Ejection Fraction (LVEF): (a) HF with preserved Ejection fraction (HFpEF), LVEF >50%; (b) HF with reduced ejection fraction (HFrEF), LVEF